Board

Download as doc, pdf, or txt
Download as doc, pdf, or txt
You are on page 1of 160

1.

Some clinical genetic diseases in humans are disorders transmitted in patterns that can be
predicted from knowledge of how genetic information is transmitted to offspring during meiosis
(Mendel's Law). Which of the following statements is true regarding psychiatric disorders?
A. They have etiological homogeneity.
B. They have well-defined phenotypes.
C. They have reproducibly associated chromosomal rearrangement syndromes.
D. They have clear-cut inheritance patterns.
E. They will be easier to understand based on the findings of the Human Genome Project

2. Which of the following personality disorders shows evidence of genetic transmission, on the basis of
family risk, twin, and molecular studies?
A. Antisocial.
B. Avoidant.
C. Borderline.
D. Dependent.
E. Schizotypal

3. ∆The first stage of Erikson’s psychosocial development is


A. Identity vs confusionl.
B. intiative vs guilt.
C. Trust vs mistrust.
D. Intimacy vs isolationl.
E. Integrity vs despair.

4. Defense mechanisms include all the following EXCEPT


A. Projection.
B. Rationalization.
C. Repression.
D. Resolution
E. Not be too aggressive with questions about suicidal ideation, because the material will come out as
necessary over time.

5. ∆Regarding the physical layout of an interview room, the psychiatrist should attend to all of the
following except
A. Chairs for patient and physician should be of equal height.
B. There should be a desk between patient and physician for appropriate therapeutic distance.
C. A playroom is advised for interactions with children.
D. In the emergency room, the physician should have quick access to the door.
E. Extraneous noise should be minimized or avoided.

6. The behavior of a patient who remains silent during an interview illustrates which type of
resistance?

A. Conscious.
B. Ego.
C. Id.
D. Superego.
E. Countertransference.

7. After listening to several examples of similar behavior in the patient's life story, the psychiatrist
offers the observation "When you can't perform up to your own high standards, you appear to act to
try to please someone else." This response by the psychiatrist is an example of

A. Reflection.
B. Interpretation.
C. Transition.
D. Self-disclosure.
E. Facilitation.

8. ∆Which of the following is not considered a perceptual disturbance?

A. Derealization.
B. Illusion.
C. Depersonalization.
D. Delusion.
E. Hallucination.

9. After fully exploring the patient's personal history, the psychiatrist proceeds to the next history
item by asking, "Is there a family history of psychiatric illness, for example, suicidal behavior, mental
retardation, or anxiety, mood, psychotic, substance abuse, or personality disorder?" This would be an
example of

A. Excessive direct questions.


B. Preemptive topic shift.
C. Run-on questioning.
D. Put-down.
E. Checklist questioning.

10. Regarding physician contacts with the patient's relatives or friends, the physician should generally

A. Talk to family and friends first for collateral history.


B. Agree to withhold from the patient information gained from collateral sources.
C. Suspend the usual practice of physician–patient confidentiality when imminent dangerousness is
present.
D. Withhold from the patient information learned from collateral sources.
E. Disclose the content of the interview to concerned family members, even in the absence of an
emergency.

11. When interviewing a depressed or suicidal patient, the psychiatrist is advised to do all of the
following except

A. ∆Begin with assessment of physical appearance and motor behavior.


B. ∆Thoroughly explore neurovegetative signs or symptoms and their effect on patient function.
C. ∆ Be attentive to recent psychosocial stressors (especially losses) and recent anniversaries of
significant past losses.
D. Allow the patient to set the pace of the interview, avoid excessive engagement, and tolerate long
silences.
E. ∆Assess both suicidal cognitions and the patient's ability to control self-destructive impulses.

12. On mental status examination, the patient's stream of thought reveals a lack of goal directedness,
excessive details, and difficulty with closure. This thought pattern is called
A. Circumstantiality.
B. Tangentiality.
C. Thought blocking.
D. Perseveration.
E. Neologism.

13. ∆Delusions that cannot be understood by other psychological processes are referred to as primary
delusions. Which of the following is an example of this type of delusion?

A. Poverty.
B. Nihilism.
C. Thought insertion.
D. Persecution.
E. Guilt.
Gent

14. Findings of the Human Genome Project

A. Have led to biological tests for the presence or absence of a clinical disorder.
B. Imply that genetic linkage studies will be less difficult as a result of the diagnostic precision in our
field.
C. Have not included genes responsible for complex psychiatric disorders.
D. Include a mutation rate in males that is twice as high as that in females.
E. Include discovery of more than 200,000 total genes, more than was expected.

15. A 24-year-old woman with no history of medical or psychiatric disorders sought help in the
psychiatric outpatient clinic for anxiety related to her mother's cancer treatment. Her mother
eventually went into remission, and the patient terminated treatment; however, a few months later
she recontacted the clinic, reporting that she had experienced more anxiety while participating in a
clinical trial as a healthy subject. The research trial involved administration of cholecystokinin. This
woman is at risk for which of the following disorders?

A. Bipolar disorder.
B. Depression.
C. General anxiety disorder.
D. Panic disorder.
E. Posttraumatic stress disorder.

16. A methodological limitation of adoption research, in terms of its ability to separate genetic from
environmental influences, is the fact that

A. Most children are adopted away at birth.


B. The environment begins at birth.
C. Adopted children cannot be considered a representative sample of the population.
D. Adoptive parents often have as much psychiatric illness as biological parents.
E. A false genetic hypothesis (type I error) has been accepted.

17. Sigmund Freud's hierarchy of threats during early childhood are experienced in which of the
following sequences?
A. Helplessness, separation, castration anxiety, punishment by guilt.
B. Separation, helplessness, annihilation, castration anxiety.
C. Annihilation, separation, castration anxiety, punishment by guilt.
D. Separation, helplessness, punishment by guilt, castration anxiety.
E. Helplessness, separation, annihilation, punishment by guilt.

18. Rene Spitz's genetic field theory specifies the appearance of three "organizers" in which
developmental sequence?

A. Smiling response, separation anxiety, signal for "no."


B. Separation anxiety, stranger response, signal for "no."
C. Stranger response, separation anxiety, signal for "no."
D. Smiling response, stranger response, signal for "no."
E. Smiling response, stranger response, separation anxiety.

19. Common developmental themes of adolescence include which of the following?

A. Dependence vs. peer group.


B. Family vs. privacy.
C. Idealization vs. intellectualized control.
D. Identity, role, and character.
E. Identity vs. inferiority.

20. Arnold Gesell's cross-sectional scheme describes the development of behavior across all of the
following sectors except

A. Motor.
B. Adaptive.
C. Language.
D. Affective.
E. Personal–social

21. John Bowlby's five "components" of attachment behavior (as opposed to responses to separation)
include

A. Sucking, protest, clinging, following, smiling.


B. Sucking, clinging, following, crying, smiling.
C. Clinging, sucking, despair, following, smiling.
D. Clinging, grief, despair, following, sucking.
E. Protest, sucking, clinging, following, crying.

22. Margaret Mahler's separation–individuation subphases are specified in which sequence?

A. Differentiation, object constancy, and practicing.


B. Object constancy, practicing, and differentiation.
C. Object constancy, differentiation, and practicing.
D. Differentiation, practicing, and object constancy.
E. Practicing, differentiation, and object constancy.

23. Lewis and Michelson's structural analysis of emotions refers to


A. Emotional state, expression, and experience.
B. Emotional expression, experience, and reciprocity.
C. Emotional state, experience, and reciprocity.
D. Emotional expression, reciprocity, and mastery.
E. Emotional state, expression, and mastery.

24. The emergence of stranger anxiety is regularly observed by age

A. Younger than 4 months.


B. 4–8 months.
C. 8–12 months.
D. 12–16 months.
E. 16–24 months.

25. ∆ Ainsworth's "strange situation procedure" is used to assess the quality of attachment. Which of
the following describes a securely attached child?

A. Mild protest with mother's departure, seeks mother upon her return, easily placated by mother.
B. Mild protest with mother's departure, no approach to mother upon her return.
C. No protest on mother's departure, no approach to mother upon her return.
D. Marked upset with mother's departure, readily accepts mother's effort to comfort upon her return.
E. Indifference to mother's departure, rejection of mother on her return.

26. Regarding Offer and Offer's study of adolescent development, the three specified developmental
routes are

A. Continuous growth, surgent growth, and tumultuous growth.


B. Tumultuous growth, surgent growth, and early maturational growth.
C. Surgent growth, tumultuous growth, and early maturational growth.
D. Continuous growth, erratic growth, and surgent growth.
E. Tumultuous growth, early maturational growth, and continuous growth

27. A husband and wife frequently engage in unproductive arguments. Even when the couple
converses without arguing, the husband reveals little about his past relationships, and what he does
divulge comes out a bit disorganized. Particularly frustrating to the wife is the husband's practice,
during the couple's arguments, of talking in a nonlinear, general fashion, accusing her of doing
everything "wrong," just like his mother. According to the topographical model of the mind, which of
the following is true about the husband in this example?

A. His system position is probably consistent with primary process.


B. His system position is probably consistent with secondary process.
C. His dynamic position is probably not repressed.
D. His system position is probably word-oriented and declarative.
E. He is probably motivated by the reality principle.

28. A husband is plagued by feelings of incompetence. When he comes home, he scans the house for
any sign that it is his wife and not himself who is incompetent. He attacks her on flimsy pretexts,
assuming the role of self-righteously competent spouse saddled with an incompetent wife. Not only
does the wife feel attacked and upset, she also believes that she is incompetent, even though others
disagree with her self-assessment. The psychological defense or process illustrated in this example is

A. Denial.
B. Projection.
C. Projective identification.
D. Reversal.
E. Undoing.

29. To facilitate transference, the psychotherapist should

A. ∆Ask a lot of questions to show interest.


B. ∆Avoid talking unless asked a question.
C. ∆ Discuss a personal opinion to facilitate disclosure by the patient.
D. Wait for most details, because the manner in which the patient reports material is also a clue.
E. ∆Not be too aggressive with questions about suicidal ideation, because the material will come out
as necessary over time.

30. Hunger is an example of a patient's drive to bond with another person, according to which theory
of the mind?

A. Self psychology.
B. Object relations.
C. Classical structural.
D. Topographical.
E. Psychosexual.

31. A 44-year-old woman in outpatient psychodynamic psychotherapy is discussing a dream she had
the night before. The psychotherapist engages the patient in a discussion about the dream's mystical
and cross-cultural aspects, in accordance with a leading theory which postulates that a common set of
ideas, symbols, and images are shared by all humankind. This psychotherapy is likely based on the
work of

A. Adler.
B. Horney.
C. Jung.
D. Rank.
E. Sullivan.

32. ∆An advantage of (MRI) over (CT) is that MRI. . .

A. Provides better detection of calcified brain lesions.


B. Is less anxiety provoking.
C. Is less expensive.
D. Provides better visualization of lesions in the brain stem.
E. Requires a shorter period of patient cooperation.

33. A supplemental laboratory/diagnostic test recommended for evaluating physical conditions in


nondemented, mood-disordered patients is

A. Chest X ray.
B. Electrocardiogram.
C. Complete blood count.
D. Magnetic resonance imaging scan.
E. Serum vitamin B12 and folate levels.

34. ∆Prior to starting a patient on lithium carbonate, the psychiatrist should order which of the
following tests?

A. Reticulocyte count.
B. Platelet count.
C. Erythrocyte sedimentation rate.
D. Liver function tests.
E. Thyroid function tests.

35. ∆A single, 53-year-old man is found lying on the floor in his apartment, having apparently lost
consciousness. In the emergency room, he is alert, appears oriented, and reports no complaints;
however, he has a large bruise on the left side of his head above the ear. He is admitted overnight for
observation and in the morning is noted to be lethargic and disoriented, despite having taken no
medications. Which of the following tests would be contraindicated?

A. Carotid ultrasound.
B. Computed tomography.
C. Electroencephalogram.
D. Lumbar puncture.
E. Magnetic resonance imaging.

36. Research advantages of functional magnetic resonance imaging (fMRI) over positron emission
tomography (PET), single photon emission computed tomography (SPECT), and regional cortical blood
flow (rCBF) studies include all of the following except

A. fMRI offers superior spatial/temporal resolution.


B. fMRI avoids exposure to ionizing radiation.
C. fMRI does not need a source of radiopharmaceuticals.
D. fMRI does not have the same degree of limitations on multiple studies.
E. ∆fMRI can be used by patients with pacemakers

37. Clinical scales on the Minnesota Multiphasic Personality Inventory–2 include all of the following
except

A. Hypochondriasis.
B. Hysteria.
C. Lie.
D. Psychopathic deviance.
E. Psychasthenia.

38. Psychometric testing instruments notable for their value in assessment of a wide variety of
symptom patterns include all of the following except

A. Minnesota Multiphasic Personality Inventory–2 (MMPI-2).


B. Symptom Checklist–90—Revised (SCL-90-R).
C. Brief Psychiatric Rating Scale (BPRS).
D. Hamilton Rating Scale for Depression (HRSD).
E. Millon Clinical Multiaxial Inventory—III (MCMI-III).
39. Regarding the Hamilton Rating Scale for Depression (HRSD), all of the following are true except

A. It contains 17–24 items.


B. It has 3- to 5-point severity scales for each item.
C. It is weighted toward the neurovegetative rather the cognitive symptoms of depression.
D. It is often used in conjunction with the Beck Depression Inventory (BDI).
E. It is self-administered.
40. Regarding the Wisconsin Card Sorting Test (WCST), all of the following are true except

A. It is a test of conceptualization.


B. It requires the subject to match cards on the dimensions of color, number, or form.
C. It requires the subject to derive "rules" based on interpretations of patterned visual stimuli.
D. The examiner changes the "rule" after 10 correct sorts.
E. The examiner prompts the subject that the rule change is imminent.

41. Screening instruments that may be administered rapidly for prompt and efficient assessment of
personality in a managed care system include all of the following except

A. International Personality Disorder Examination Screen (IPDE-S).


B. Iowa Personality Disorder Screen.
C. Self-directedness subscale from the Temperament and Character Inventory.
D. Screen for personality disorders developed from the Inventory of Interpersonal Problems.
E. Minnesota Multiphasic Personality Inventory–2 (MMPI-2).

42. Instruments used for the assessment of psychodynamics and patient enabling factors include all
of the following except

A. Rorschach.
B. Thematic Apperception Test (TAT).
C. Minnesota Multiphasic Personality Inventory–2 (MMPI-2).
D. Social Adjustment Scale—Self-Report (SAS-SR).
E. Millon Clinical Multiaxial Inventory—III (MCMI-III).

43. A 49-year-old Korean Woman recently immigrated to the United States. She understands
very little English and is having difficulty making friends in her neighborhood. As a result of
childhood trauma, she finds it hard to experience intimacy in serious relationships. She has a
significant family history of depression. In the Biopsychosocial model, which of the following
would be considered a socio-cultural factor:
A. Her medical history
B. Her aversion to intimacy after childhood trauma
C. Her age and gender
D. Her family history of depression
E. *Her difficulties assimilating into a new culture

44. While cross-examining a female defense witness, a male attorney becomes annoyed. The
witness remains calm. The attorney grows more frustrated, then accuses the witness of being
defensive and asks why she is so angry with him. This reattribution of one’s own emotion onto
another is an example of:
A. Transference
B. Countertransference
C. *Projection
D. Displacement
E. Reaction Formation

45. Freud’s theories of normal human development are based on:


A. *Application of findings from abnormal adult patients
B. Direct observation of a number of children
C. His studies with “Little Albert” and attachment
D. Application of Classical Conditioning to children
E. The principle of Spontaneous Recovery in dreams
46. In Systematic Desensitization, a client is taught a response that is incompatible with anxiety,
such as relaxation. The events that cause anxiety are ranked in a hierarchy of increasing severity.
The client is then asked to visualize the anxiety-provoking events, in increasing severity, while
trying to relax at each step. This therapy is a direct application of:
A. *Classical Conditioning
B. Behavior Modification
C. Operant Conditioning
D. Air Conditioning
E. Stimulus Generalization

47. According to John Bowlby and theories of attachment, which of the following statements is
TRUE:
A. Attachment occurs only to animate objects
B. Abusive parenting prevents attachment between mother and child
C. Children with normal attachment to their parents don’t experience a fear of darkness
D. *Failure to achieve attachment can lead to difficulties with trusting others
E. The terms “bonding” and “attachment” are synonymous

48. A teenager has a “gut”, strong reaction to a mild reprimand by her boss at “Pizza World”.
Adult employees, who receive the same admonishment, have a much milder reaction. This
differential adolescent reaction is consistent with functional imaging studies that demonstrate:
A. *Greater activity in the amygdala of teens
B. Greater activity in the frontal lobes of teens
C. Greater activity in the nucleus accumbens of teens
D. Decreased activity in the cerebellum of teens
E. Decreased activity in the hippocampus of teens

49. According to Erikson, the period of Young Adulthood is marked by a shift towards:
A. *Intimacy, prompted by loneliness and an absence of committed love
B. Identity, prompted by a move away from the family of origin
C. Role Confusion, prompted by a desire to explore sexual orientation
D. Industry, prompted by a need for productivity at work
E. Generativity, prompted by a need for productivity at work

50. After being given the news that he has only hours to live, a man becomes angry with his
physician and refuses to pay his bill, claiming the physician is incompetent. According to
Kubler-Ross, this reaction typically (but not always) appears at which stage in the Grieving
Process:
A. 1st Stage
B. *2nd Stage
C. 3rd Stage
D. 4th Stage
E. 5th Stage

51. An 85-year-old woman is admitted to the ICU after a hip fracture. She is found to be
incompetent. She is widowed and has 2 living children and a younger brother. Her children
want her to undergo a hip-replacement surgery. Her brother, and his son (her nephew) are
opposed to the procedure. Whose wishes should be honored according to proxy statutes?
A. Her brother
B. *Her children
C. Her nephew
D. Her spouse
E. Her pastor

52. A move to satisfy basic needs is called:

A. drive*
B. instinct
C. will
D. motivation
E. need

53. Non-goal directed movement carried out in a uniform way is called:

A. perseveration
B. stupor
C. stereotypy*
D. vorbeireden
E. mitmachen

54. Movement of body in response to light pressure until the movement is halted is called:

A. parakinesia
B. Mitgehen*
C. blocking
D. waxy flexibility
E. negativism

55. ∆Dulling of the sensitivity of the normal emotional response is called:

A. apathy
B. blunted affect*
C. dissociation
D. flattened affect
E. incongruity

56. Asymbolia is :

A. inability to write
B. inability to read
C. inability to recognize mathematical symbols
D. inability to recognize writing*
E. both A and B

57. Poverty of thought as expressed in language is called:

A. alogia*
B. dysphonia
C. logoclonia
D. paralogia
E. agrammatism
58. Vorbereiden is also known as:

A. logoclonia
B. coprolalia
C. talking past the point*
D. paralogia
E. schizophasia

59. Vorbereiden is characteristic of:

A. schizophrenia
B. Ganser syndrome*
C. depression
D. delusional disorder
E. schizotypal personality disorder

60. ∆Cotard’s syndrome is also known as:

A. nihilism*
B. reference
C. persecution
D. love
E. infestation

61. The core features of catatonia include all the following EXCEPT:

A. posturing
B. stereotypies
C. waxy flexibility
D. delusions*
E. ambitendence

62. Sensory distortions include all the following EXCEPT:


A. hyperaesthesias
B. hypoaesthesias
C. changes in quality
D. dysmegalopsia
E. illusions*

63. Features of Pseudohallucinations include the following EXCEPT:

 experienced as emanating from within the mind


 they lack the substantiality of normal perception
Clocated in objective space*
D.unwilled, and not subject to conscious control or manipulation
retention of insight

64. Spastic repetition of syllables is called:

A. logoclonia*
B. echolalia
C. dysarthria
D. logorrhea
E. flight of idea

65. Which of the following statistical tests is most appropriately used to evaluate differences
among mean depressive total scores of women in three different depressive groups?
A. dependent t-test
B. chi-square test
C. analysis of variance*
D. independent t-test
E. Fisher’s exact test

66. .In a cohort study, the ratio of the incidence rate of mortality among women who suffer
from depression to the incidence rate of mortality among women without depression is the
A. attributable risk
B. odds-risk ratio
C. incidence rate
D. prevalence ratio
E. relative risk*

67 . To estimate the relative risk in a case-control study, which of the following is calculated?

A. attributable risk
B. odds-risk ratio*
C. incidence rate
D. prevalence ratio
E. sensitivity

68 Questions that inform the physician about the cultural context of a patient’s
illness narrative were developed and championed by ∆ Clean Champion ∆
A. Hans Selye
B. Walter Cannon
C. John Bowlby
D. Arthur Kleinman*
E. Sigmunf Freud

69. Administration of what substance elicited the same behavioral effects as


mating in FEMALE prairie voles?
A. Dopamine*
B. Oxytocin
C. Substance P
D. Leptin
E. Serotonin
70. A patient you have never met before insists that she knows you and
elaborately describes your first meeting in the hospital’s outpatient clinic a
few weeks earlier (You have actually never met her). On rounds the next day she
insists you and she were classmates in high school years earlier. This
represents:
A. Confabulation*
B. Emergence
C. Priming
D. Conformity
E. Delusion

71. The Porteus Maze test (tracing the path out of a maze without reversing your
direction, crossing a line or lifting your pencil) is used to assess which
cognitive operation?
A. Reasoning*
B. Memory
C. Attention
D. Arousal
E. Concentration

72. Which of the following might be expected in someone with damage to the
basal-orbital frontal cortex?
A. Anterograde amnesia
B. Impulsivity*
C. Apathy
D. Blindsight
E. Retrograde amnesia

73. Anton Mesmer modeled his explanation of what we would today call hypnosis on
the pioneering work of:
A. Charcot
B. James
C. Newton*
D. Gallileo
E. Kraepelin

74. The Hawthorne Effect is:


A. A change in performance related to environmental manipulations*
B. A synonym for the placebo effect
C. The conformity of individuals to group norms
D. The hopelessness that follows inescapable punishment
E. A change in performance related to lack of interest

75. Your genotype most directly shapes the environment you are exposed to at
what age?
A. Birth
B. Latency years (pre-pubertal)
C. Teen years
D. Adulthood*
E. Toddler years

76. Rapid, repeated presentations of an unconditioned stimulus resulting in a


transient reduction in its capacity to elicit an unconditioned response is known
as:
A. Habituation*
B. Extinction
C. Blocking
D. Generalization
E. Discrimination

77. Smoking to prevent withdrawal symptoms is an example of:


A. Punishment
B. Positive reinforcement
C. Negative reinforcement*
D. Counteractive drug response
E. None of the above

78. Defense mechanisms are typically:


A. Psychopathological
B. Psychotic
C. Self-punitive
D. Out of awareness*
E. Conscious

79. Which of the following neurotransmitters seems most implicated in aggressive


behavior?
A. Dopamine
B. Norepinephrine
C. Substance P
D. Serotonin*
E. Glutamate

80. According to sociobiologic explanations (e.g. evolutionary explanations) of


aggression, males are more aggressive than females because they
A. Use more alcohol
B. Are more likely to have sustained brain damage
C. Have been punished more severely as children
D. Must compete for reproductive opportunities!!*
E. Are more muscular

81. A synchronized EEG (electroencephalogram) and an attenuated but present EMG


(electromyogram) describe which of the following states?
A. Wake
B. NREM*
C. REM
D. A and C
E. None of the above

82. Penile tumescence is associated with which one of the following


physiological states during sleep?
A. Absent skeletal muscle tone*
B. Homeothermic temperature regulation
C. Reduce brain O2 consumption
D. Regular respiratory rate
E. Regular heart rate
83. The major function of sleep is believed to be:
A. Avoidance of predation
B. Conservation of energy
C. Gene expression
D. Enhancement of synaptic efficacy*
E. None of the above

84. Which of the following abnormalities are observed in sleep studies of many
depressed persons?
A. Reduced REM latency
B. Decreased REM activity and density
C. Increased REM as a percent of total sleep time
D. Shortening of the 1st REM episode
E. A&C*

85. Attempts to quantify stress in humans have used which measure:


A. Life change units*
B. Gastric ulceration
C. Tobacco use
D. Capacity to delay gratification
E. Alcohol use

86. Exhaustion of the adrenal cortex due to chronic stimulation is most likely
to underlie
A. Conditioned fear
B. Stress analgesia
C. Autoimmune illness*
D. Stress-induced alcohol abuse
E. Stimulus generalization

87. Immunosurveillance of cells that develop cancerous predispositions (the so


called rogue cell) is thought to be most significantly reduced by:
A. Oxytocin
B. Serotonin
C. Cortisol*
D. Dopamine
E. Substance P

88. “Acquisition and extinction are quick” describes which schedule of


reinforcement?
A. Fixed ratio*
B. Variable ratio
C. Fixed interval
D. Variable interval
E. B&C

89. Slot machines are an example of what type of schedule of reinforcement?


A. Fixed ratio
B. Variable ratio*
C. Fixed interval
D. Variable interval
E. C&D

90. Which neuroanatomical structure appears to play a critical role in mediation


of behavior by rewards?
A. Suprachiasmatic nucleus
B. Hypothalamus
C. Nucleus accumbens*
D. Raphe nuclei
E. Locus coereleus

91. The use of disulfiram (Antabuse) in the treatment of alcohol dependence


represents which type of approach?
A. Antagonist
B. Agonist
C. Aversion*
D. Mixed
E. Reverse agonist

92. Initial and ordinary (self-protective) pain sensations are subserved by


which neurotransmitter in the dorsal horn?
A. CCK
B. Brain derived neurotrophic factor
C. Glutamate
D. Substance P*
E Serotonin
93. The major method used to investigate the unconscious mental processes of a patient in psychoanalysis
is:

(A) symbolism
(B) repression
(C) free association*
(D) primary process
(E) examination of the preconscious

94. All of the following statements about defense mechanisms are true EXCEPT

(A) they are used to keep conflicts out of consciousness


(B) they are unconscious mental techniques
(C) they decrease anxiety
(D) they are always manifestations of immature functioning*
(E) they help maintain an individual’s sense of self-esteem

95. ∆The defense mechanism used when unacceptable feelings are prevented from reaching
awareness is known as

(A) Repression*
(B) regression
(C) displacement
(D) rationalization
(E) isolation of affect

96. ∆A previously toilet-trained child is hospitalized and begins wetting the bed again. This is
an example of

(A) isolation of affect


(B) displacement
(C) projection
(D) regression*
(E) denial
97. All of the following statements concerning psychoanalysis are true EXCEPT

(A) the therapist plays an active role*


(B) treatment is usually conducted 4 times/week
(C) treatment is usually conducted for 3-4 years
(D) inhibition of free association is called resistance
(E) resistance blocks unconscious ideas that are unacceptable to the conscious mind

98. A patient’s reexperiencing of feelings about the parent with the therapist during
psychoanalysis is known as

(A) interference
(B) resistance
(C) association
(D) transference*
(E) cognitive dissonance

99.Which of the following is a function of the superego?

(A) maintaining relationships to the outside world


(B) reality testing
(C) maintaining object relationships
(D) controlling secondary process thinking
(E) controlling id impulses*

100. The 60% monozygotic twin concordance for schizophrenia is evidence for the role of

(A) heredity
(B) environment
(C) either A or B in different studies
(D) both A and B at the same time*
(E) neither A nor B

101. Problem solving mental activity is known as

(A) thinking*
(B) perception
(C) attention
(D) fantasy
(E) memorization

102. All of the following would be considered symptoms of mental illness except:

A. Loss of interest
B. Poor sleep
C. Thoughts of death
D. Psychomotor retardation*
E. Poor concentration
103. Which psychiatric conditions have been proposed to be classified from a dimensional
rather than a categorical perspective?

A. Personality disorders*
B. Psychotic disorders
C. Dementias
D. Sleep disorders
E. Mood disorders

104. ∆ “Axis III” illnesses include which of the following?

A. Diabetes*
B. Major depression
C. Borderline personality disorder
D. Obsessive-compulsive disorder
E. Obsessive-compulsive personality disorder

105. Which of the following statements about myelin is TRUE:

A. astrocytes form myelin around axons in the peripheral nervous system


B. myelinated axons have smaller Length Constants compared to unmyelinated axons
C. in the central nervous system, myelin is formed by oligodendrocytes*
D. schwann cells form the epineurium around peripheral nerves
E. ion channels are distributed uniformly along myelinated axons

106. Which of the following statements about neurons is TRUE:

A. dendrites are usually unmyelinated*


B. when an axon reaches its target, it generally synapses without branching
C. neuronal cell bodies contain little rough endoplasmic reticulum
D. axonal transport is necessary only for neurons with small diameter axons
E. unmyelinated axons are not ensheathed by glia

107. Which of the following statements about synaptic events is TRUE:

A. presynaptic terminals usually do not contain mitochondria


B. calcium influx is not essential for synaptic transmission
C. transmitters in the synaptic gap can be inactivated by diffusion*
D. sodium influx is essential for transmitter release
E. a major excitatory transmitter is GABA

108. Which of the following statements about graded synaptic potentials is TRUE:

A. they have a threshold


B. they are slow events compared to action potentials*
C. they are only excitatory
D. they result from transmitter binding to metabotropic receptors
E. they do not decay with distance along the dendrite
109. All of the following statements about glutamate receptors are true EXCEPT:

A. calcium ions normally block this channel*


B. they can be ionotropic or metabotropic in type
C. activation of the NMDA receptor alone causes little voltage change
D. current through NMDA receptors is greater when the neuron is already depolarized
E. activation of the NMDA receptor can cause ionic current carried by calcium ions:

110. In contrast to ionotropic receptors, metabotropic receptors:

A. require more transmitter for an effect


B. act more quickly in producing an effect
C. contain a channel that allows ion flow
D. produce a response that greatly outlasts binding to the receptor*
E. activate chemical intermediates with short lifetimes in the neuron

111. The epidural space around the brain:

A. Is a potential space located between the periosteum and the dura mater*
B. Is a potential space that may fill with blood and cause hydrocephalus
C. Contains cerebrospinal fluid
D. Is a potential space located between the dura and arachnoid mater
E. A and C

112. Which of the following is NOT TRUE?

A. There is choroid plexus is each ventricle.


B. CSF is resorbed into the venous sinuses from arachnoid granulations.
C. CSF is continually produced and resorbed; it turns over about 5 times/ day
D. Communicating hydrocephalus results from blockage of the cerebral aqueduct*
E. Increased CSF pressure can cause papilledema

113. The normal distribution, or normal curvet has the following properties EXCEPT:

A. it is bimodal*
B. it is continuous
C. it is symmetrical about its mean
D. the mean, median, and mode are all equal
E. the area under the curve is one

114. Power of study is related to all the following EXCEPT:

A. sample size
B. size of effect
C. reliability of your measures
D. adopted significance level
E. power can be increased by increasing B. and D*.
115. The following are true regarding non-parametric methods EXCEPT:

A. do require precise values*


B. less powerful
C. less dependent on sample size
D. sample size should be less than 50
E. increase the chance of type II error

116. Categorical measures (nonparametric) are best summarized as

A. Median
B. Quartiles
C. Percentiles
D. Proportions*
E. Mean

indicates whether the test is superior to other measurements in


117. Validity that
approaching true validity is known as:

A. convergent validity
B. divergent validity
C. incremental validity*
D. criterion validity
E. concurrent validity

118. An observer error, which arises in data collection when there is carryover
from one judgment to another is called:

A. halo effect*
B. size effect
C. Hawthorne effect
D. Validity effect
E. Statistical effect

119. When every member of the population is chosen. This type of


sampling is called:

A. using random sampling


B. periodic sampling*
C. cluster sampling
D. cluster randomized sampling
E. snow ball sampling

120. Designs in which patients are used as their own controls are called:
A. open label design
B. cross-over design
C. double blind design
D. single blind design
E. all of the above

121. Parametric tests include all the following EXCEPT:

A. t test
B. Chi squared test*
C. ANOVA
D. Pearson Moment Correlation
E. Simple Linear Regression

122. Basal forebrain contains the following nuclei EXCEPT:


A. diagonal band of Broca
B. nucleus accumbens
C. septal nuclei
D. substantia innominata
E. amgdala*

123. Functions of the dorsolateral prefrontal cortex (DLPFC) include all the following
EXCEPT::

A. perceptual judgment
B. memory
C. programming and planning sequences of behavior
D.  writing regulation*
E. Problem solving

124. ∆Frontal lobe syndrome (personality change) is characterized by all the following
EXCEPT:

A. disinhibition
B. increased social and ethical control*
C. sexual indiscretions
D. poor judgment
E. elevated mood (fatuous euphoria)

125. left-sided lesions (affecting the arcuate fasciculus) of the inferior parietal lobule
results in all the following EXCEPT :
A. sensory aphasia * temporal
B. tactile agnosia
C. dysphasia
D. right-left disorientation
E. finger agnosia (can be elicited by the ‘In-Between Test’)

126. The drug implicated in causing growth retardation in


children:
A. amitriptyline
B. clonazepam
C. haloperidol
D. diphenhydramine
E. methylphenidate*

127. Ebstein's anomaly of the heart is most often associated


with prenatal exposure to:
A. diazepam
B. haloperidol
C. desipramine
D. chlorpromazine
E. lithium carbonate*

128. Each of the following drugs is helpful in treating


obsessive-compulsive disorder EXCEPT
A. bupropion*
B. fluvoxamine
C. sertraline
D. fluoxetine
E. clomipramine

129. The generally accepted therapeutic range of blood levels


for lithium carbonate is:
A. 0.8 to 1.0 mEq/liter
B. 0.8 to 1.5 mEq/liter*
C. 8.2 to 2.5 mEq/liter
D. 8 to 15 mEq/liter
E. 10 to 20 mEq/liter

130. It is advisable to measure serum lithium concentrations


how many hours after a dose?
A. 2 hours
B. 4 hours
C. 6 hours
D. 12 hours*
E. 24 hours

131. The drug least likely to cause extra pyramidal symptoms


is:
A. phenelzine*
B. amoxapine
C. perphenazine
D. haloperidol
E. fluphenazine

132. The psychotropic drug with the lowest degree of lipid


solubility is:
A. chlorpromazine
B. haloperidol
C. amitriptyline
D. lithium carbonate*
E. diazepam

133. ∆The drug indicated in ADHD in children is:


A. amitriptyline
B. clonazepam
C. haloperidol
D. diphenhydramine
E.methylphenidate*

134. ∆Regular complete blood counts are most important when


using which of the following drugs:
A. clonazepam
B. carbamazepine*
C. propranolol
D. dantrolene
E. valproate

135. Obsessive-compulsive disorder is associated with


abnormalities of which neurotransmitter system:
A. epinephrine
B. serotonin*
C. melatonin
D. acetylcholine
E. norepinephrine

136. Patients with obsessive-compulsive disorder have been


shown to have abnormalities in which brain region?
A. Cerebellum
B. Caudate*
C. Hippocampus
D. Striatum
E. Corpus callosum

137. The risk of developing schizophrenia in a non-twin sibling


of a schizophrenic patient is:
A. 1%
B. 10%*
C. 25%
D. 50%
E. 75%

138. An intelligence quotient (IQ) of 100 corresponds to an


intellectual ability for the general population in the:
A. 20th percentile
B. 25th percentile
C. 40th percentile
D. 50th percentile*
E. 65th percentile

139. Fatal cardiac arrhythmias have been reported among


children treated with:
A. desipramine*
B. fluoxetine
C. venlafaxine
D. sertraline
E. alprazolam

140. Evan, age 6 years 3 months, was referred for psychiatric


evaluation. Developmental testing revealed that he was
functioning at a mental age level of 4 years. For purposes of
school placement, his IQ score should be reported as:
A. 64*
B. 67
C. 72
D. 80
E. normal range

141. An increase in lithium concentrations is not likely to


result from the use of:
A. indomethacin (Indocin)
B. caffeine*
C. phenylbutazone (Azolid)
D. ibuprofen (Motrin)
E. naproxan (Naprosyn

142. The following statements are true EXCEPT:  tricyclist

A. Drugs must not be ionized to be absorbed by passive diffusion


B. In an acid pH, basic drugs will be poorly absorbed
C. Gastric emptying is delayed by MAOIs
D. Food increases the absorption of diazepam
E. Rectal administration results in extensive 1st pass metabolism*

143. Regarding Drug interactions, one is true :

A. Carbamazepine can inhibit the metabolism of TCAs


B. Phenothiazines can induce their own metabolism*
C. Haloperidol potentiates the metabolism of TCAs
D. Cytochrome P450 inhibition by cimetidine is an important factor in healthy subjects
E. Slow acetylators predominate in Europe and Japan

144. The following are true EXCEPT one:

A. In the elderly, there is a reduction in plasma albumin


B. There is a loss of body weight in the elderly*
C. In pregnancy, the increase in plasma volume results in a decrease in the free fraction of a drug
D. First-order kinetics are exponential
E. Alcohol and aspirin undergo zero-order kinetics

145. The following statements about the distribution of neurotransmitters are correct
EXCEPT one:

A. Acetylcholine is found in the basal ganglia


B. Dopamine cell bodies are found in substantia nigra
C. 5-HT predominates in the raphe nuclei in the brainstem
D. Noradrenaline predominates in the locus coeruleus
E. GABA is found in the peri-aqueductal grey matter*

146. The following are false statements about receptors EXCEPT:


A. 5-HT2A agonists improve slow-wave sleep
B. 5-HT1A antagonists are anxiolytic
C. D2 receptors are found in the limbic system*
D. Antagonism of alpha-2 adrenoceptors leads to reduced NA release
E. Most antipsychotics are D2 agonists

147. Regarding benzodiazepines (BZDs) all the following are true EXCEPT:  trickcyclist

A. BZDs bind to the gamma-2 subunit of the GABAB supramolecular complex*


B. BZDs can inhibit the effects of other neurotransmitters such as 5-HT
C. Oxazepam and Lorazepam are short-acting BZDs
D. The BZ1 receptor mediates the anti-anxiety effect of BZDs
E. The BZ2 receptor is concentrated in the amygdala and septo-hippocampal pathways

148. Regarding Benzodiazepines, one is true:

A. Tolerance develops in 4-6 weeks of therapy


B. Withdrawal can cause rebound hypersomnia
C. BZDs are effective in phobic states
D. Zopiclone and Zolpidem act upon the BZ2 receptor in a similar way to BZDs
E. Withdrawal symptoms peak at 7-8 days*

149. Factors associated with dependence and withdrawal problems include all the following
EXCEPT:

A. Long duration of treatment


B. Passive-dependent personality traits
C. Age < 40 years*
D. Concurrent use of Buproprion
E. High dose and rapid withdrawal

150. Regarding Buspirone, all the following are true EXCEPT:

A. Buspirone is a 5-HT1A agonist


B. Buspirone has little effect on DA systems*
C. Effects are usually evident after 4-6 weeks
D. It can cause galactorrhoea
E. Dysphoria has been reported

151. One of the following statements about antidepressants is true:


A. Mianserin is an SSRI
B. Reboxetine is a NaSSA
C. Mirtazapine is a NARI
D. Trazodone is an SNRI
E. Sertraline is an SSRI*

152. Regarding Tricyclic Antidepressants (TCAs), all the following are true EXCEPT: 
TCA
A. Treatment results in subsensitivity of NA and 5-HT receptors on cell bodies
B. Tertiary amines have a higher affinity for the 5-HT uptake site
C. Tertiary amines are more sedating
D. Secondary amines have more anticholinergic side effects*
E. Tertiary amines have a quicker peak plasma level

153. Side effects of TCAs include all the following EXCEPT:

A. Sinus bradycardia*
B. Impairment of memory
C. Postural hypotension due to alpha-1 adrenoceptor antagonism
D. Weight gain due to histamine H1 antagonism
E. Negative inotropism

154. Regarding SSRIs, one is true:

A. They reach peak levels within 1 hour of ingestion


B. Long-term use results in increased 5-HT2 function
C. Fluoxetine has a half-life of around 7 days
D. OCD is an indication for their use*
E. They should be avoided in people with cardiac disease

155. Common side effects of SSRIs include all the following EXCEPT:

A. Diarrhoea
B. Constipation
C. Loss of appetite
D. Weight gain*
E. Tremor

156. Interactions of SSRIs include:

A. Use with MAOIs can result in the 5-HT toxicity syndrome


B. Lithium
C. Diazepam and fluoxetine
D. Warfarin
E. All of the above*
157. Regarding MAOIs, one is true:  TCA

A. Most are selective for MAOI-A


B. Phenelzine has fewer side effects than isocarboxacid
C. Hypotension is a major problem with tranlycypramine
D. Fast acetylators metabolize MAOIs at the same rate as slow acetylators
E. It is said that atypical depression responds best to MAOIs*

158. Recognized side effects of MAOIs include all the following EXCEPT:

A. Mania
B. Seizures
C. Blurred vision
D. Peripheral neuropathy
E. Alopecia*

159. One is true regarding Mirtazapine:  TCA

A. Has little effect on histamine receptors


B. Blocks alpha-2 autoreceptors*
C. Is alerting in its profile
D. Can reduce appetite
E. Can cause leucocytosis

160. One is False regarding Trazodone:

A. Is sedating
B. Has 5-HT agonist properties
C. Should be taken with food
D. Can cause priapism
E. Has no effect on cognitive function*

161. One is false regarding Venlafaxine:

A. Is an SNRI
B. Lacks anticholinergic effects
C. Has an effect on the seizure threshold
D. Can be given with MAOIs*
E. Can cause hypotension

162. One of the following is true of movement disorders:  TCA

A. Young females are at highest risk of acute dystonic reactions


B. Extrapyramidal symptoms tend to occur within a few hours of drug administration
C. Extrapyramidal symptoms are due to blockage of D1 receptors in the basal ganglia
D. Increasing the dose can sometimes help extrapyramidal side effects
E. Tardive dyskinesia is due to D2 receptor hypersensitivity*

163. The following are at increased risk of tardive dyskinesia EXCEPT:


A. Female
B. Affective disorders
C. Continuous treatment*
D. Organic brain disease
E. Increasing age
   
164. One of the following is a recognized side effects of antipsychotics :
 
A. Sedation due to muscarinic blockade
B. Nasal congestion*
C. Increased sex drive
D. Psoriasis
E. Hypertension

165. Recognized side effects of antipsychotics include all the following EXCEPT:  
A. Retinal pigmentation
B. Leucocytosis*
C. Prolonged QT interval
D. Weight gain
E. Torsade de pointes
   
166. Regarding neuroleptic malignant syndrome, one is true:
 
A. Onset occurs after 1-2 months of treatment
B. Onset is slow and insidious
C. Symptoms include hypotonicity,
D. Mortality is 50 %
E. Secondary conditions include thromboembolism, renal failure, and cardiovascular collapse*
   
167. One of the following is false regarding Risperidone:
 
A. Has high affinity for the 5-HT2A receptor
B. Can cause hyperprolacinaemia
C. Weight loss is frequent*
D. Is as effective as conventional antipsychotics (at least)
E. Headache and anxiety may occur
   
168. All the following are true regarding Clozapine EXCEPT:
 
A. Has low affinity for D2 receptors
B. Has high affinity for D1 and D4 receptors
C. Has few effects on adrenoceptors*
D. Can increase blood levels of warfarin and digoxin
E. Metabolism by the cytochrome P450 system is not significant
   
169. Side effects of Clozapine include all the following EXCEPT:
 
A. Bradycardia*
B. Weight gain
C. Hypersalivation
D. Decrease in seizure threshold
E. Neutropenia and agranulocytosis
   
170. One is true regarding Olanzapine:

A. Has lower affinity for the D2 and 5-HT2A receptors than Clozapine
B. Has higher affinity for the D1 receptor than Clozapine
C. Causes negligible weight gain
D. Has high levels of Extra-pyramidal side effects
E. Can cause marked sedation*

171. All the following are true regarding Quetiapine EXCEPT:


 
A. Has a similar binding profile to clozapine
B. Has a high affinity for muscarinic receptors*
C. Has lower affinity for all receptors than clozapine
D. Causes less weight gain than olanzapine and clozapine
E. Can cause constipation
   
172. One is true regarding Amisulpride:
 
A. Is a D2/ D3 agonist
B. Enhances autoreceptors at low doses and can decreasesynaptic dopamine levels
C. Does not increase prolactin levels
D. Has a similar level of EPS across the whole dose range
E. Is clinically effective for negative symptoms of schizophrenia at low doses (<300mg/ day)*

173. ∆borderline syndrome is characterized by all the following EXCEPT: 

A. Splitting
B. Poor impulse control
C. Euphoric affect*
D. Primitive idealization
E. Feelings of emptiness
   
174. Somatic symptoms of severe anxiety include all the following EXCEPT: 

A. Impotence
B. Diarrhoea
C. Constipation
D. Hypoventilation*
E. Sighing
   
   
175. All the following are true regarding Autochthonous delusions EXCEPT:  
A. Synonymous with primary delusions
B. Rarely preceded by a delusional atmosphere*
C. A source of secondary delusions
D. Can occur in schizophrenia
E. 'Brain waves'
   
176. ∆experiences include all the following EXCEPT:

A. Jamais vu
B. Delusional perception*
C. Derealization
D. Visual illusions
E. Deja-vecu

177. All the following are true regarding Dissociative states EXCEPT:  TCA  
A. Occur in hysteria
B. May be seen under hypnosis
C. Include fugue states
D. Exclude multiple personalities*
E. Are seen in conversion disorder
   
178. Formal thought disorder includes all the following EXCEPT:
 
A. Drivelling
B. Conversion*
C. Flight of ideas
D. Perseveration
E. Transitory thinking
   
179. ∆One is true regarding Concrete thinking:  

A. Is diagnostic of schizophrenia
B. Is diagnostic of organic brain disease
C. May occur in manic-depressive psychosis
D. Is a defect of conceptual abstract thought*
E. Is tested by 7,s serial test
   
180. ∆ One is False regarding Formication:  TCA  

A. Is not the medical term for fornication


B. May be seen in delirium
C. Is a passivity phenomenon*
D. Is a tactile hallucination
E. May be called the 'cocaine bug'
   
181. Normal experiences include all the following EXCEPT:  TCA
 
A. Hypnagogic hallucinations
B. Hypnopompic hallucinations
C. Depersonalization
D. Iillusions
E. Over-inclusive thinking*
   
182. One is false regarding delusional perception:
 
A. Has two stages
B. Is an autochthonous delusion
C. Is often preceded by 'delusional mood'
D. Occurs secondary to a hallucination*
E. Is a primary delusion
 
183. Pseudodementia may be characterized by:
 
A. Onset with depressive features*
B. Abnormal EEG
C. Presence of localizing neurological signs
D. Negative Past or family history of manic-depressive psychosis
E. Chronic course
 
184. Echolalia occurs in all the following EXCEPT:  TCA
 
A. Catatonic schizophrenia
B. Autistic disorder
C. Mental handicap
D. Manic-depressive psychosis*
E. Senile dementia

185. all the following are true regarding Autosomal Dominant EXCEPT:

A. the phenotypic trait is present in all individuals carrying the dominant allele
B. the phenotypic trait does not skip generations – vertical transmission takes place
C. males and females are affected
D. male to male transmission can take place
E. transmission is solely dependent on parental consanguineous matings*

186. Recombination fraction varies between

A. zero and 0.5*


B. 1 and 2
C.0.5 and 1
D. 1.5 and 2
E. none of the above

187. The last phase of Mitosis is:

A. interphase
B. prophase
C. metaphase
D. anaphase
E. telophase*

188. All the following are true regarding X-linked recessive EXCEPT:

A. all male offspring manifest the abnormal phenotypic trait


B. male-to-male transmission does not take place
C. all of the daughters of an affected male will be carriers
D. female heterozygotes are affected*
E. affected females are rare

189. All the following are true regarding Prevalence EXCEPT:

A. it is a measure of disease frequency,


B. it can be expressedas a percentage
C. it has units*
D. itis determined by a cross-sectional study
E. it has to be related to a time period, for example weekly, monthly, or
Lifetime

190. The following structures are in the midbrain EXCEPT:  


A. Superior colliculus T
B. Inferior colliculus T
C. Substantia nigra T
D. Ventral tegmental area T
E. Medical longitudinal fasciculus*

191. Features of cerebellar disease include all the following EXCEPT:   TCA
A. Pendular nystagmu *
B. Dysdiadochokinesis T
C. Intentional tremor F
D. Past pointing T
E. Scanning dysarthria

192. Causes of pre-senile dementia include all the following EXCEPT :  TCA 
A. Jakob-Creutzfeldt disease T
B. Simple schizophrenia*
C. Subacute spongiform encephalopathy T
D. Alcoholism F
E. Punch-drunk syndrome

193. Components of the Papez circuit include all the following EXCEPT:  TCA
A. Fornix T
B. Mamillary body T
C. Hippocampus T
D. Locus cereleous*
E. Cingulate gyrus

194. Functions of the limbic system may include all the following EXCEPT:  
A. Emotional behaviour T
B. Motivation T
C. Sexual activity T
D. Logical reasoning* T
E. Memory

195. Principal outputs of the basal ganglia go to all the following EXCEPT:  
A. Cerebral cortex* F
B. Red nucleus T
C. Tectum T
D. Subthalamic nucleus T
E. Substantia nigra T
 
196. Diplopia occurs in:  
A. Neuropathy of oculomotor nerve* T
B. Parkinson’s disease F
C. Neuropathy of facial nerve F
D. Huntington’s disease F
E. Diabetes insipidus

197. Memory defect occurs with lesions in:  


A. Medial-dorsal thalamic nucleus* T
B. Wernicke’s area F
C. Walls of fourth ventricle T
D. Broca’s area F
E. Parietal cortex

198. Functions of the non-dominant cerebral hemisphere may include all the following
EXCEPT:  
A. Holistic T
B. Ideational* F
C. Pictorial T
D. Geometric T
E. Non-linear

199. Possible feature of lower motor neuron disease includes:  


A. Atonic muscles* T
B. Hyperreflexia T
C. Loss of 100 % of muscle bulk F
D. Clasp-knife rigidity F
E. Tardive dyskinesia

200. Features of Creutzfeldt-Jacob disease include all the following EXCEPT:  TCA
 
A. Neuronal degeneration T
B. Atrophy of caudate and putamen especially* F
C. Spongeiform changes T
D. Does affect the whole CNS F
E. Glial proliferation T
F
1. All the following are true regarding the Preoperational stage EXCEPT:

A. children can think in images and symbols


B.  childrenacquire language
C. childrenplay games of pretend
D. children are able to make general, logical statements*
E. rules are inviolate

2. All the following are features of Stage of Concrete Operations EXCEPT:

A. understanding of logical principles that apply to external objects


B. conservation of length & volume
C. reversibility
D. sort objects into categories - classification
E. hypothetico - deductive thinking*

3. According to Erikson’s psychosocial development, intimacy vs. stagnation occurs in:

A. early adulthood*
B. middle adulthood
C. old age
D. adolescence
E. none of the above

4. Easy children are characterized by all the following EXCEPT:

A. regular rhythmicity
B. positive approaches to new situations
C. rapid, positive adaptability to change or novelty
D. mild intensity of emotional reaction
E. adapt slowly and do not tolerate disappointment*

5. Authoritarian parents

A. tend to be strict, punitive, and unsympathetic*


B.  encourage independence
C. often praise their children
D. tend to have friendly children
E. tend to have trustful children

6. According to Festinger's cognitive dissonance theory, people seek to avoid...


A. excessive stimulation
B. contradictions between behaviors and beliefs*
C. rebellious or negative reactions
D. music which is not harmonious
E. people who are argumentative

7. What is the idea behind Maslow's famous pyramid?


A. to preserve Maslow's theory forever
B. as one takes care of basic needs, the "higher needs" become more relevant*
C. the road to enlightenment consists of many small steps
D. it was a sort of temple or meeting place for his students
E. it showed the three factors underlying self-actualization, one on each corner
F.
8. The James-Lange or "body reaction" theory of emotion says...
A. you feel emotion, then a bodily reaction
B. you react with your body first, then you feel emotion*
C. the somatic nervous system is the seat of emotion
D. emotion is adrenalin plus context
E. emotions and visceral reactions are simultaneous

9. Ainsworth described the "anxiously attached, ambivalent" infant as one who...


A. doesn't mind being put down but acts worried as soon as the mother leaves
B. is happy when around the mother but refuses to leave the mother's side
C. clings or cries as the mother puts her down, then later pouts or fights with her *
D. does not cry when the mother leaves and may avoid her when she returns E.
laughs one minute and cries the next

10. Longitudinal research on adult personality development shows...


A. "change is the only constant"
B. slow but steady improvement in intellect
C. remarkable stability on almost every personality rating scale *
D. dramatic change in 30-40% of adults
E. seven stages of adult personality development

11. When an organism has been accidentally reinforced after some behavior so the behavior
increases, but the reinforcement does not actually depend on the behavior at all, the result is
     A. conditional neurosis.  
     B. spontaneous recovery.
    C. superstitious behavior.*   
    D. unconditional reinforcement
E. discriminant response

12. In general, reinforcers and punishers are more effective if they are delivered
     A. after some delay.  
     B. immediately after the behavior*.
     C. before the desired behavior.   
     D. no matter what the organism does.
E. after long delay

13. The developing brain of a baby is especially sensitive to damage from outside influences
during what period of time?
A. The first three months
B. The fourth and fifth months
C. Right before birth
D. Up until age 2*
E. Right after birth
14. In Schacter and Singer's study on emotions, in which they injected volunteers with adrenaline
to trigger physiological arousal, what determined WHICH emotion each person experienced?
A. How much adrenaline was injected
B. Whether the volunteer was male or female
C. The type of physiological arousal that was triggered
D. The situation in which the volunteer was placed*
E. The temperament of the volunteer

15. According to the humanistic motivation theory of Abraham Maslow, self-actualized people
are
A. realistic, spiritual, and unconventional.*
B. self-centered, selfish, and egocentric.
C. introverted, withdrawn, and isolated.
D. hard-working, tense, and achievement-oriented
E. extroverted, unrealistic, and extravagant

16. Research on the effects of practice on memory shows that


    A. too much practice causes interference and reduces memory.
    B. practice is more effective in a few long sessions.
    C. practice is more effective in many short sessions*.
    D. practice is more effective in a few short sessions.
E. practice is as effective in many or few short sessions

17. According to the text, what is the relationship between creativity and intelligence?
     A. This question has never really been studied.
     B. There is no firm relationship between creativity and IQ.*
     C. Creative people are more intelligent than others.
     D. Creative people are less intelligent than others.
E. Creative people are always genius

18. According to Jung, our myths and dreams are full of archetypes, which are
A. the unique, individual symbols of our unconscious minds.
B. the three aspects of the personality: id, ego, and superego.
C. inherited ideas and symbols that are common to all people*.
D. the lower level needs in the hierarchy of motivation
F. the higher level needs in the hierarchy of motivation

19.What is the best approach that a physician can take with a hostile
patient who is hospitalized?
a. offer straightforward explanations of procedures*
b. be sympathetic about the patient's fears
c. set firm limits on the patient's behavior
d. avoid isolating the patient
e. let the patient share in the treatment decisions

22. The side effect that is least likely to be caused by fluoxetine (Prozac)
is:
a. orgasmic dysfunction
b. insomnia
c. nausea
d. fatigue
e. constipation*

24. A 40-year old physician was hospitalized following a Femoral


fracture in an automobile accident. Orthopedic surgery was uneventful and
for the first 2 recovery days the patient was restless but well oriented. On
the third day after surgery the patient rapidly became confused and fearful,
and reported visual and tactile hallucinations of snakes and scorpions
crawling over the bed clothes. Physical examination revealed dilated
pupils, coarse tremor of the hands and eyelids, profuse diaphoresis, and a
rapid, pounding heartbeat. His oral temperature was 100 degree F. His
previous medical history was unremarkable. He is described as a moderate
social drinker and denied use of other drugs. The diagnosis most strongly
suggested is:
a. alcohol withdrawal*
b. postanesthesia delirium
c. septicemia
d. subdural hematoma
e. uremia

25. Adverse reactions of tricyclic antidepressants are most likely to be


clinically significant in patients with:
a. migraine
b. Parkinsonism
c. insomnia
d. pseudotumor cerebri
e. benign prostatic hypertrophy*

28.Possible signs of posterior column damage include all the following EXCEPT:  
A. Positive rombergism F
B. Diminished tendon reflexes T
C. Hypertonicity* F
D. Loss of vibration sense T
E. Loss of proprioception T
29.Features of upper motor neuron lesions could include all the following EXCEPT:  
TCA
A. Clonus T
B. Cogwheel rigidity* F
C. Extensor plantar response F
D. Preservation of muscle bulk T
E. Increased tendon reflexes
T
30.Features of multi-infarct dementia include all the following EXCEPT:  

A. A. Neurofibrillary tangles* F
B. B Hypertension T
C. C. Cerebral ischaemia T
D. D Cerebral infarction
E. T
31.The limbic system includes all the following EXCEPT
A. Parahippocampal gyrus T
B. Hypothalamus T
C. Corpus callosum* F
D. Anterior nucleus of thalamus T
E. Subcallosal gyrus

33. circle of Willis is formed by:  


A. Superior cerebellar artery F
B. Posterior spinal artery F
C. Posterior communicating artery* T
D. Middle cerebral artery F
E. Anterior inferior cerebellar artery
F
34.After listening to several examples of similar behavior in the patient's life story, the
psychiatrist offers the observation "When you can't perform up to your own high standards, you
appear to act to try to please someone else." This response by the psychiatrist is an example of
A. Reflection.
B. Interpretation.
C. Transition.
D. Self-disclosure.
E. Facilitation.

37.Parietal lobe symptoms include the following EXCEPT:

A. Astereognosis.
B. Dressing apraxia.
C. Acalculia.
D. Ideomotor apraxia.
E. Aphasia*.

 
38. Parasympathetic stimulation causes all the following EXCEPT:

A. Slow heart rate.


B. Dilated pupils*.
C. Increased acid secretion of stomach.
D. Salivary secretion.
E. Dencreased diameter of the bronchioles.

39.Confusional state may be caused by all the following EXCEP:

A. Paracetamol.
B. Cimetidine.
C. Procyclidine.
D. Dothiepin.
E. Methylcellulose*.

42.Woman with agoraphobia improves with repeated exposure to crowds. Which one of the
following is the essential psychological process involved?
 
a)                distraction
b)                extinction  *
c)                 instrumental conditioning
d)                massed practice
e)                selective abstraction
 
43. A patient asks you about a recent newspaper report on the neuregulin-1 gene and psychiatric
disorder.  You explain to her that the most likely psychiatric link is with which one of the
following disorders?
 
a)     Anorexia Nervosa 
b)     Attention Deficit Hyperactivity Disorder (ADHD)
c)      Bulimia Nervosa  
d)     Major Depressive Disorder
e)     Schizophrenia   *

44. You are teaching medical students about the differences between physiological Prion protein
and pathological Prions which lead to the development of variant Creutzeld-Jakob disease.
Which one of the following would you tell them about PrPsc (Prion Protein Scrapie)? It has a:
 
a)     amino acid sequencing difference
b)     bovine origin
c)      covalent bond difference from PrPc (normal Prion protein)
d)     higher beta sheet content   *
e)     origin from infected blood

45 . Echolalia is seen in:

A. learning disability
B. dementia
C. head injury
D. Tourette’s syndrome
E. all of the above*
46 . Definition of a delusion includes all the following EXCEPT:

A. it is always self-involved/ self-referential


B. held unshakably
C. modified by experience or reason*
D. content often bizarre
E. not dependent on disintegration of general intellectual functioning

47 . All the following are true regarding Delusional intuition (autochthonous) EXCEPT:

A. ‘out of the blue’, ‘brain wave’


B. arise fully formed as sudden intuitions
C. occur in a two stage*
D. a.k.a. Wahneinfall (Ger.)
E. consistent with ‘double orientation’

51. Criterion validity refers to:

(A) judgment by “experts” that the items that make up a test “make sense”
(B) judgment that the items that make up a test cover the domain of knowledge relevant for the test’s
purposes
(C) comparison of a test to another established test*
(D) the number of true cases who are detected by a test instrument
(E) the ability of a test to predict who will develop the disorder at some point in time in the future

52. Which of the following epidemiological methods is primarily concerned with assessing the
effect of preventive or therapeutic interventions designed to alter the development or outcome
of illness?

(A) descriptive studies


(B) analytic studies
(C) historical studies
(D) experimental studies*
(E) community diagnosis studies

53. Which of the following statistical indices provides a measure of agreement between clinicians,
correcting for chance variance?

(A) Spearman’s r
(B) the kappa statistic*
(C) Cohen’s beta statistic
(D) the mu statistic
(E) the F statistic

54. Which of the following statements is TRUE?


a. Peripheral nerve is similar to smooth muscle in terms of the connective tissue
investment
b. Most nerves contain afferent and efferent fibers and thus carry both motor
and sensory signals*
c. Nodes of Ranvier are most easily seen in cross-section of peripheral nerve
d. None of the above

55. Which of the following cells is reponsible for myelin formation in the peripheral
nervous system?

a. Astrocyte
b. Oligodendrocyte
c. Schwann cell *
d. Microglial cell
e. Satellite cell

56. A neuron with many short dendrites and a single long axon is a:

A. Multipolar neuron *
B. Bipolar neuron
C. Unipolar neuron
D. None of the above

57. An inhibitory neuron could affect the neuron with which it synapses by:

a. Producing an IPSP within the neuron


b. Hyperpolarizing the neuron
c. Increasing K+ efflux from the neuron
d. Increasing Cl- influx into the neuron
e. All of the above *

58. Which of the following organelles is responsible for the appearance of Nissl bodies
in the cell bodies of motor neurons?
a. Smooth endoplasmic reticulum
b. Rough endoplasmic reticulum *
c. Golgi apparatus
d. Mitochondria
e. Basal bodies  

1. All of the following statements are true EXCEPT:


a. Occipital lobe is associated with memory and speech*
b. Frontal lobe is associated with problem solving
c. Parietal lobe is associated with orientation and perception of stimuli
d.
f. Occipital lobe is associated with visual processing
g. Temporal lobe is associated with perception and recognition of auditory stimuli

2. All of the following statements are correct EXCEPT:


a. Emotional experience and expression involve all parts of nervous system
b. Cannon-Bard theory is an early theory of emotion
c. Papez circuit is responsible for the central functions of emotion
e. Left cerebral hemisphere seems to be dominant for emotion*
f. Relationships among somatic sensations and subjective emotions are not clear

3. All of the following are Maslow’s hierarchy of needs EXCEPT:


a. Needs to satisfy hunger and thirst
b. Belongingness and love needs
c. Safety needs
d. Esteem needs
e. Ecstasy needs*
4. The following are basic structure of Papez Circuit EXCEPT:
a. Hypothalamus
b. Hippocampus
c. Angular gyrus*
h.
w. A
x. mygdala
z. Cingulate gyrus
5.

6. All the following are true about gender and IQ EXCEPT:


o The brain areas correlated with IQ differ between the sexes
o Men and women apparently achieve similar IQ results
o Men generally score higher in the verbal sections of IQ test
o Compared to women, men show more gray matter areas related to intelligence*
o Men tend to score higher in the parts of the test that cover spatial and quantitative
abilities

6. All the following are true of Wechsler intelligence test EXCEPT:


a. Tests comprehension
b. Tests vocabulary
c. Uses block design tests
d. Tests arithmetical ability
e. Used only for children*
7. Which ONE of the following is a projective personality
test?
a. Rorschach Inkblot test*
b. Halstead-Reitan battery
c. Wechsler Intelligence tests
d. Eysenck Personality Questionnaire (EPQ)
e. Minnesota Multiphasic Personality Inventory (MMPI)

8. All the following are personality tests EXCEPT:


a. Luria-Nebraska battery tests *
b. Rorschach Inkblot test
c. Thematic Apperception Test
d. Eysenck Personality Questionnaire (EPQ)
e.innesota Multiphasic Personality Inventory (MMPI)

9. ∆All the following are clinical features of depression EXCEPT:


a. Flight of ideas*
b. Depressed mood
c. Poverty of speech
d. Reduced rate of blinking
e. Impaired attention and concentration

10. All the following are types of cognitive distortion in depression EXCEPT:
a. Arbitrary inference
b. Over-generalization
c. Selective abstraction
d. Body image distortion*
e. Minimization and magnification

11. All the following statements about suicide are correct EXCEPT:
a. Most people who commit suicide are depressed
b. Suicide is the first leading cause of death in adolescents*
c. 75% of people with depression can be successfully treated
d. Completed suicide is more common in men than in women
e. Suicide among those with depressive disorders can exceed 15%
12. ∆Which of the following is NOT a feature of mania?
a. Over-activity
b. Hyposexuality*
c. Elation of mood
d. Over-talkativeness
e. Grandiose delusions

14. Which one of the following is not a measurement scale of attitude?


a. Likert scale
b. Thurstone scale
c. Cognitive dissonance*
d. Visual analogue
e. Semantic Differential

15.The following are characteristics of doctors that enhance persuasive communication


except:
a. Expertise
b. Motivation
c. Proximity
d. Attractiveness
e. Social background*

16.Which one of the following has no social effects on measurement of behavior?


a. Diffusion of responsibilities*
b. Response set
c. Bias to the muddle
d. Hello effect
e. Hawthorne effect

17. According to cognitive dissonance theory the following factors increase dissonance
except:
a. Low pressure to comply
b. Increased perceived choice of options
c. Awareness of responsibility for consequences
d. Change behavior*
e. Expected unpleasant consequence of behavior to others

18.Regarding Classical conditioning the following are true except:

a. Subject is active, likely to be using consciously controlled behaviors. *


b. CS should be simultaneous with or just anticipate UCS
c. UCR and CR are essentially the same
d. CS can only introduced if S-R link is already exists
e. Subjects are more likely to acquire CS than others
19.The following are Models of observational learning except:

a.high status
b.High competence
c.High social power
d.Some characteristics in common with the observer
e.High self esteem and high confidence of the observer enhances perception of these
characters*

20.Which one of the following factors has no effect on the social learning theory?
a. Cognitive appraisal
b. Cognitive dissonance*
c. Inference
d. Goal seeking
e. . Affiliation

21.Regarding memory, which one of the following is false?

a. Episodic memory is personally experienced episodes and events


b. Semantic memory is time and context-specific*
c. Procedural memory includes motor skills and priming
d. Recall in 3 minutes is retrieving already registered information
e. The Neuronal Substrate of the semantic memory is Temporal Neocortex

22. ∆The following are motor disorders except:


a. Perseveration
b. Echopraxia
c. Mannerism
d. Stereotypy
e. Knight's move (derailment) *

23.The following are disorders of speech except:


a. Poverty of speech
b. Pressure of speech
c. Stupor*
d. Neologism
e. Mutism

24. ∆ the following are characteristic features of obsessional thoughts except:

a. They are recurrent, persistent thoughts, impulses, or images.


b. The patient regards as absurd and alien
c. The attempts are made (at least early on) to resist or ignore them.
d. Frequently the obsessions are of an aggressive or sexual nature.
e. The patient believes them as thoughts inserted into his mind*

25.the following are characteristic features of delusions except:


a. A false Firmly held belief
b. Changing with evidence to the contrary*
c. Out of keeping with the person’s educational and cultural background
d. Content often bizarre
e. Often infused with a sense of great personal meaning.
26. Which one of the following is a psychological theory of behavioural change?

A. Gene effect
B. Limbic function
C. Biochemical factors
D. Social learning factors *
E. Chemical neurotransmission

27. Which one of the following is a component in stage theory of behavioural change?

A. Consciousness raising*
B. Self de-evaluation
C. Stimulus-response
D. Conditioning
E. Reversibility

28.The following statements concerning theories of behavior change are correct Except:

A. Social psychological theories are concerned with how events and experiences external to
a person influence his behavior
B. Social cognitive approaches give a role for social learning on behavior change
C. Self-efficacy is derived from social learning theory
D. Cognitive behavior theories stress the role of social learning techniques *
E. Learning theories form a basis for behavioral change

29. Which one of the following is considered in personality theory of behavior change?

A. Physical environment
B. Individual traits *
C. Social environment
D. Economic environment
E. Previous experiences

30. ∆Which one of the following is a prominent psychotic feature?

A. Anhedonia
B. Overvalued idea
C. Impaired reality testing *
D. Objective ability to evaluate and judge the world outside the self
E. Social withdrawal

31. ∆Which one of the following statements is false?

A. Hallucination is a sensory perception in the absence of a real stimulus


B. Delusion means false belief that is subject for change by persuasion*
C. Schizophrenia is a chronic psychotic disorder
D. Psychosis can result from general medical conditions
E. Brief psychotic disorder lasts less than 30 days
32. ∆Schizophrenia is characterized by all the following features Except:

A. Intact socio occupational functioning *


B. Formal thought disorder
C. Disturbed speech
D. Disturbed behavior
E. Incongruity of affect

33. ∆ Prodromal features of schizophrenia include which one of the following :

A. Hallucinations
B. Delusions
C. Abnormal indulgence in philosophical matters *
D. Formal thought disorder
E. Over indulgence in social activities

34.Which one of the following is a biological determinant of Behavior is determined?

A. Learning factor
B. Social influences
C. Psychological influences
D. Growth and developmental factors *
E. Sociocultural factors

35. All of the following are involved in neurotransmission Except:

A. Acetyl choline
B. Valproate *
C. Dopamine
D. Norepinephrine
E. Serotonin

36. Which one of the following is not a component of the neuron?

A. Perikaryon
B. Presynaptic membrane
C. Postsynaptic membrane
D. Amygdaloid *
E. Synaptic cleft

37. Which one of the following statements is true?

A. The reticular Activating System is involved in emotions


B. Neurotransmitters are intraneuronal structures
C. Neurotransmitters are chemical messengers *
D. Chromosomal abnormalities are unrelated to behavior abnormalities
E. The limbic system is a biological determinant of arousal

38. Which one of the following is an amino acid neurotransmitter?


A. Glutamate
B. Noradrenaline
C. Serotonin
D. Dopamine
E. Substance P

39. Adjustment to social regulations without the loss of too much independence occurs at
what stage of erikson’s eight stage of man:

A. intiative vs guilt
B. autonomy vs shame*
C. trust vs mistrust
D. industry vs inferiority
E. identity vs confusion

40. Intimacy versus isolation occurs in:

A. adolescence
B. middle adulthood
C. early adulthood*
D. old age
E. both A and B

41. According to Kohlberg, rules are obeyed to avoid punishment is a feature of:

A. stage 1 of moral development*


B. stage 2 of moral development
C. stage 3 of moral development
D. stage 4 of moral development
E. stage 5 of moral development

42. Normally developed children usually babble by:

A. 2 months
B. 10 months
C. 6 months*
D. 12 months
E. 14 – 18 months

43. One dimensional classification of objects is a feature of:

A. Sensorimotor stage
B. preoperational stage
C. Concrete stage*
D. Formal operational stage
E. Both A and B

44. Autonomous adult attachment is best predicted by:

A. Secure childhood attachment*


B. Insecure-avoidant childhood attachment
C. Insecure-ambivalent childhood attachment
D. Disorganized childhood attachment
E. Both B and C
45. The following are features of authoritative parents EXCEPT:

A. responsive
B. warm
C. child-centered
D. children of such parents tend to be independent
E. children from this category may be more aggressive*

1. What is Pavlovian conditioning, in a nutshell?


o modification of behavior by reinforcement
o an anticipatory biological response *
o any influence upon learned behavior by reflexes
o learning from the effects of behavior
o establishing a connection between behavior and its consequences
o
2. What is a formal definition of "operant"?
o a contingent or noncontingent stimulus-induced response
o a behavior
o a discriminative stimulus
o a behavior under the control of a stimulus
o a class of behaviors with an equivalent effect on the environment *
o
3. To what does the term "positive" refer, in the term "positive reinforcement"?
o pleasant emotions
o logical positivism
o the scientist's assumption that the procedure will increase frequency of behavior
o the application or addition of a stimulus to a situation*
o an excitatory influence on neurons
4. Rehearsal resembles...
o echoic memory
o acoustic interference
o autobiographical stories
o encoding
o "re-hearing" something*
o
5. A person with a low set-point eats a very rich meal. What normally happens next?
o nausea
o body temperature rises*
o the set point is raised for a day or two
o body temperature falls
o extreme sleepiness
o
6. Which behavior is common in situations of motivational conflict?
o "voodoo death"
o depression
o lethargy or sleepiness
o vacillation
o getting "stuck" on the first decision *
o
7. How does self-efficacy influence our response to life's everyday problems?
o it leads to a "can-do" attitude *
o it contradicts the idea of optimum complexity
o it traps people in self-defeating behavior
o it sometimes makes simple problems seem complex
o it makes a person feel cut off and lonely
8. Hebb, attempting to make a motivational law out of the Yerkes-Dodson research,
proposed that motivation is greatest when animals act...
o without knowing the outcome of their behavior
o in response to "optimal stimuli"
o while at an optimal level of arousal *
o to reduce stimulus inputs
o to maintain complexity
o
9. The "need for cognitive consistency" can also be described as a need for...
o "optimal complexity"
o "stimulus-seeking"
o social support
o "congruity"*
o "ludic behavior"
o
10. What is the relationship of genes to chromosomes?
o they are the same thing
o there are many chromosomes on each gene
o there are about 46 genes per chromosome
o there are about 20,000 genes per chromosome *
o here are billions of chromosomes per gene
o
11. The "heritability index" tells us the extent to which...
o a behavior is learned or genetic
o test scores of monozygotic twins correlate more than those of dyzygotic twins *
o a child "takes after" a mother or father
o a genetic trait is typical of a family
o there is variability in the genes
o
12. Piaget's studies of "conservation" trace the development of...
o perception
o internal representation or "knowledge" *
o object constancy
o sensory-motor activity
o reflexes
o
13. Longitudinal research on adult personality development shows...
o "change is the only constant"
o slow but steady improvement in intellect
o remarkable stability on almost every personality rating scale *
o dramatic change in 30-40% of adults
o seven stages of adult personality development
o
14. Which of the following is one of the five stages Kubler-Ross described as typical of
terminally-ill patients?
o complacency
o bargaining*
o irritation
o dependence
o desperation
o
15. In what way was Freud influential, in the field of personality theories?
o Freud was the first person to talk about "catharsis"
o most personality theories in the first part of the 20th Century were reactions to
Freud *
o Freud's id/ego/super-ego scheme appears in most of today's theories
o Freud was the first to identify multiple personality
o Freud brought scientific method to psychology
o
16. How did Freud describe the super-ego?
o the agent of adaptation
o an internalization of parental values *
o the primary process
o it
o the "engine
o
17. Why did Adler call sublimation "the healthy defense mechanism"?
o it provides a harmless but pleasant change of mood and emotion
o it occurs only in "nice" people
o it is the result of higher education
o it protects the ego from damage
o it benefits humanity*
o
18. In the classic Asch experiment on conformity, where subjects were asked to
compare line segments and say which one matched a standard, what happened
when there was a dissenter in the group?
o he was "shunned"
o the group was more likely to vote to punish him
o other subjects tried to persuade him
o it actually strengthened the beliefs of other subjects
o most subjects were "freed" from the conformity effect *
o
19. When we make the "fundamental attribution error" we blame somebody’s
misfortunes on...
o historical factors
o an evil leader
o their personal qualities *
o factors beyond their control
o ourselves
o
20. The dominant theory which explains "bystander apathy" blames it on…
o alienation in modern life
o modern city life
o less feeling of responsibility when there are many witnesses*
o hostility toward strangers
o people being afraid a criminal will find out they reported him and strike back

38.A study is done to determine the effectiveness of a new antidepressant. To do


this , 100 depressed patients were assigned to one of two groups, the new drug (50
patients) or a placebo (50 patients). The patients are then followed over a 6-month
period. The study is best described as a

A. cohort study
B. cross-sectional study
C. historical cohort study
D. case-control study
E. clinical treatment trial

39.The most widely fully structured diagnostic interviews used in epidemiological


surveys is

A. The Composite International Diagnostic Interview


B. Present State Examination
C. Goldberg Health Questionnaire
D. Clinical Interview Schedule
E. Structured Clinical Interview for DSM-IV

Gustatory hallucinations occur in all the following EXCEPT:

A. schizophrenia
B. temporal lobe epilepsy
C. lithium carbonate
D. disulfiram therapy
E. citalopram therapy *

50. The following are true of autoscopic (heautoscopic) hallucinations EXCEPT


:
A. first described by Féré in 1891
B. a.k.a. phantom mirror image
C. M:F = 1:1*
D. mean age of 40
E. neurological and psychiatric disorder occur in 60%

54. All the following are true regarding apraxia EXCEPT:

A. Ideational apraxia isinability to imitate or put together complex gestures and


actions, despite understanding the request
B. Ideational apraxia is always bilateral
C. Dressing apraxia is a form of ideational apraxia
D. Dressing apraxia due to lesions of non-dominant parietal lobe
E. Constructional apraxia is twice as frequent in left- as in right-sided lesions*

55. Which one is false regarding the double phenomenon ( Doppelgänger) ?

A. is an awareness of oneself as being both outside alongside, and inside oneself
B. is the subjective phenomenon of doubling
C. it is cognitive and ideational, rather than perceptual
D. often a variety of depersonalization
E.  occur only in abnormal people*
57. Passivity Phenomena does not include:

A. Thought rumination*
B. Thought broadcasting
C. ‘Made actions’
D. Passivity of emotion
E. Passivity of Impulse
58. The most common form of abnormal expression of emotion in pseudobubar palsy

A. Fatuous affectal
B. Dissociation of affect
C. Emotional indifference
D. Emotional incontinence*.
E. Perplexity

59. Feature of psychogenic pain include all the following EXCEPT:

A. less well localized; diffuse; non-anatomical


B. constant and persists unremittingly
C. more associated with underlying disturbance of mood
D. well described quality*
E. more likely to get worse in severity and extent with time

64. Which synaptic process acts primarily postsynaptically:


A. facilitation
B. adaptation
C. autoreceptor inhibition
D. shunting inhibition
E. depression

____ 1. Utilizing a variety of theoretical approaches rather than a single approach is referred to as:
a. the humanistic approach
b. the eclectic approach*
c. the nativist approach
d. the behaviorist approach

____ 6. A resting potential is:


a. an electrical signal that travels along the axon of a neuron
b. the small gap that exists between adjacent neurons
c. an electrical signal that travels along the dendrites of a neuron
d. the tiny electrical charge that exists when a neuron is neither receiving nor sending information*
________ 9. One of the key functions associated with the frontal lobes is:
a. initiation of voluntary muscle movements*
b. processing of neural signals related to touch
c. processing of auditory information
d. processing of visual information
____ 10. The aspect of the environment that is manipulated or changed by the researcher during the course of an
experiment is:
a. the independent variable*
b. the dependent variable
c. a confounding variable
d. a placebo
____ 11. When concepts are defined in terms of the way in which they will be measured, those concepts are said to
be:
a. hypothetical variables
b. externally valid
c. internally valid
d. operationally defined*
____
____ 14. The main advantage of a single-blind research study is that it minimizes the effect of:
a. expectations by both the experimenter and the participants
b. expectations in the participants*
c. the independent variable
d. any confounding variables
________ 16. Telling potential research participants the nature and possible risks of the research prior to their
participation is part of the ethical requirement of:
a. informed consent*
b. Debriefing
c. confidentiality
d. Compensation

____ 20. One of the key functions associated with the parietal lobes is:
a. initiation of voluntary muscle movements
b. processing of auditory information
c. processing of visual information
d. processing of neural signals related to touch*
____ 21. The part of the neuron that stores neurotransmitters is the:
a. terminal buttons*
b. Dendrites
c. Soma
d. Axon
____ 22. A hypothesis:
a. is a conclusion based on the results of a research study
b. is not necessary if the scientific method is being used
c. normally wouldn't be generated until a study has been completed
d. is a prediction about the characteristics of a behavior under investigation*
____
____ 24. Within the forebrain, the structure that plays an important role in the regulation of eating, drinking, body
temperature, and sexual behavior is the:
a. Thalamus
b. Medulla
c. Tectum
d. Hypothalamus*

____ 25. Glial cells are the cells in the nervous system that: ∆ Glial cells  Garbage remover ∆
a. remove waste and help neurons communicate more efficiently*
b. make the initial contact with the environment
c. convey information from one internal processing site to another
d. carry messages and commands away from the brain and spinal cord
____ 26. Early studies of brain damage indicated that patients who lacked the ability to easily understand spoken
language often had damage to:
a. an area known as Broca's area
b. the right side of the brain
c. an area known as Wernicke's area*
d. the cerebellum
____ 27. Any uncontrolled variable that changes systematically with the independent variable is:
a. a dependent variable
b. a correlation coefficient
c. a theoretical construct
d. a confounding ‫ تائه‬variable*
____ 28. The key function associated with the cerebellum is:
a. control of basic life support functions (such as heart rate and breathing)
b. control of motivation and emotional responses
c. speech production and speech comprehension
d. coordination of complex motor skills*
____
____ 29. Both sodium and potassium ions:
a. carry a negative charge
b. carry a positive charge*
c. are concentrated outside the neuron's cell body
d. are concentrated inside the neuron's cell body
____ 30. Action potentials:
a. are stronger when the incoming stimulation is more intense
b. are seldom strong enough to reach the terminal buttons
c. travel more slowly if the incoming stimulation is less intense
d. are generated in an all-or-none fashion*
____
____ 31. One of the key functions of the amygdala is:
a. the control of motivational and emotional behaviors*
b. acting as an important gathering point for sensory input
c. the regulation of eating, drinking, and sexual behavior
d. acting as a gateway into long-term memory
____
____ 32. Within the hindbrain, the structure that is linked to the control of general arousal is the:
a. Amygdala
b. Cerebellum
c. substantia nigra
d. reticular formation*

____ ____

34. .Limbic structures include: all the following EXCEPT:

A. Insula*
B. Fornix
C. Cingulum
D. Hippocampus
E. Amygdala

35.The following may be due to disorders of the extrapyramidal system EXCEPT:

A. Clasp-knife rigidity*
B. Tremor of the lips ?
C. Spasmodic torticollis
D. Akathisia
E. Festinant gait

36.The following structures are part of the neo-cortex EXCEPT:

A. Broca's area
B. Hippocampus*
C. Operculum 
D. Superior temporal gyrus
E. Cingulum [F for cingular gyrus]

37.The following are associated with disorder of the parietal lob EXCEPT:

A. Gerstmann's syndrome
B. Astereognosis
C. Tactile inattention 
D. Dressing apraxia
E. Word blindness* 

 38.The cerebellum contains one of the following structures:

A. Dentate nucleus*
B. Substantia nigra
C. Red nucleus
D. Hypocampus
E. Globus pallidus

39.Characteristic clinical signs of cerebellar dysfunction include all the following EXCEPT:.

A. ataxia
B. hypertonia*
C. disdiadochokinesia
D. nystagmus
E. intension tremor

40. Characteristic features of Gerstmann's syndrome include all the following EXCEPT:

A. Ophthalmoplegia*
B. Finger agnosia
C. Acalculia
D. Agraphia
E. Left right disorientation

41 A frontal meningioma can cause all the following EXCEPT:


A. [a]visible changes on the skull Xray
B. [b]unilateral papilloedema
C. [c]epilepsy
D. [d]loss of taste*
E. [e]expressive dysphasia

42. Neck stiffness may be caused by all the following EXCEPT:

A. acute poliomyelitis
B. cerebral AIDS
C. thoracic spondylosis*
D. Huntingdons Chorea
E. subarachnoid heamorrhage

43. Frontal lobe lesions is associated with:

A. agraphia
B. apathy*
C. memory impairment
D. hypersexuality
E. dyscalculia

44. The following symptom suggests a cortical rather than brain-stem lesion:  

A. diplopia
B. vertigo
C. dysarthria
D. dysphasia*
E. dysphagia

45. Unconsciousness can be induced by a small area of damage in one of the following: 

A. reticular formation*.
B. cerebral cortex
C. thalamus
D. limbic system
E. internal capsule

46. The grasp reflex is a recognised symptom ofone of the following:

A. depressive psychosis
B. cerebellar disorders
C. hypomania
D. widespread disorderes of the brain stem
E. frontal lobe lesions*

48 Recognised features of upper motor neurne lesions of the facial nerve include all the
following EXCEPT:

A. trigeminal neuralgia*
B. sparing of the upper part of the face from paralysis [upper spares upper]
C. paralysis of the muscles of the lower part of the face
D. an apparent abscence of paralysis during involuntary contraction of the muscles of facial
expression
49. The parts visible on the outer surface of an undamaged postmortem brain include the
following EXCEPT :

A. amygdaloid nuclei
B. optic chiasma
C. dura mater
D. trigeminal nerve roots

50. Recognised features of the stimulation of the parasympathetic nervous system include all
the following EXCEPT:

A. penile erection
B. ejaculation
C. pupillary constriction
D. salivation
E. bronchiolar dilatation*

51..Prolactin is stimulated by all the following EXCEPT:-**

A. DA -agonists.*
B. TRH.
C. 5HT.
D. Stress.

52. The following nucleus is neurosecretory:-**

A. Edinger-Westphal.
B. Red nucleus.
C. Para-ventricular*.
D. Amygdala.
E. Caudate nucleus

53. Anterior pituitary function is regulated by all the following EXCEPT:

A. The nigro-striatal pathway*


B. Hypothalamic control
C. CNS control
D. Negative feedback control
E. Enviromental influence

54. The following are true regarding the reticular activating system EXCEPT:

A. Is excited by somatic afferents


B. Activates all areas of the cerebral cortex
C. Is inhibited by auditory afferents*
D. Produces behavioural arousal
E. Causes an increase in EEG fast wave activity

55. The following are true regarding Endorphine EXCEPT:**+

A. associated with gate theory of pain


B. never found extra cerebrally*.
C. increase after exercise.]
D. act on dopamine receptors
E. potentiate dopamine

56.The following drugs cause tremors EXCEPT:-**

A. Haloperidol.
B. Amitriptyline.
C. MAOITS.
D. Lithium.
E. BZD*.

57.Drugs effective in the symptomatic treatment of delirium tremens include all the following
EXCEPT:**

A. Ascorbic acid.
B. Amitriptylline*.
C. Haloperidol.
D. Diazepam.
E. Chlormethiazole

58.The following drug is likely to produce ataxia at a therapeutic dose:-**

A. Haloperidol'
B. Imipramine.
C. Chlorpromazine.
D. Carbamazepine*.
E. Pimozide.

59.Beta Adrenoceptor agonists cause all the following EXCEPT:

A. Tremor.
B. Pupil dilatation*
C. Increased blood flow to the muscle
D. Stronger heart contraction.
E. Increases melatonin secretion

60.The following drugs cause sedation in majority of cases EXCEPT:-**

A. Desipramine*.
B. Trimipramine.
C. Mianserin.
D. Dothiepin.
E. Amitryptylline.

61.The following statements are true EXCEPT:-

A. 1st pass metabolism in benzodiazepines produces active metabolises.


B. Lithium potentiates action of TCADS.
C. Tryptophan potentiates the action of TCADS.
D. Lithium is less effective in elderly compared to the middle aged patients in prophylaxis
for depression*.
E. Thioridazine causes retinal pigmentation
62 The following are true regarding Neuroleptic malignant syndrome EXCEPT:-**

A. Increased neutrophils.
B. Males affected more than the females.
C. Untreated mortality is 20%.
D. Worse with drug holidays*.
E. Caused by D2 receptor blockade.

63.Prolonged administration of lithium carbonate may cause all the following EXCEPT:

A. weight gain
B. craving for carbohydrates*
C. psoriasis like lesions
D. thirst
E. osteoporosis

64. In the trial of a new Psychiatric drug, one is true:

A. permission from an ethical committee is not advised


B. double-blind procedures are mandatory by law
C. th e patient's second degree relatives must be included in consenting
D. Patients 16 and over can give consent*
E. the solicitor must be informed

65. Noradrenaline is a precursor of:

A. adrenaline*
B. tyrosine
C. phenylalaninine
D. dopamine
E. typotophan

66. The following phaimacokinetic principles are true EXCEPT

A. Many sustained release preparations rely on colonic absorption.


B. Low bioavailability usually implies high first pass metabolism.
C. At steady state, concentrations of a drug increase by upto 20% from one dose to the the
next*
D. For lipid soluble drugs, reabsorption occurs in the renal tubule

67.The following test assumes a normal distribution:

A. T tests*.
B. X2 (chi-squarred)
C. Analysis of variance by ranks
D. Spearman rank]

68.The following statements is true of prospective but untrue of retrospective studies:

A. Patients can be randomly assigned to treatment categories or groups*


B. Evaluative measures can not be modified on the basis of feed-back results ?
C. Information from clinical files can be used in the analysis of results
D. An interview with the patient's relatives is a necessary procedure
E. All the relevant information can be obtained from existing records.

69.The effectiveness of a given treatment cannot be tested without using:

A. An untreated control
B. Double-blind procedure
C. A valid measure of change*
D. Patient self-reports
E. Physiological measures

70.In normal distribution:

A. The mean and mode fall at the same point*


B. The standard deviation is equal to the value of the mean
C. Several sources of variation may be represented simultaneously ?
D. The two 'tails' are unequal and not symmetrical,
E. About one-third of the cases fall within one standard deviation on either side of the mean

71.Incidence refers to:

A. The number of new cases emerging in a designated period and population*


B. The point prevalence of an illness
C. The period prevalence of an illness
D. The readmission rate
E. The number of beds occupied in a designated population

72 All the following are true regarding. reliability of an instrument EXCEPT:

A. is high if.the ratiing of the same instrument by different raters using that instrument are
similar
B. concern the reliability of it's measurements
C. means it measures what it is intended to*
D. is high if its measurents accord with those of another instrument supposed to have the
same function
E. is concerned with the reproducibility of its measurements

73. Regarding a placebo in a clinical trial, one is false:

A. should be given with as much confidence as the'active' drug


B. is pharmacologically inert
C. should be identical in appearance to the drug being studied
D. has no effects*
E. is best adminstered by someone who is unaware of the tablet's contents

74. All the following are true regarding double blind trials EXCEPT:

A. An effective means of comparing two active drugs


B. An effective means of comparing an active drug and a placebo
C. Including a design in which the judge is not aware of which patient received the drug
being tested
D. An effective means of comparing the efficacy of individual and group psychotherapy*
E. Including a design in which the patient is not aware of which patient received the
treatment being tested

75.The following are non-parametric tests EXCEPT:

A. Kolmogorov Syndrome
B. Mann Wittney U
C. Wilcoxon
D. ANOVA* ]
E. McNemar

76. Halo Effect is:

A. the judging of another by one"s own attitudes


B. the overall judging of an individual on the basis of one attribute*
C. the judging of people by the standards of one person
D. the judging of an individual by the moral standards one approves of
E. the judging of an individual by his behaviour in the interview situation

77.The following statements are true of acetylcholine in the nervous system EXCEPT:

A. it is the chemical transmitter of the parasympathetic nervous system


B. some receptors are blocked by atropine [blocks muscarinic receptors]
C. some receptors are blocked by curare [blocks nicotinic receptors
D. it stimulates beta receptors in the sympathetic nervous system*
1. Multipolar neurone has

A. one axon and more than one dendrite


B. one axon and one dendrite
C. one dendrite and one axon
D. one dendrite and more than one axon
E. no axon and more than one dendrite

2. The cells in the cerebral cortex can be divided into the following basic neuronal types
EXCEPT:
A. pyramidal cells
B. stellate cells (granule cells)
C. multiform (polyform) cells
D. cells of Martinotti
E. vertical cells*

3. The Blood brain barrier consists of all the following EXCEPT:


A. Arachnoid mater
B. dura mater*
C. Endothelial lining of the brain capillaries
D. Gliovascular membrane (perivascular end feet of astrocytic processes)
E. Capillary basement membrane
4. Diencephalon includes the following structures EXCEPT:

A. Thalamus
B. Epithalamus (inc. pineal gland, and
habenular gland)
C. Hypothalamus (inc. pituitary gland)
D. Subthalamus
E. Tectum*

5. The mesolimbic dopaminergic system innervates the following structures EXCEPT:


A. olfactory tubercle
B. dorsolateral hypothalamus*
C. stria terminalis
D. cingulate cortex
E. parahippocampal gyrus

6. The Pyramidal System descends through the following structures EXCEPT:


A. corona radiata
B. internal capsule
C. ventral midbrain
D. thalamus*
E. medulla oblongata (forms the pyramids)

7. basal forebrain contains the following nuclei EXCEPT:


A. diagonal band of Broca
B. nucleus accumbens
C. septal nuclei
D. substantia innominata
E. red nucleus*

8. Functions of the dorsolateral prefrontal cortex (DLPFC) include all the following
EXCEPT:
A. problem-solving
B. perceptual judgement
C. social behavior*
D. programming and planning sequences of behaviour
E. verbal regulation

9. Gerstmann’s syndrome is characterized by all the following EXCEPT:

A. right-left disorientation
B. finger agnosia
C. dysgraphia
D. dyscalculia
E. dysmnesia*

10. Origin of the Glutamate system includes all the following EXCEPT:

A. cerebral cortical granule cells*


B. hippocampal pyramidal cells
C. primary sensory afferents
D. cerebellar granule cells
E. cerebellar climbing fibres

11. The following are true regarding GABA systems EXCEPT:


A. GABA is the principle inhibitory neurotransmitter
B. formed from the transamination of alpha-ketoglutarate
C. is particularly absent in the substantia nigra, globus pallidus, and nucleus
Accumbens*
D. GABA reduces the likelihood that postsynaptic neurons will fire
E. GABA systems’ normal inhibition of dopamine systems is lost in
Huntington’s disease

13. .Prolactin is stimulated by all the following EXCEPT:

A. DA -antagonists.
B. TRH.

C. 5HT.

D. Stress.

E. Adrenaline*..

14. Anterior pituitary function is regulated by all the following EXCEPT:y

A. The nigro-striatal pathway*


B. Hypothalamic control

C. CNS control

D. Negative feedback control

E. Enviromental influence

15. Parasympathatic action characteristically includes:

A. Dilatation of the pupil


B. Increase in heart rate

C. Decrease in salivation

D. Bronchial constriction*

E. Decrease in peristalsis

17. 5HT is involved in all the following EXCEPT:

A. Sleep.
B. Sex.

C. Aggressive outbursts.
D. Thought.*

E. Mood.

19. .With regard to the EEG one of the following is False:

A. an adult under severe stress is likely to show a frequency of >15Hz


B. a relaxed adult with eyes closed would show a frequency of 12-30Hz*

C. a child with a behavioural disorder would show a low frequency high amplitude
pattern? [dysmaturation?]

D. benzodiazepine overdose causes slow wave activity

E. a person in deep sleep shows a frequency of <7 Hz [delta &theta]

30. A descriptive study typically would be used for


a. A comparison of two antianxiety drugs
b. The evaluation of the results of a new medical treatment
c. A comparison of two kinds of sampling methods
d. Documentation of a cluster of Kaposi’s sarcoma cases*
e. Evaluation of the effects of self-instruction versus lecture on anemia

31. In studying human development, the remeasurement of a crosssectional


sample of people after a given fixed interval of time has passed is
called
a. Time-order design
b. Sequential design*
c. Multivariate design
d. Cross-sectional design
e. Longitudinal design

32. The revised third and fourth editions of the Diagnostic and Statistical
Manual of Mental Disorders (DSM) offer an improvement over earlier psychiatric
classifications. Their major advantage is that they
a. Identify mental illness as a disease
b. Deal with predisposing factors
c. Confirm and support the nosologic system
d. Present an all-inclusive classification system
e. Present a multiaxial classification system*

33. A meta-analysis is primarily


a. A combination of the data of several studies to help answer a hypothesis*
b. A power calculation of different results
c. A compilation and comparison of similar methods studying the same issue
d. An analysis of different diagnostic criteria to generate a norm
e. Used to reconcile experimental differences in treatment regimens

34. In psychiatry, the electroencephalogram (EEG) can be helpful in the diagnosis of


a. Intermittent explosive disorder *
b. Panic disorder
c. Bipolar disorder
d. Social phobia
e. Frotteurism

65. .Blockage of the left posterior cerebral artery may cause all the following EXCEPT

A. Aphasia.*
B. Dyslexia.
C. Hemianopia.
D. Visual agnosid.
E. Diplopia.

66. Non-dominant temporal lobe abnormalities may produce:

A. Aphasia.
B. Reduced appreciation of music*.
C. Impaired learning of new words.
D. Difficulty planning tasks.
E. Confabulation.

67. In frontal lobe damage the following are commonly found EXCEPT:

A. Long term memory impairment*.


B. Disinhibition.
C. Impaired attention.
D. Impaired judgment.
E. Apathy.

68. The Grasp reflex can be found in all the following EXCEPT:

A. Frontal lobe tumours.


B. Babies under the age of six weeks.
C. Motor neurone disease.
D. Wemicke's psychosis*.

38.The following are common manifestations of TIA (transient ischemic attacks) of the
vertebrobasilar system EXCEPT:.

A. Transient vertigo .
B. Transient ocular palsy .
C. Transient ipsilateral monocular blindness*
D. drop attacks .

40.In the assessment of parietal lobe lesions, All are true EXCEPT:

A. Ideational (ideomotor) apraxia is the inability to copy a movement when the instruction is
understood there is no motor weakness.
B. Apparent dressing apraxia may be caused by left sided neglect.
C. Constructional apraxia is revealed by an inability to copy two dimensional designs using
matchsticks.
D. Difficulty in writing is likely to be caused by astereognosis.* 
E. Prosopagnosia is inability to recognize familiar faces.

41.Parietal lobe symptoms include the following EXCEPT:


SAMPLE QUESTIONS

10. Data are empirical when they are

A. externally valid
B. internally valid
C. falsifiable
D. observable *

11. What term refers to a researcher's responsibility to explain the nature and purpose of a study to subjects
at the end of an experiment?

A. Confidentiality
B. Debriefing *
C. Interrogation
D. Nondisclosure

12. Nonexperimental methods should be used when

A. an experiment would be unethical or impossible *


B. external validity is less important than internal validity
C. the experimental hypothesis is not testable
D. a causal relationship must be established
F. Astereognosis.
G. Dressing apraxia.
H. Acalculia.
I. Ideomotor apraxia.
J. Aphasia*. 

2.Parasympathetic signs include the following EXCEPT:-

F. Slow heart rate.


G. Dilated pupils*.
H. Increased acid secretion of stomach.
I. Salivary secretion.
J. Decreased diameter of the bronchioles.

26.Confusional state may be caused by all the following EXCEPT:

F. Paracetamol.
G. Cimetidine.
H. Procyclidine.
I. Dothiepin.
J. Methylcellulose*.

Sample Questions for Human Behavior, Test 1, 2007

‫كلها معادة‬
18.
16. Jenny ran
When calculating
all treatment
a t statistic
conditions
for independent
at night, to groups,
control for
the the
numerator
effect ofistime
the of day. She used the control
technique of
A. variances of the groups divided by the number of subjects
A. number
B. balancingof degrees of freedom
C.
B. squared
elimination
deviations from each treatment mean
C. difference
D. constancy of conditions*
between the treatment group means
D. isolation of directions
19. When the computed F ratio is significant,
17. Scientists generally reject the null hypothesis if the probability of obtaining a pattern of data by
A. each treatment mean is significantly different from every other treatment mean
chance alone is less than
B. there is a significant difference across the treatment means *
C. each treatment mean is significantly different from the grand mean
A. D..01 MSW is larger than MSB
B. .05 *
20. C. In.10a two-way ANOVA, between-groups variability includes all of the following EXCEPT variability
D. .15with
associated
E. .5
A. factor one
B. factor two
C. the interaction between the factors
D. the higher-order interactions *

21. The appropriate distribution for chi square is based on

13. A. the total number


A regression line of subjects
B. the degrees of freedom *
C.
A. whether
depicts athe nullrelationship
causal hypothesis was rejected
between twoorvariables
not
D.
B. the numerator
depicts of the
the typical formula
score in a sample
C. is a line of best fit on a scatterplot *
22. Results that
D. is the are externally
vertical axis on avalid meet two requirements: they are internally valid and they can be
scatterplot

A.
14. demonstrated
Which of the following is an analytic statement?
B. tested
C. replicated *
A. Dieters' weights will fluctuate.*
D. manipulated
B. Mypet is a cat and not a cat.
C. Lucy is married and single.
23. The overallispurpose
D. Today Sundayofand
theTuesday.
discussion section of a research report is to

A. evaluate and interpret the experiment's results*


15. Decreasing
1 explainthe sample size hypothesis was developed
B. how the research
C. briefly review prior research in the area
A. increases confounding
D. summarize the study
B. decreases size effect
C. increases external validity
D. decreases statistical power*
1. The immune system is not an autonomous system. This discovery was
confirmed by demonstrating that the immune system can be altered by
a. Stress
b. Suppressed emotions
c. Diet
d. Conditioning*
e. Relaxation

2. For almost three years, a 50-year-old woman has been caring for her mother who is
chronically ill with Alzheimer’s disease. A recent immunologic assessment of the caregiver
daughter found that her
a. Cellular immune system control of latent viruses was poor*
b. Percentage of T lymphocytes was high
c. Helper/suppressor ratio was higher than normal
d. Circulating neutrophils were decreased in number
e. Natural killer cells were higher in number

3. In studying the interaction of psychological stress and DNA repair, suppressed


DNA repair was found in persons with
a. Chronic stress more often than those with acute stress
b. Low distress over an extended period of time
c. A diagnosis of major depression*
d. Increased anxiety and depression from bereavement
e. Recently diagnosed cancer

4. A 40-year-old man develops depressed mood, anhedonia, initial and


terminal insomnia, loss of appetite, a 10-lb weight loss, and difficulty with
sexual arousal. The clinical features of the patient’s psychiatric illness suggest
dysfunction of the
a. Frontal lobes
b. Pituitary
c. Hippocampus
d. Hypothalamus*
e. Corpus callosum

5. The fact that the pituitary secretion of endorphins is closely linked to the secretion of
adrenocorticotropic hormone (ACTH) suggests that endorphins facilitate the ability to respond to
a. Retarded growth
b. Hypertension
c. Stress*
d. Chronic pain
e. Tachycardia

6. A 25-year-old male reports to his physician that he has not been able to
sleep for over two days and has been having “strange reactions.” These
reactions are most apt to be caused by
a. Increased levels of blood cortisol
b. Physiologic stress in response to sleep deprivation
c. The effects of the rebound phenomenon
d. Perceptual distortions*
e. Feelings of excessive tiredness

7. A 47-year-old, recently divorced, male bookkeeper experienced a gnawing


epigastric pain over the past week, very similar to a bout of pain he had
experienced a year ago and which was diagnosed as a duodenal ulcer. The
factor most likely responsible for the exacerbation of his ulcer is
a. Expression of a characteristic personality
b. Chronic anxiety and stress*
c. Frequent consumption of spicy foods
d. Socioeconomic status
e. Family history of ulcer disease

8. A 26-year-old woman has a several-year history of ulcerative colitis.


Recently she has been treated with alprazolam 0.5 mg tid. This pharmacologic
treatment can be expected to
a. Result in long-term improvement
b. Reduce interpersonal dilemmas
c. Produce a mild stimulus
d. Be used without development of tolerance
e. Reduce high levels of stress*

9. The diathesis-stress model of psychophysiologic disorders postulates


the presence of which of the following major factors?
a. Inadequate coping style
b. Individual response stereotype*
c. Lack of health belief resolution
d. Adequate homeostatic restraints
e. Unconditioned stimulus

10. Harry Harlow’s work with inanimate surrogate mothers for rhesus
monkeys suggests that the early experience critical to the ultimate development
of normal attachment and sexual behavior is
a. Positive reinforcement
b. Protection from danger
c. Contact comfort*
d. Need-reduction by nursing
e. Sexual stimulation

11. The theory of pain that states that psychological processes directly
exert influence on the pain perception process is the
a. Gate control theory*
b. Nociceptor theory
c. Pattern theory
d. Polymodal nociceptor theory
e. Specificity theory

12. The biologic mechanism that allows behavioral factors to have the
greatest influence on the development of coronary heart disease is the
a. Complex interaction of sympathetic-adrenal-medullary (SAM) and pituitaryadrenal-
cortical activity*
b. Acute behavioral stressors that can raise thresholds for ventricular fibrillation
c. Limbic system that allows emotional responses to activate the endocrine system
d. Heart rate and pressor response to behavioral stressors that increases the turbulence
and sheer stress that promote arterial injury
e. Stress-induced increased levels of corticotropin (ACTH) and cortisol secretion
and decreased levels of brain norepinephrine

13. Which of the following is the most powerful endogenous opioid peptide?
a. Dynorphin*
b. Morphine
c. Neoendorphin (-endorphin)
d. Endorphin
e. Enkephalin (-endorphin)

14. The hormone with the greatest role in aggression is


a. Thyroxine
b. Testosterone*
c. Estrogen
d. Progesterone
e. Aldosterone

15. When an inactive substance or condition induces a therapeutic change,


the procedure (result) is called
a. Modulated conditioning
b. Reaction formation
c. Placebo effect*
d. Fantasy reaction
e. Nonpharmaceutical reaction

16. Damage to dopamine neurons in the midbrain is a central feature of


the pathophysiology of Parkinson’s disease. The loss of midbrain dopamine
in this disease is accompanied by
a. An increase in the dopamine transporter
b. A decrease in dopamine 1 receptor density
c. An increase in dopamine 2 receptor density
d. A decrease in dopamine synthesis in remaining dopamine neurons
e. An increase in both dopamine 1 and dopamine 2 receptor density*

17. A type I diabetic patient has been treated with relaxation techniques
daily for one month. This treatment is likely to affect the management of
her diabetes by
a. Increased levels of plasma cortisol
b. Increased sensitivity to insulin
c. Increased glucose-stimulated secretion of insulin
d. Significant improvement in glucose tolerance*
e. No significant change in requirements for exogenous insulin

18. A college student takes part in a sleep laboratory experiment in which


he is awakened repeatedly when his electroencephalogram (EEG) indicates
that he has entered rapid-eye-movement (REM) sleep. This disruption of
normal sleep is most likely to produce
a. A rebound phenomenon of increased dreaming*
b. An increase in anxiety and irritability
c. Acceleration of memory formation of emotionally toned words
d. A decrement in intellectual function
e. A temporary increase in nightmares

19. Which of the following statements about natural killer cells is true?
a. Their activity increases with the secretion of corticosteroids through the
hypothalamic-pituitary-adrenal axis
b. They play a role in inhibition of tumor growth*
c. They are ineffective in deterring the spread of distant cancer
d. Their activity increases under conditions of psychosocial stress

20. In studies of older twins, which of the following characteristics has the
highest degree of heritability?
a. Nonclinical depression symptoms
b. Longevity
c. Interpersonal skills
d. General cognitive ability*
e. Intellectual ability

21. Which of the following characteristics has the greatest genetic heritability
change over time?
a. Blood pressure
b. Body weight*
c. Alcoholism
d. Heart rate
e. Nicotine addiction

22. Which of the following anxiety disorders has the strongest degree of
heritability?
a. Panic disorder
b. Generalized anxiety
c. Specific phobia*
d. Posttraumatic stress disorder
e. Obsessive-compulsive disorder

23. Parents of a child with attention deficit hyperactivity disorder (ADHD)


consult with you to inquire about the heritability of the disorder. The husband
had attention span and learning problems in secondary school. One
of his brothers did also, and an uncle may have had learning problems. You
response would include the fact that ADHD has a heritability of approximately
a. 70%*
b. 60%
c. 50%
d. 40%
e. 30%

24. Of the following illnesses or syndromes, which is most related to genetic


factors?
a. Schizophrenia*
b. Panic disorder
c. Bipolar disorder
d. Alcoholism
e. Antisocial personality

25. Ethology was originally developed by which of the following concepts?


a. Behavior is best studied under controlled conditions
b. Behavior is best studied comparatively
c. Behavior is the result of interaction between genetic endowment and environment
d. Innate behavior is triggered by a sign stimulus or releaser
e. Previous learning is unnecessary for the successful expression of a fixed action*
pattern

26. A young man consults you because his older brother has schizophrenia
and he is concerned about his risk of developing schizophrenia. Obviously
several issues need to be addressed. With regard to the genetic risk, a
sibling of a patient with schizophrenia is more likely to develop the disease
than a person with a negative family history by a factor of about
a. 3
b. 6
c. 9*
d. 12
e. 15

27. The most frequent genetic cause of mental retardation is


a. Bartholin-Patau syndrome
b. Edwards’ syndrome
c. Down’s syndrome*
d. Turner’s syndrome
e. Klinefelter’s syndrome

28. Which of the following studies is most effective in estimating the


genetic risk of an alcoholic offspring’s becoming an alcoholic if adopted or
raised by a nonalcoholic family?
a. Family studies
b. Studies of twins reared together
c. Adopted twin studies
d. Nontwin adoption studies*
e. Pedigree and linkage studies

29. A person with Klinefelter’s syndrome has the genotype


a. XYX
b. YY
c. XXY*
d. XYY
e. XXX

30. Heredity accounts for approximately what percentage of total variation


in IQ scores within a family?
a. 5%
b. 25%
c. 50%*
d. 75%
e. 100%

31. From studies of twins reared together and reared apart, which of the
following characteristics has been confirmed as having the highest heritability?
a. Aggressiveness*
b. Extroversion
c. Empathy
d. Stress reaction
e. Neuroticism

32. A young woman had an initial episode of psychosis at age 19, with
paranoid delusions, auditory hallucinations, and disorganized behavior.
She responded well to an atypical neuroleptic, but has not been able to live
independently and develop an occupation. She has had two subsequent
psychotic episodes similar to the initial one. The etiology of her chronic
psychiatric illness is most strongly related to
a. Environmental factors
b. Polygenic inheritance*
c. A chromosomal abnormality
d. Simple recessive inheritance
e. An inborn error of metabolism

33. A disorder resulting from a single gene defect that may produce severe
behavioral disturbance is
a. Major affective disorder, bipolar type
b. Dyslexia
c. Phenylketonuria*
d. Porter’s syndrome
e. Down’s syndrome

34. Which of the following choices shows the highest heritability in later
life?
a. Type A behavior
b. Locus of control
c. Depressive symptoms
d. Dementia
e. Self-perceived competence*

35. Which of the following traits has been most useful for behavior geneticists
studying heritability of personality?
a. Agreeableness and likeability
b. Conscientiousness and conformity
c. Culture and openness
d. Extraversion and neuroticism*
e. Friendliness and achievement

36. Ethology has made major contributions to understanding human


behavior through the following concept:
a. Instinctual driver
b. Ethnic bonds
c. Operant conditioning
d. Critical period*
e. Developmental stages

37. An infant smiles every time the parents approach the crib and look at
him. Emotional expressions in an infant, such as smiling, are stereotyped
sequences of movements that are under the control of
a. A specific sign stimulus*
b. Genetic influences
c. A conditioned response
d. Imprinting
e. Cultural determinants

38. Which of the following is least likely to be considered an ontogenetic


stage of synaptic development and modification?
a. Synapse formation under genetic and developmental control
b. Maintenance of newly developed synapses occurring during critical periods
c. Regulation of transient and long-term effectiveness of synapses
d. Integration of cellular structure for human mentation*
e. Alteration of preexisting pathways and development of new patterns

39. Major patterns of human behavior have the potential of containing


major innate (i.e., genetic, or not learned) components as well as learned cultural
components. Which of the following behavioral patterns is a learned
cultural behavior?
a. Intelligence
b. Brow flash response
c. Facial expressions of anger, fear, disgust, and joy
d. Smiling
e. Handshake greeting*

40. The mammillary bodies are part of the


a. hypothalamus.
b. thalamus.
c. hippocampus.
d. basal ganglia.

41. In order to subdivide the cortex into 52 distinct areas, Korbinian Brodmann made
comparisons between different cortical regions using
a. stained tissue samples, which revealed differences in cellular morphology and
architecture*.
b. extracellular single-cell recording, which revealed differences in neuronal response properties
such as selectivity and sensitivity.
c. histochemical techniques, which revealed differences in chemical content (e.g.
neurotransmitters).
d. cortical stimulation, which revealed differences in evoked behavioral responses.

42. Hemorrhagic lesions and degeneration of the mammillary bodies as a result of a thiamine
deficiency is associated with
a. Korsakoff’s disease*.
b. Pick’s disease.
c. Huntington’s disease.
d. Herpes simplex encephalitis.

43. Which of the following techniques is the most direct measure of neural events?
a. PET
b. CT
c. EEG*
d. MRI

44. A cluster of symptoms that appear together following brain damage is called a
a. single dissociation.
b. double dissociation.
c. syndrome*.
d. degenerative disorder.

45. Which of the following would be an example of a correlational strategy of academic


research?

a. testing a hypothesis with an experimental group


b. having a group fill out questionnaires*
c. using the scientific method
d. a clinician diagnosing his/her patient
e. using the electric shock method to illustrate classical conditioning

46. If you had a correlation of positive one, the related items would be:

a. completely unrelated
b. moderately correlated
c. very highly correlated
d. identical*
e. lowly correlated

47.In analyzing data, measurements are sometimes categorical, for example "smiles" or
"frowns". This type of measurement is called

a. qualitative measurement*
b. quantitative measurement
c. reliability
d. validity
e. psychometrics
48..A25 year old male patient sustains a serious head injury in an automobile accident. He had
been aggressive and assaultive, but after the accident he is placid and cooperative. He makes
conditional suggestive gestures and comments and masturbates a great deal. The patient’s injury
is most likely to affect what portion of the brain?
(A) Frontal lobes*
(B) Temporal lobes
(C) Parietal lobes
(D) Occipital lobes
(E) Basal ganglia

49. Sleep-arousal mechanisms are effected by damage to the


(A) cerebellum
(B) basal ganglia
(C) thalamus
(D) reticular system*
(E) amygdale

50.Increased pain perception is associated with damage to what area of the brain?
(A) Thalamus*
(B) Corpus callosum
(C) Basal ganglia
(D) Reticular system
(E) Hypothalamus

51.What is the major area of the brain implicated in Alzheimer’s disease and amnestic disorders?
(A) Amyglada
(B) Parietal lobes
(C) Hippocampus*
(D) Thalamus
(E) Basal ganglia

52.All of the following statements about neurotransmission are true EXCEPT


(A) receptors are proteins in the membranes of neurons
(B) presynaptic receptors bind neurotransmitters
(C) postsynaptic receptors change the ionic condition of membranes
(D) neurotransmitters exert their effects only on postsynaptic neurons*
(E) postsynaptic receptors bind neurotransmitters

53.Which of the following statements about dopamine is true?


(A) At autopsy, the brains of schizophrenics show decreased numbers of dopamine receptors
(B) Blockade of dopamine receptors results in decreased prolactin levels
(C) Antipsychotic drugs that block postsynaptic dopamine receptors may cause Parkinson-like
symptoms*
(D) Dopamine enchances the release of prolactin
(E) Patients with mania show dopamine hypoactivity

54.Which of the following statements about GABA is true ?


(A) It is primarily an excitatory neurotransmitter
(B) It is an amino acid neurotransmitter*
(C) It mediates postsynaptic inhibition in the CNS
(D) Increased GABA activity has been reported in Huntington’s disease GABA activity is seen
in epilepsy

55. Allof the following structures connect the two cerebral hemispheres EXCEPT
(A) the corpus callosum
(B) the anterior commissur
(C) the hippocampal commissure
(D) the posterior commissure
(E) the amygdale*

56..Degeneration of cholinergic neurons is involved primarily


(A) mania
(B) depression
(C) movement disorders*
(D) anxiety
(E) sleep disorders

57.Which of the following statements about monoamines and their metabolites is true?
(A) Mania is associated with dopamine hypoactivity
(B) Free dopamine is metabolozed by 5-HIIA
(C) There is more plasma monoamine oxidase (MAO) in unmedicated schizophrenics than in
controls
(D) There is no association between levels of MHPG and suicidal behavior
(E) Decreased HVA is seen with clinical improvement in patients treated with neuroleptics*

58..Which of the following signs is characteristic of a patient in non-REM sleep?


(A) Paralysis of skeletal muscles
(B) Descreased blood pressure*
(C) Increased pulse
(D) Increased respiration
(E) Agitation

59..Allof the following statements are valid about intelligence testing EXCEPT
(A)affected by the subject’s motivation
(B)culturally biased
(C) tend to emphasize verbal ability
(D)high scores are incompatible with psychosis*
(E)measures skills enhanced by traditional education

60 Predictive validity is
(A) the extent to which knowledge that a person has a particular mental disorder is useful in
predicting the future course of the illness
(B) related to management and treatment
(C ) relevant to the specificity with which patients with bipolar I disorder improve when treated
with lithium
(D) the basis on which Emil Kraepelin differentiated manic-depressive psychosis from dementia
precox
(E) all the above*

61..Prevalence is the
(A) proportion of a population that has a condition at one moment in time*
(B) ratio of persons who acquire a disorder during a year’s time
(C) risk of acquiring a condition at some time
(D) standard deviation
(E) rate of first admissions to a hospital for a disorder

1. According to psychoanalytic theory, the part of the personality that strives for immediate
gratification
of basic drives is the:
A) id.*
B) ego.
C) superego.
D) collective unconscious.

2. When she was 8 years old, Tina was sexually abused by her uncle. At 14, Tina felt
uncomfortable whenever she saw this uncle but was unable to understand why she felt this way.
A psychoanalyst would be most likely to suggest that Tina is using the defense mechanism of:
A) repression.*
B) reaction formation.
C) rationalization
. D) regression.

3. Characteristic patterns of behavior and motivation are called:


A) aptitudes.
B) fixations.
C) projections.
D) traits.*

4. A person who is careless and disorganized most clearly ranks low on the Big Five trait
dimension known as:
A) emotional stability.
B) extraversion
. C) agreeableness.
* D) conscientiousness.

5. After experiencing inescapable brutalities as a prisoner in a Nazi concentration camp, Mr.


Silver became apathetic, stopped eating, and gave up all efforts to physically survive the ordeal.
Mr. Silver's reaction most clearly illustrates:
A) an inferiority complex.
B) repression.
C) learned helplessness*.
D) an internal locus of control.

6. Mental health workers label behavior as psychologically disordered when they judge it:
A) prejudicial, unconsciously motivated, ingenuine, and insane.
B) biologically based, unconsciously motivated, aggressive, and difficult to change.
C) selfish, habitual, and avoidable.
D) atypical, disturbing, maladaptive, and unjustifiable.*

7. DSM-IV is a widely used system for:


A) identifying the causes of psychological abnormality.
B) distinguishing sanity from insanity.
C) treating depression.
D) classifying psychological disorders.*

8. Kim is so alarmed by spiders and insects that she avoids most outdoor activities and even
refuses to enter the basement of her own house alone. Kim appears to suffer from
A) obsessive-compulsive disorder.
B) dissociative disorder.
C) mood disorder.
D) phobia*.
9. Chuck is preoccupied with thoughts of jumping out the window of his tenth-floor apartment.
In order to reduce his anxiety, he frequently counts his heartbeats aloud. Chuck would most
likely be diagnosed as experiencing :
A) panic disorder
B) generalized anxiety disorder
C) obsessive-compulsive disorder*.
D) phobia.

10. Andre suffers from acrophobia, a fear of high places. Andre's therapist suggests that his
reaction to heights is a generalization of the fear triggered by a childhood playground accident in
which he fell off a sliding board. The therapist's suggestion reflects a ________ perspective.
A) learning *
B) psychoanalytic
C) trait
D) humanistic

11. Mr. Choi's therapist wants to help him become aware of his conflicting childhood feelings of
love and hate for his parents. The therapist's goal best reflects a primary aim of:
A) client-centered therapy
B) cognitive therapy.
. C) psychoanalysis*.
B) cognitive therapy.
D) systematic desensitization.

21. Which therapeutic approach emphasizes that people are often disturbed because of their
irrational interpretations of events?
A) drug therapy
B) client-centered therapy
C) systematic desensitization
D) cognitive therapy *

13. Attribution theory was designed to account for:


A) the process of revealing intimate aspects of ourselves to others.
B) the impact of both heredity and environment on social behavior.
C) the loss of self-awareness that occurs in group situations.
D) how people explain others' behavior.*

14. An example of the fundamental attribution error is illustrated in our tendency to


underestimate the extent to which others' behavior is influenced by:
A) genetics.
B) social roles.*
C) their political philosophy
D) personality traits.

15. The discomfort we feel when two thoughts are inconsistent is called:
A) cognitive dissonance*.
B) group polarization
C) deindividuation.
D) the fundamental attribution error.

16. According to Milgram, the most fundamental lesson to be learned from his study of
obedience is that:
A) people are naturally predisposed to be hostile and aggressive.
B) even ordinary people, who are not usually hostile, can become agents of destruction.*
C) the desire to be accepted by others is one of the strongest human motives.
D) people value their freedom and react negatively when they feel they are being coerced to do
something.

17. Social facilitation refers to the tendency to:


A) neglect critical thinking because of a strong desire for social harmony within a group.
B) perform well -earned tasks more effectively in the presence of others.*
C) experience an increasing attraction to novel stimuli as they become more familiar.
D) comply with a large request if one has previously complied with a small request.

18. In one experiment, men talked charmingly because they thought the women on the phone
were beautiful. The fact that the women responded warmly best illustrates the impact of:
A) social facilitation.
B) facilitated communication.
C) the foot-in-the-door phenomenon
D) self-fulfilling prophecies*.

19. Prejudice is best defined as:


A) the tendency to favor members of one's own group.
B) a fearful suspicion of people one has never met.
C) an unjustifiable attitude toward a group and its members.*
D) the belief that victims of misfortune deserve their fate.

20. Most children believe their school is better than the other schools in their town. This best
illustrates:
A) the just-world phenomenon..
B) ingroup bias.*
C) the fundamental attribution error
D) the reciprocity norm.

21. When 12-year-old Jamilah saw an old man lying on the sidewalk in apparent discomfort, he
prepared to offer help. But when he noticed several adults walk past the man, he concluded that
the man did not need any help. His reaction most clearly illustrates one of the dynamics involved
in:
A) the fundamental attribution error.
C) the foot-in-the-door phenomenon.
B) social loafing.
D) the bystander effect.*

22. Which of the following best describes Freud’s psychological theory of development?
A. Deficit theory
B. Drive theory*
C. Learning theory
D. Conflict theory
E. Experiential theory

23. Which of the following represents a basic concept of community psychiatry?


a. Social model
b. Fee-for-service
c. Psychodynamic construct
d. Medical model
e. Continuity of care*
24. Social skills training for individuals with persistent mental illness is an essential
component of which of the following?
a. Functional assessment
b. Vocational rehabilitation
c. Behavioral family management
d. Psychiatric rehabilitation*
e. Assertive community treatment.

25. According to Aaron Beck, the primary defect in depression involves which of the
following?
a. Cognitive distortion*
b. Aggression turned inward
c. Learned helplessness
d. Early childhood loss
e. A history of ambivalent relationships

26. Which of the following represents an objective psychological test?


a. Rorschach Test
b. Draw-a-person Test
c. Sentence Completion Test
d. Thematic Apperception Test (TAT)
e. Minnesota Multiphasic Personality Inventory (MMPI)*

27. The best type of study for determining the relationship between a certain risk factor and
development of a disease is a :
a. Cohort study*
b. Cross-sectional study
c. Case-history study
d. Case-control study
e. Crossover study
28. Handwashing rituals reflect which of the following defense mechanisms?
a. Regression
b. Repression
c. Undoing*
d. Reversal
e. Projection

29. In “Mourning and Melancholia, “ Freud postulated that the decline in self-esteem that
was experienced by an individual with depression was the result of anger turned inward
against the self. Freud explained that the anger is turned inward due to:
a. An identification with the lost object*
b. A harsh superego
c. A failure to establish good internal objects
d. High levels of constitutional aggression
e. A loss of reality testing

30. The validity of an assessment instrument refers to the extent to which it:
a. Measures what it intends to measure*
b. Can be used successfully in cross-national studies.
c. Provides consistent results when used at different times
d. Provides consistent results when used by different examiners
e. Produces higher rates of false-negative measures.
31. According to the 1990 Global Burden of Disease Study, the leading cause of
neuropsychiatric disability worldwide is:
a. Panic disorder
b. Alcohol use
c. Schizophrenia
d. Bipolar disorder
F. Unipolar major depression*

32. Which of the following stages of the life cycle as described by Erikson corresponds to
Freud’s anal phase?
a. Basic trust versus basic mistrust
b. Autonomy versus shame and doubt*
c. Initiative versus guilt
d. Industry versus inferiority
e. Identify versus role diffusion

33. Learned helplessness is a model for:


a. Anxiety.
b. Depression*
c. Thought disorder
d. Organic dysfunction
e. Personality disturbance.

34. In contrast to psychodynamic psychotherapy, supportive psychotherapy is


fundamentally designed to be:
a. Inexpensive and delivered by nonmedical providers.
b. Focused on the patient’s mechanisms of coping with day-to-day stresses.*
c. Time-limited with a preestablished duration of weeks to months rather than years.
d. Less stigmatizing and more acceptable to patients, families, and employers.
e. Centered on medical management and psychopharmacology.

35. A male patient enters psychotherapy because of problems in his relationship with his
wife. During the sessions the patient talks about his family of origin, his boss, and his
problems at work, but never discusses details about his wife. This is an example of which
of the following?
a. Sublimation
b. Repression
c. Denial
d. Transference
e. Resistance.*

36. In a study designed to evaluate the efficacy of a new (experimental) form of treatment in
comparison with a standard or placebo (control) treatment, the best way to control for the
influence of known and unknown extraneous variables on treatment outcome is:
a. Assigning to the experimental treatment only those subjects who do not respond
adequately to the control treatment.
b. Conducting the experimental and control treatments in a carefully controlled
environment.
c. Conducting the experimental treatment at a different location from where the
control treatment is conducted.
d. Periodically reassessing treatment outcome during a long follow-up period.
e. Randomized assignment of subjects from a single population to the experimental
treatment or control treatment*.
39. Schemata in cognitive therapy are best described as which of the following?
A. Proposed methods to restructure maladaptive cognitive patterns.
B. Structural units into which cognitive processes can be broken down.
C. Organized, established cognitive structures based on the abstraction of relevant prior
experience*.
D. Negatively distorted thought processes or beliefs that an individual has acquired
through negative life experiences.

40. A child in the latency phase experiences:


A. A significantly less preoccupation with the parental roles*.
B. A need for immediate satisfaction of biological urges.
C. A passionate struggle for autonomy from parents.
D. The introjection of an ego ideal and the subsequent formation of the superego.

41. An important countertransference issue in an interethnic psychotherapeutic relationship


is:
A. Avoidance of transference
B. Denial of differences.*
C. Competitiveness.
D. Grandiosity

42. Integration of clinical and experimental observations in early child development supports
which of the following statements about infant/parent interactions?
A. There is strong empiric support for an initial phase of primary autism.
B. Cross-modal perception is not involved until the second year of life.
C. Infants rarely initiate reciprocal interactions with caretakers.
D. Infants are born with sophisticated perceptual abilities that facilitate
attachment.*
E. The “social smile” is the infant’s first indication of recognizing others.

43. Repeated presentation of conditioned stimulus without being paired with its unconditioned
stimulus will result in which of the following?
A. Stimulus generalization
B. Partial reinforcement
C. Escape learning
D. Discrimination
E. Extinction*

44. Which of the following best illustrates a double bind in a family?


A. Betty’s parents encourage her to attend college, but they advise her to work to earn
money for her expenses.
B. Betty’s parents encourage her to attend college, but they complain that her expenses will
be a hardship for the family.*
C. Betty’s parents encourage her to attend college, but they discourage her younger
brother’s wish to attend college.
D. Betty’s parents encourage her to attend college, but they want her to make the decision by
herself.
E. Betty’s parents insist that she attend college, but Betty would prefer to join the navy.

45. A study that follows a group chosen from a well-defined population over an extended
period of time is called a:
A. Cohort study
B. Case-control study
C. Case-history study
D. Retrospective study
E. Cross-sectional study.

45. Dysprosody is an abnormality of:


A. Mood
B. Affect
C. Speech*
D. Thought process
E. Thought content.

46. .A patient’s scores on the Revised Wechsler Adult Intelligence Scale (WAIS-R) subtests
for picture arrangement and block design are very low compared to scores on other subtests.
These results are most suggestive of:
A. Alcoholism
B. Schizophrenia
C. Poor verbal skills
D. A lesion in the frontotemporal lobes
E. A lesion in the nondominant hemisphere.*

47. A child who fears riding a bike becomes less fearful after watching other children having
fun while doing so. This illustrates which of the following learning principles?
A. Reinforcement
B. Habituation
C. Conditioning
D. Attribution
E. Modeling*

48. Sinusoidal waves at 9 to 11 Hz are recorded in the electroencephalogram (EEG) during


which of the following circumstances?
A. Wakefulness with eyes closed*
B. Wakefulness with eyes open
C. Light sleep
D. Deep sleep
E. Hypnosis.

49.Which of the following terms best represents the combination of several unconscious
impulses, wishes, or feelings that are attached to a single manifest dream image?
A. Secondary revision
B. Day residues
C. Displacement
D. Libido
E. Condensation*

50. A person who smokes a pack of cigarettes per day stops smoking and experiences the
need for a cigarette after every meal. This phenomenon is an example of:
A. Oral dependence
B. Facilitated tolerence
C. Relapse vulnerability
D. A withdrawal symptom
E. An environmental trigger*
51. At which of the following receptors does phencyclidine’s major action occur?
A. Gamma -aminobutyric acid (GABA) type A
B. N - methyl-D-aspartate acid (NMDA)
C. Serotonin type 2 A (5-HT2A)
D. Dopamine type 2(D2)
E. Mu-opioid

52. Which of the following would be a useful screening test to evaluate an 8-year-old child’s
academic performance?
A. Denver Developmental Screening Test (DDST)
B. McCarthy Scales of Children’s Abilities (MSCA)
C. Wide Range Achievement Test (WRAT)*
D. Vineland Adaptive Behavior Scales.
E. Halstead-Reitan Test.

.53. Children can first recognize their mother’s face as distinct from other faces at:
A. 1 month*
B. 3 months
C. 5 months
D. 8 months
E. 12 months

54. Family studies have found that the first-degree relatives of individuals with bipolar
disorder, type II, have elevated incidence rates of which of the following disorders?
A. Schizoaffective
B. Generalized anxiety
C. Major depressive*
D. Histrionic personality
E. Panic

55. Which of the following terms best defines the degree of spread of scores about the mean?
A. Mode
B. Range
C. Median
D. Standard deviation*

56. In learning theory, the presentation of an aversive stimulus that is contingent upon the
occurrence of a particular response in known as:
A. Extinction
B. Punishment*
C. Positive reinforcement
D. Negative reinforcement

57. The hippocampus and the parahippocampal gyrus mediate:

A. Sensory memory (feelings)


B. Procedural memory (skills)
C. Pattern memory (gestalts)
D. Declarative memory (facts).*
E. Auditory memory (sounds)

58 .Which of the following demographics is thought by cross-cultural researchers to best


indicate both a common ancestry and a shared sense of identity, beliefs, and history?
A. Race
B. Ethnicity*
C. Religion
D. Language
E. Socioeconomic status

58. Rare occurrences of priapism are most frequently associated with the use of which of the
following medications?

A. Bupropion
B. Trazodone*
C. Amitriptyline
D. Benztropine
E. Diazepam

59. The pentobarbital challenge test is useful for:


a. Diagnosing atypical cases of porphyria
b. Confirming a diagnosis of amphetamine toxicity.
c. Estimating the starting dose of the drug for detoxification.*
d. Testing genetic vulnerability to alcohol or sedative drug abuse.
e. Predicting response to hypnosis for chronic pain control

60. The adolescent who belittles parents and begins to rebel in order to defend against the
regressive pull back to childhood is using which of the following ego defenses?

A. Ascetism
B. Projection
C. Displacement
D. Intellectualization
E. Reversal of affect.*

61. Which of the following questions is useful for evaluating immediate recall?

A. Where were you yesterday?


B. Where did you go to school?
C. Can you tell me today’s date?
D. What did you have for dinner?
E. Can you repeat these six numbers?*

62. Which of the following developmental disability syndromes is associated with a triple-repeat
genetic abnormality?

A. Rett’s disorder
B. Down syndrome
C. Fragile X syndrome*
D. Asperger’s disorder
E. Williams syndrome.

63. The etiology of benign intracranial hypertension has been linked to:

A. Iron deficiency
B. Hypervitaminosis
C. Hypervitaminosis *.
D. Vitamin E deficiency
E. Vitamin B12 deficiency

64. A patient with sensory impairment over the contralateral face, arm, and leg involving
pinprick, touch, vibration, position, two-point discrimination, and stereognosis most likely
has a lesion in which of the following structures?

a. Anterior frontal lobe


b. Posterior parietal lobe
c. Thalamo-parietal projections*
d. Posterior limb of internal capsule.

65. Which of the following surgical procedures can be used in the treatment of epilepsy
refractory to medications?

A. Electrical stimulation of the vagus nerve*.


B. Electrolytic lesion of the globus pallidus.
C. Deep brain stimulation of the cerebellum.
D. Deep brain stimulation of the subthalamic nucleus

66. Which of the following is the mechanism by which disulfiram is helpful in the treatment of
alcohol dependence?

A. Aversion therapy*.
B. Exposure therapy.
C. Negative reinforcement.
D. Neurochemical effect on craving.

67. Which of the following best characterizes the current explanation for group differences
in prevalence of psychiatric illness?

A. Differential exposure to stress.


B. Variations in family structure.
C. Factors that promote vulnerability to stress*.
D. Rural versus urban environment.

68. The reliability of a particular diagnosis would be considered excellent if the :

A. disorder aggregates in families.


B. disorder correlates with response to treatment.
C. disorder correlates with a known pathogenic mechanism
D. disorder is more common among identical twins than among fraternal twins.
E. diagnosis is agreed upon by clinicians in different centers.*

69. Which of the following is a self-administered diagnostic


instrument?

A. The SCID.
B. The Hamilton D.
C. The CGI.
D. The GHQ.
E. None of the above*
70. Which of the following is NOT a risk factor for non-compliance?

A. youth
B. deficient social supports
C. being elderly*
D. having impaired cognition.
E. substance abuse.

71. Which of the following biogenic amines is synthesized in the locus caeruleus?

A. taurine
B. dopamine
C. acetylcholine
D. norepinephrine *
E. 5-Hydroxytryptamine.

72. Poor copers tend to be

A. flexible in their defences


B. pessimistic*
C. practical
D. insightful about their problems
E. resourceful in finding solutions

74. The side effects of sedation and weight gain are associated with mirtazapine’s
antagonism of which of the following receptors?

A. H1*
B. 5-HT3
C. 5-HT2A
D. ą2
E. 5-HT1A

75. An example of primary prevention is:

A. crisis intervention
B. early identification
C. psychiatric rehabilitation
D. mental health education program*
E. reduction of residual effects of mental illness.

76. According to psychoanalytic theory, feelings of persecution are most reflective of which
of the following defense mechanisms?

A. Projection*
B. regression
C. isolation
D. repression
E. reaction formation
77. Which of the following instruments is most helpful in the assessment of children suspected
of having attention-deficit hyperactivity disorder (ADHD)?

A. Conners Teacher Rating Scale *


B. Wechsler Intelligence Scale for Children -III.
C. Woodcock-Johnson Psycho-educational Battery –Revised
D. Vineland Adaptive Behavior Scales.
E. Rorschach Inkblot Test.

78. The most likely chromosomal abnormality associated with fragile X syndrome is

A. chromosome 7 deletion
B. q27, long arm of X chromosome*
C. trisomy 21
D. chromosome 15 deletion
E. trisomy 15

79. Which of the following terms refers to a difficulty in recognizing and describing one’s
emotions?

A. neologism
B. repression
C. reaction formation
D. alexithymia*
E. abreaction.

80. While intoxicated a psychotomimetic drug, a young man reports” seeing sounds” and
“patterns of colors like fireworks or colored flames” associated with real auditory stimuli.
Which of the following terms best describes this type of sensory experience?

A. Synesthesia*
B. oneiric state
C. bouffee delirante
D. isakower phenomenon
E. lilliputian hallucination

81. Pseudologia fantastica is most commonly associated with which of the following?

A. malingering
B. schizophrenia
C. hypochondriasis
D. factitious disorder *
E. somatoform disorder

82. . Individuals with Down’s syndrome have a high incidence which of the following?
A. Multi-infarct dementia
B. dementia pugilistica
C. Alzheimer’s dementia*
D. NPH
E. all the above.

83.Which of the following laboratory findings is common in patients with bulimia nervosa?
A. hyperproteinemia
B. elevated amylase *
C. increased red blood cells
D. hyperkalemia
E. hyperchloremia.

84. Which of the following is the treatment of choice for obstructive sleep apnea?

A. unvulopalatoplasty
B. temazepam
C. tracheostomy
D. nasal continuous positive airway pressure *
E. dextroamphetamine

85. Which of the following is a projective test?

A. beck Depression Inventory


B. minnesota Multiphasic Personality Inventory-2
C. wechsler Adult Intelligence Scale-Revised
D. thematic Apperception Test *
E. wisconsin Card Sorting Test

86. Activities of daily living in dementia patients include each of the following EXCEPT

A. bathing.
B. dressing.
C. toileting
D. feeding.
E. using the telephone*.

89.Which of the following psychological tests demonstrates the highest reliability?


A. Rorschach Test
B. Sentence Completion Test (SCT)
C. Thematic Apperception Test (TAT)
D. Wechsler (Adult Intelligence Scale (WAIS)*
E. Minnesota Multiphasic Personality Inventory (MMPI)

90. Autonomous functions of the ego include all the following EXCEPT
(A) perception
(B) language
(C) motor development
(D) repression*
(E) intelligence

91. Which of the following statements about dreams is not true?


(A) Dreams have a definite bur disguised meaning
(B) Dreams are considered a normal manifestation of unconscious activity
(C) Dreams represent unconscious wishes
(D) The core meaning of the dream is expressed by its manifest content*
(E) Nocturnal sensory stimuli may be incorporated into the dream

92. The major defense mechanism used in phobia is


(A) projection
(B) identification
(C) displacement*
(D) undoing
(E) reaction formation

93. The term “habit training” was coined by


(A) Adold Meyer*
(B) Carl Gustav Jung
(C) Otto Rank
(D) B.F. Skinner
(E) Joseph Wolpe

94. The self-system concerns Harry Stack Sullivan’s concept of the


(A) unconscious
(B) personality*
(C)libido
(D) defense mechanisms
(E) Oedipus complex

95. Erik Erikson used the term” generativity versus stagnation” to describe the conflict occurring
in
(A) childhood
(B) adolescence
(C) young adulthood
(D) middle adulthood*
(E) late adulthood

96.Consistent and affectionate maternal behavior during infancy provides the child with a
continuing of
(A) trust*
(B) autonomy
(C) intiative
(D) industry
(E) identity

97. Erik Erikson’s stage of generativity versus stagnation is characterized by


(A) interests outside the home
(B) establishing and guiding the next generation
(C) self-absorption
(D) bettering society
(E) all the above*

98.According to Sigmund Freud’s structural theory of the mind, the psychic apparatus is divided
into
(A) id, ego, and superego*
(B) ego, unconscious, and id
(C) superego, ego, and unconscious
(D) unconscious, conscious, and preconscious
(E) none of the above
102.A physician who has been given a diagnosis of terminal pancreatic cancer constantly
discusses the technical aspects of his case with many of the other physicians in the hospital. This
is an example of which one of the following defense mechanisms?
(A) Repression
(B) Displacement
(C) Dissociation
(D) Regression
(E) Intellectualization*

104.Which of the following defense mechanisms is classified as the most mature?


(A) Sublimation*
(B) Repression
(C) Rationalization
(D) Projection
(E) Regression

108.Which of the following statements correctly describes aspects of Freud’s structural theory of
the mind?
(A) The id is conscious
(B) The mind is divided into the preconscious and conscious
(C) The superego is unconscious*
(D) The id is closely associated with reality
(E) The ego is under the domination of primary process thinking

109.Which of the following is a function of the superego?


(A) Maintaining relationships to the outside world
(B) Reality testing
(C) Maintaining object relationships
(D) Controlling secondary process thinking
(E) Controlling id impulses* E

110.All of the following elements of classical conditioning must be learned EXCEPT


(A) the conditioned response
(B) the unconditioned response*
(C) the conditioned stimulus
(D) the acquisition phase
(E) stimulus generalization

111.Which of the following schedules of reinforcement is most resistant to extinction?


(A) Continuous
(B) Fixed ratio
(C) Fixed interval
(D) Variable ratio*
(E) Discontinued

112.A child who likes and looks up to her physician states that she wants to become a doctor
when she grows up. This behavior is an example of
(A) stimulus generalization
(B) modeling*
(C) shaping
(D) positive reinforcement
(E) variable reinforcement

113.All of the following statements about learning are true EXCEPT


(A) learning involves the acquisition of behavior patterns
(B) classical conditioning is a method of learning
(C) operant conditioning is a method of learning
(D) history of the individual is important in all forms of learning*
(E) stimuli may be internal or external to the individual

114.A 2-year-old child is afraid of nurses in white uniforms. When his grandmother comes to
visit him wearing a white jacket, he begins to cry. The best explanation for this phenomenon is
(A) stimulus generalization*
(B) habit
(C) instrumental conditioning
(D) learning by trial and error
(E) an emitted operant

115.Which of the following statements about reinforcement in operant conditioning is true ?


(A) It establishes a connection between a stimulus and a response*
(B) It can only be positive
(C) It decreases the rate at which a behavior occurs
(D) It precedes a behavior
(E) It was first described by the work of Pavlov

116.All of the following statements about biofeedback are true EXCEPT


(A) it is based on classical conditioning*
(B) it has been used in the treatment of generalized anxiety disorder
(C) control over physiologic activity is learned
(D) it can be used to control autonomic activity
(E) relaxation of striated muscle can be achieved

117.All of the following factors are important in the successful use of biofeedback EXCEPT
(A) the patient must receive continuous information about the physical parameter
(B) the physical parameter must be detectable and measurable
(C) the patient’s motivation is important
(D) it is used to gain control over the central nervous system*
(E) a large amount of practice is required

118.Although a mother slaps a child on the hand every time she touches a stove, the child begins
to touch the stove more frequently. This is an example of
(A) punishment
(B) negative reinforcement
(C) positive reinforcement*
(D) aversive conditioning
(E) classical conditioning

119.Which of the following statements concerning narcolepsy is true?


(A) The REM sleep of a narcoleptic person is abnormal*
(B) Narcolepsy occurs in approximately 50 out of every 10,000 persons
(C) Narcolepsy occurs most frequently in the elderly
(D) Sedatives are used to treat narcolepsy
(E) Cataplexy rarely occurs

120.Which neurotransmitter is particularly involved in creasing of REM sleep?


(A) Serotonin
(B) Norepinephrine
(C) Acetylcholine (ACh)*
(D) Dopamine
(E) Histamine

121. Which of the following functions IS NOT a function of the non-dominant cerebral
hemisphere?
A. Holistic
B. Ideational*
C. Pictorial
D. Geometric
E. Non-linear

122. The following structure is not a part of the limbic system?


A. Parahippocampal gyrus
B. Hypothalamus
C. Corpus callosum*
D. Anterior nucleus of thalamus
E. Subcallosal gyrus

123. Diplopia occurs in:


A. Neuropathy of oculomotor nerve*
B. Parkinson’s disease
C. Neuropathy of facial nerve
D. Huntington’s disease
E. Diabetes insipidus

124.Which of the following is not associated with benign intracranial hypertension?


A. Chlortetracycline administration
B. Myxoedema*
C. Polycythaemia
D. Oral contraceptives
E. Hypoparathyroidism

125. Which of the following is not a cause of mononeuritis multiplex?


A. Sarcoidosis
B. Bronchial carcinoma
C. Leprosy
D. Polyarteritis nodosa
E. Trauma*

127.Components of the Papez circuit include:


A. Fornix
B. Mamillary body
C. Hippocampus
D. Thalamus
E. All of the above

129.Which of the following IS NOT a cause of papilloedema?


A. Central retinal vein thrombosis
B. Hypoparathyroidism
C. Cavernous sinus thrombosis
D. Hypocapnia*
E. Cranial arteritis
130. Which of the following structures is NOT located in the pons:
A. Reticular formation
B. Substantia nigra*
C. Locus coeruleus
D. Trigeminal nerve nucleus
E. Vestibular nuclei
1. One trait that dominates a personality so much that it influences nearly everything a person
does is a:
a. Global Trait
b. Cardinal Trait *
c. Specific trait
d. Central Trait
e. Secondary trait

2. The method of science requires that independent observers must:

A. verify data.
B. state a hypothesis in a form that can be tested
C. derive a hypothesis from theory
D. All of the above *
E. None of the above

3. Talkative vs. silent; frank, open vs. secretive; adventurous vs. cautious; sociable vs. reclusive
—these traits describe which dimension of personality?

a. Agreeableness
b. Conscientiousness
c. Extraversion*
d. Culture
e. Emotional Stability

4. Who is the pioneer that proposed the 16 basic dimensions of normal personality and devised a
questionnaire (16PF) to measure them?

A. Carl Jung
B. Raymond Cattell *
C. Julian Rotter
D. Gordon Allport
E. None of the above

5. When analyzing a past president to describe his personality by coding letters or speeches he
has written, you are using a process called

a. Big Five
b. content analysis *
c. personality assumptions
d. Implicit Personality Theory
e. cognitive approach

6. The best known lexical research on terms relevant to personality was conducted by whom and
how many terms were compiled in this study?

A. Sheldon/3
B. Ancient Greeks/4
C. c. Allport/17,953 *
D. Gordon/5
E. Cattell/18,835
7. During psychology you are falling asleep. You know last night you only got three hours of
sleep; therefore you have a valid reason. The person sitting in front of you is sleeping also. You
automatically think that they are lazy and really don’t care about the class. This is an example of

a. Halo effect
b. Implicit Personality Theory
c. Observer bias *
d. stereotype
e. None of the above

8. Positive correlations (from 0.01 to 1.00) indicate that, as one factor goes ________, the other
factor goes __________.

A. down, up
B. up, down
C. up, up*
D. both a and b
E. All of the above

9. Which strategy IMPROVES the reliability of a research design?

a. make the test shorter


b. standardize all circumstances under which the test is administered,
including instructions.
c. delete all items from the test that do not correlate with the other items.
d. b and c only *
e. a, b, and c are correct

10. Which of the statements below are FALSE?

A. Since Personality psychology has competing viewpoints, it is considered paradigm*


B. Science judges the truth of statements based on the empirical method.
C. Personality encompasses diverse qualities of individuals (e.g., biology, social influences,
childhood experiences, etc.)
D. Science demands that hypotheses be discarded if they are not confirmed by observation.
E. According to the method of science, the study of private experience is problematic.

11. Which of the following examples is most likely to have a negative correlation?

a. Number of hours spent studying and test scores


b. Amount of violent television viewed and number of aggressive acts committed
c. How many times the subject smiles per day and their unhappiness level*
d. Weight and height
e. Your test scores and the Russian economy in 1994

12. The Ancient Greeks hypothesized that there were 4 different personality characteristics.
The four characteristics are:

A. irritability, optimism, pessimism, calmness


B. calmness, depression, pessimism, irritability
C. depression, irritability, pessimism, optimism
D. irritability, depression, optimism, calmness *
E. calmness, optimism, pessimism, depression
13. Sheldon proposed three different types of traits. The traits that would best describe a football
player would be:

a. mesomorph *
b. endomorph
c. ectomorph
d. activomorph
e. None of the above

14. example of a correlational strategy of academic research?

A. testing a hypothesis with an experimental group


B. having a group fill out questionnaires *
C. using the scientific method
D. a clinician diagnosing his/her patient
E. using the electric shock method to illustrate classical conditioning

15. If you had a correlation of positive one, the related items would be:

A. completely unrelated
B. moderately correlated
C. very highly correlated
D. identical *
E. lowly correlated

17. If you say that a movie is really good and you like it you would also assume that your best
friend would like it. What implicit personality theory is this?

a. stereotype
b. perceived similarity*
c. Halo effect
d. observer bias
e. All of the above

18. A more neutral term personality type categories or a typical or exemplary instance of a
category is

a. stereotype
b. observation bias
c. openness
d. open-minded
e. prototype *

19. From the Big Five personality dimensions, behaviors such as speaking fluently, displaying
ambition, and exhibiting a high degree of intelligence is

a. Agreeableness
b. Openness
c. Extraversion
d. Conscientiousness*
e. Neuroticism

20. Psychologists seek to understand Ahmed’s personality through his thoughts, beliefs and their
impact on his behavior in certain situations. This is known as the
a. trait approach
b. humanistic approach
c. cognitive approach*
d. psychoanalytic approach
e. All of the above

21. Ahmed’s personality is researched through 3 factors: extraversion, neuroticism and


psychoticism. This personality trait model is called:

a. Eysenck’s Trait Model *


b. Myers-Briggs Type Indicator
c. Smith Model
d. Cattell’s Sixteen factor Model
e. Big Five

22. Which of the following would be a phenomenological approach to the study of personality?

a. emphasize subject’s feelings


b. emphasize subject’s thoughts
c. emphasize the meaning that people ascribe to events, not simply the objective events
themselves
d. attempt to understand an individual in his or her fullness, rather than only selected aspects of
the person relevant to a small number of traits or other dimensions
e. All of the above *

23. What is the trait approach to personality?

a. Assumes that people are motivated by unconscious emotional conflicts


b. Assumes that each individual has stable personality characteristics*
c. Assumes that people’s thoughts and beliefs are central to personality
d. Assumes that people have an innate tendency to become self-actualized
e. Assumes that people are motivated by conflicts originating in childhood

24. William Sheldon suggested that body build was associated with personality traits. Which one
best describes a person who’s sensitive and intellectual?

a. conservative
b. neurotic
c. endomorph
d. mesomorph
e. ectomorph*

25. Eysenck originally developed ________________, a highly researched factor theory of


personality.

a. an interpersonal trait model


b. a sixteen factor model
c. a three factor model *
d. the Big Five
e. the Implicit personality theory

a.
29. Piaget's studies of "conservation" trace the development of...
a. perception
b. internal representation or "knowledge" *
c. object constancy
d. sensory-motor activity
e. reflexes

51. The theory of attachment predicts that attachment will be strongest:

A.      During breast feeding

B.      At the time kindergarten begins

C.      When the infant becomes mobile*

D.      Children are reared without a consistent caregiver

E. None of the above

52 A physician repeatedly administers a drug to a patient that causes nausea. Eventually the
appearance of the physician (even before administration of the drug) produces nausea in the
patient. The physician is a:

A.      Conditioned response

B.      Conditioned stimulus*

C.      Unconditioned stimulus

D.      Unconditioned response

E. All of the above

E.

. 53 What is the most plausible link between high self-efficacy and improved performance in
Bandura’s self-efficacy construct?

A.      Entitlement and self-absorption

B.      Unconscious motivation

C.      Basic trust

D.      Effort and persistence*

E. Reward*

54. In what state is the electroencephalogram indistinguishable from wakefulness?

A.      Coma

B.      REM*

C.      Stage 1-2 (NREM)


D.      Stage 3-4 (NREM)

E. C &D

55. The biological clock is thought to reside in what structure?

A.      Pituitary

B.      Hippocampus

C.      Amygdala

D.      Hypothalamus*

E. None of the above

56. Which of the following is a NOT an essential feature of addictive behavior?

A.      A seeming unwillingness or inability of the individual to control, reduce or eliminate a


behavior

B.      The development of neuroadaptation in an individual to the effects of a known reinforcer*

C.      A significant adverse effect from the behavior on the individual or others

D.      The excessive or inappropriate self-administration of a known reinforcer by an individual

E. A&D

57. Long-term exposure to which of the following has been associated with neuronal cell loss in
the hippocampus?

A.      Corticosteroids*

B.      Epinephrine

C.      Beta endorphin

D.      Nitric oxide

E. None of the above

58. Which of the following complications of chronic heavy alcohol consumption is most
likely to persist beyond the first week of abrupt cessation?

A. Delirium
B. Sleep fragmentation *
C. Tactile hallucinations
D. Autonomic hyperactivity
E. Generalized seizures.

59. While in family therapy, a previously distant couple begins to communicate more
frequently and intimately. At the beginning of therapy, the 13-year-old daughter used to sit close
to her mother, frequently looking to the mother for reassurance and support and saying very
little. Now, however, as the parents have become closer, the daughter has become openly hostile
to and dismissive of the father. The daughter’s change in behavior is best explained by the
concept of:

a. Trangulation. *
b. Double binding.
c. Pseudo-hostility.
d. Enmeshment.
e. Transmuting internalization.

60. A 50-year-old individual with depression believes that he is responsible for the
destruction of the world. This is an example of a:

a. Delusion of control.
b. Delusion of persecution.
c. Mood-congruent delusion. *
d. Delusion of reference,
e. Pseudo-hallucinatory experience.

61. Abulia refers to an impairment in the ability to:

a. State or identify mood.


b. Swallow solid food.
c. Stand or walk without assistance.
d. Spontaneously move and speak. *
e. Make or understand human gestures.

62. The psychiatrist is treating a 34-year-old patient with symptoms of anxiety and a history
of cocaine abuse. The patient has brought suit against her current employer for harassment at
work and has entered her anxiety disorder as evidence of damage. The psychiatrist is obliged to
inform the patient that , if subpoenaed, the psychiatrist:

a. Cannot testify because it is breach of confidentiality.


b. Cannot testify because the psychiatrist is not the court-appointed expert.
c. Will only have to testify about the patient’s anxiety symptoms.
d. May have to testify about all of the patient’s psychiatric problems. *
e. Will no longer be able to provide treatment to the patient.

63. A 62-year-old man with diabetes mellitus is referred to a psychiatrist by his family
because he is not making any sense. When he is asked what is wrong. He answers, “thar szing is
phrumper zu stalking”. This expression has normal intonation, but no one in his family can
understand it. The man verbally responds to other questions with similar utterances, but fails to
successfully execute any instructions. The diagnosis most likely is:

a. Schizophrenic world salad.


b. Conduction aphasia.
c. Wernicke’s aphasia. *
d. Balint’s syndrome.
e. Drug intoxication

64. A delusion is
A. a sensory experience in the absence of external stimuli
B. a misinterpretation of actual perceptions
C. a false belief not in accord with a person intelligence or culture *
D. a manifestation of unacceptable feelings projected outward
E. unconscious blocking of instinctual feelings

65. Eye to eye conduct:

A. Is usually increased in depression


B. Is never a sign of aggression
C. Is an essential part of psychotherapy
D. Is not influenced by cultural factors
E. Is assessed in the mental state examination *

66. . First rank symptoms of schizophrenia:

A. Are always pathognomonic of schizophrenia


B. Include 2nd or 3rd party hallucinations
C. Incorporate all passivity phenomena *
D. Include formal thought disorder
E. Include incongruity of affect

67. The Ganser syndrome occurs in:

A. Schizophrenia
B. Manic-depressive psychosis
C. Prisoners awaiting trial *
D. Personality disorders
E. Hysteria

68. The experience of depersonalization is:

A. Usually pleasant
B. Delusional
C. Recognized as 'odd' *
D. Treatable with phenobarbitone
E. Recognized as 'true'

69. The following associations are correct:

A. Cameron and concrete thinking


B. Schneider and condensation
C. Bleuler and drivelling
D. Goldstein and over-inclusive thinking
E. Bleuler and loosening of associations *

70. The following are correct regarding hallucinations except

A. Can occur in normal states


B. Organic hallucinations occur more frequently during the day. *
C. Visual hallucinations occur in beri-beri
D. Real sleep studies suggest that hallucinatory states are associated with increased sensory
awareness
E. Auditory hallucinations are common in affective disorders.

71. Paranoia involves the process of


A. introjection
B. regression
C. conversion
D. projection *
E. isolation

72. . Which of the following psychotic symptoms indicates that the cause is most likely related to
a medical illness?

A. Auditory hallucinations
B. Impairment in reality testing
C. Tactile hallucinations *
D. Thought disorder
E. Visual hallucinations.

74. Which of the following is the best definition of a delusion:

A. False fixed belief.*


B. Perceptual misrepresentation of a sensory image.
C. Perceptual representation of a sound or an image not actually present.
D. Pathological preoccupation.
E. A persecutory obsessional idea.

75. Psychiatrist : What does the proverb “ People in glass houses should not
throw stones” mean to you? Patient: “The windows will break”
This is an example of:

A. Poverty of thinking.
B. Concrete thinking.*
C. Flight of ideas.
D. Loose associations.
E. Circumstantial thinking.

76. A 26 year old male presents to the emergency department with symptoms
agitation and restlessness. He insists on sitting in the corner of the room with his back to the
wall. Suddenly he begins to stare into the opposite corner and says “No, you can’t make me do
that.” Which of the following symptoms is this patient most likely experiencing?

A. Depersonalization.
B. Hallucinations*.
C. Delusions.
D. Ideas of reference.
E. Illusions.
77. . Two days after a hip replacement operation and while the surgical resident
is examining the 75 year old patient, he notes that the she states that the date is 1954, and that
she is at her son’s house. These impairments illustrate which aspect of the mental status
examination?

A. Concentration.
B. Memory.
C. Thought process.
D. Orientation*.
E. Level of consciousness.

78. A 24 year old lady is suffering from an acute anxiety attack. In the middle of the
attack, she feels as if she is disconnected from the world as if it is unreal or
distant. Which of the following best describe this symptom?

A. Illusion.
B. Depersonalization.
C. Derealization*.
D. Hallucination.
E. Delusion.

79. All of the following are types of formal thought disorder,


EXCEPT:

A. Circumstantiality.
B. Blocking.
C. Looseness
D. Ideas of refrence*
E. Flight of ideas

91. Characteristic features of catatonia include all the following EXCEPT:

A. Automatic obedience.
B. Cataplexy. *
C. Stereotype.
D. Echopraxia.
E. Echolalia.

92. The senseless repetition of a previously requested movement or word is called:

A. Echolalia.
B. Perseveration. *
C. Stereotypy.
D. mannerism.
E. apraxia

93. The capacity to formulate concepts & generalize them is called:

A. Concrete thinking.
B. Abstract thinking. *
C. Delusional thinking.
D. Intellectualization.
E. Rationalization.
94. A variety of phenomena which have in common the apparent disintegration of
boundaries between the self & the surrounding world is called:

A. Eidetic images.
B. Passivity phenomenon.
C. Desultory thinking. *
D. Omission.
E. Reaction formation.

95. The following occur in the elderly except:

A. Increased levels of free drugs


B. Increased half-life of psychotropic drugs.
C. Significant changes in absorption. *
D. Increased concentration of water-soluble compounds.
E. Glomerular filtration reduces by up to half by the age of 70 years

96. Sleep and EEG

A. Alpha spindles occur in stage 1


B. K complexes occur in stage 2,3 and 4
C. REM is characterised by desynchronisation with faster frequencies
D. Stage 4 is characterised by sleep spindles of fast activity.*
E. Frequent vertex sharp waves occur in stage 2

97. The paraventricular nucleus( PVN) all true except

i. Neurons synthesising oxytocin connect the PVN with the Posterior


pituitary
b. The PVN is situated in the thalamus.*
c. TRH is synthesised in the PVN
d. CRF is synthesised in the PVN
e. Neurons within the PVN affect peripheral thermogenesis

98.Stimuli for release of posterior pituitary hormones ( oxytocin and Arginine vasopressin
,AVP ) include all of the following except:

A. Nausea
B. Labour
C. Increased plasma osmolality*
D. Stress
E. Atrial peptides

99. In relation to neuropathyology of Huntington's disease the following are correct except:

A. there is striatal atrophy


B. there is flattening of the outline of the head of the causate
C. Gliosis can be marked
D. There is cortical atrophy
E. There are specific microscopic abnormalities.*
100. Features of Huntington's disease include all except.

A.Autosomal dominant transmission with complete penetrance


A. Initial presentation often psychiatric
B. Subcortical dementia
C. Caudate tail atrophy *
D. Profound weight loss

101. Studies on populations with schizophrenia and presenting with


predominantly negative symptoms indicate:

A. A.a significant higher ventricle : brain ratio


B. an overall deterioration in IQ
C. short-term memory disturbances
D. deficits in higher-order reasoning
E. no perceptual difficulties*

102. Noreadrenaline all true except:

a. noradrenaline neurons originate in the brain-stem


b. noradrenaline does not cross the blood-brain barrier
c. Clonidine is a noradrenaline antagonist *
d. presynaptic alpha2 adrenoceptors are involved in the control of noreadrenaline.
e. a significant loss of noradrenergic neurons is found in Parkinson's disease

103. .Serotonin (5-HT) all are TRUE Except:

A. the supply of L-tryptophan is the rate of limiting step in the synthesis of 5-HT
B. low levels of 5-HT have been shown to relate to violent or impulsive behaviour
C. 5-HT1A receptors are concentrated in choroids plexus
D. LSD is a 5-HT4 agonist
E. Almost all of the 5-HT in the blood is stored in the red Blood cells *

104.None of the following is excitatory excitatory amino acid except:

A. Glycine
B. GABA
C. Glutamic acid *
D. Acetylcholine
E. N-methyl-D-aspartate (NMDA)

106. Components of the limbic system include all of the following Except:

A. Cingulate gyrus
B. Anterior nucleus of the thalamus
C. Septal nuclei
D. Dorsal longitudinal fasciculus*
E. Amygdala

107. The startle response, all false except

Ais dependent on an intact reticular formation*


B. is absent in decerebrate animals
C. is extensor in all four limbs
D. is usually followed by a motionless state
E. is a learned response

108. The following are true statements except


A. the predominance of the crossed pathway can be demonstrated by the ' dichotic
listening ' test
B. unilateral lesion of Heschl's gyrus result in unilateral deafness *
C. words presented to the right ear are more accurately reported than those presented
simultaneously to the left ear
D. focal lesions in the cortex seldom give rise to auditory signs to aid localization
E. the inferior colliculus is the origin of pathways involved in postural reflexes in
response to sudden loud noises.

.109. The visual path ways all true except:


A. Axons serving the temporal visual fields cross the midline in the optic chiasm.
B. the optic tracts terminate in the lateral geniculate body.
C. damage to the optic nerve causes partial or total blindness in the ipsilateral eye.
D. lesions affecting the midline of the optic chiasm are associated with hemianopia affecting
nasal fields. *
E. lesions behind the chiasm result in homonymous defects.

110. the auditory pathway is dependent on the following structures except:


A. Trapezoid body
B. superior olivary nucleus
C. Lateral lemniscus
D. Lateral geniculate body *
E. Heschl's gyrus

111. The blood supply to the brain, all true except:


A. the basilar artery runs along the midline of the pos
B. the medulla is supplied by branches of the posterior inferior cerebellar arteries
C. the external cartroid artery divided to from the middle cerebral and anterior cerebral arteries *
D. the internal capsule is supplied by the circle of Willis
E. the middle cerebral artery supplies Broca's and Wernick's areas

112. Epilepsy and learning disability, all true except:


A. in those with IQ of 50-75 the rate of epilepsy is 20% *
B. treatment of epilepsy may lower IQ
C. Strug-Weber syndrome results in reduced IQ and epilepsy
D. the commonest cause of most epilepsy in people with learning disability is developmental
E. seizures rarely reduce IQ

113. the following are classified as generalised seizures except:


A. absence seizures
B. Myoclonic seizures
C. simple partial seizures
D. antonic seizures
E. psychomotor' attacks *

114. Enzymes involved in the formation of the catecholamins all


except:
A. dopamine beta hydroxylas
B. dopa decarboxylase
C. monoamine oxidase*
D. tyrosine hydroxylase
E. phenylalanine hydroxylase

119. Which synaptic process acts primarily postsynaptically:


A. facilitation
B. adaptation
C. autoreceptor inhibition
D. shunting inhibition
E. depression

124. Lackof prolactin elevation after a seizure suggests that the patient has had which of the
following types of seizure?

A. Pituitary.
B. Supplementary motor.
C. Grand mal.
D. Complex partial.
E. Non-epileptic. *

125.Fetal alcohol syndrome is NOT associated with:

A. Macrocephaly. *
B. Facial dysmorphisms.
C. Postnatal growth retardation.
D. Intrauterine growth retardation.
E. Learning difficulties.

126.Which of the following conditions associated with mental retardation does NOT have an
autosomal - recessive inheritance pattern?

A. Adrenogenital syndrome
B. Down syndrome *
C. Hurler’s syndrome
D. Tay-Sachs disease
E. Phenylketonuria.

127. In which of the following types of research studies is a group studied over of
prolonged time period?

A. Case- control.
B. Cross-sectional
C. Crossover
D. Cohort *
E. Retrospective

128.Individuals over age 40 years with Down syndrome frequently develop which of the
following disorders?
A. Pick’s disease
B.Normal pressure hydrocephalus.
C.Vascular dementia.
D.Alzheimer’s disease. *
E.Diffuse Lewy body disease.

129. Unilateral hearing loss associated with vertigo, unsteadiness with falls and
headaches, mild facial weakness and ipsilateral limb ataxia is most commonly associated with
tumors in which of the following locations?

A. Cerebellopontine angle *
B. Cavernous sinus
C. Superior orbital fissure
D. Apex of the temporal bone
E. Jugular foramen

130. Which of the following individuals described the schizoid personality, coined
the term “ schizophrenia,” and introduced the terms” autism” and “ ambivalence”?

A. Jean Etienne Dominique Esquirol.


B. Benjamin Rush
C. Emil Kraepelin
D. Eugene Bleuler. *
E. Kurt Schneider

131.Which of the following hormones is most commonly secreted by a functional pituitary


adenoma?

A. Prolactin hormone. *
B. Growth hormone.
C. Follicle-stimulating hormone (FSH).
D. Thyroid –stimulating hormone (TSH).
E. Adrenocorticotropic hormone (ACTH).

132. Which of the following is most critical to monitor in a patient receiving


carbamazepine?

A. Complete blood count (CBC) *


B. Creatinine.
C. Fasting glucose.
D. Triglycerides.
E.Thyroid-stimulating hormone (TSH).

133. A 10-year-old child has frequent episodes of brief lapses of


consciousness without any premonitory sensations, lasting for 2 to 10 seconds, followed by
immediate and full resumption of consciousness without any awareness that anything has
happened. These ictal episodes are most likely caused by what type of seizure?

A.Complex partial.
B.Absence. *
C.Parietal lobe.
D.Gelastic
E.Psychogenic.

134.A 32-year-old patient presents with a 1-month history of worsening headaches, episodic
mood swings, and occasional hallucinations with visual, tactile, and auditory content. A
computed tomography (CT) scan of the head is most likely to reveal a tumor in which of the
following areas?

A.Frontal lobe.
B.Temporal lobe. *
C.Parietal lobe.
D.Occipital lobe.
E.Hypothalamus.

135.Which of the following cognitive enhancers is an NMDA receptor antagonist?

A.Donepezil.
B.Galantamine.
C.Memantine. *
D. Rivastigmine.
E.Tacrine.

136.Which of the following procedures would confirm the diagnosis of non-epileptic seizure?

A.Electroencephalography (EEG) between episodes.


B.Hypnotic suggestion.
C.Observation for a 24 hour period
D.Magnetic resonance imaging (MRI) scan.
E.Videotelemetry. *

137.Which of the following diagnostic procedures is part of the basic workup for assessment of
the etiology of unexplained mental retardation in a child without obvious dysmorphology or
neurological findings?

A.Chromosomal analysis. *
B. Electroencephalography (EEG).
C. Fluorescent in situ hybridization.
D. Magnetic resonance imaging (MRI) scan.

138.. the following are true with regard to mode of inheritance Except:
A. Duchenne's mascular dystrophy is X-linked recessive
B. female carries of Lesch-Nyhan are mosaic for the condition due to random inactivation of the
X chromosome
C. Phenylketonuria (PKU) incidence shows variation across ethnic groups
D. Prader-Willi syndrome derives from a lack of maternal phenylalairine*
E. neurofibromatosis is autosomal dominant
139. the somatic phenotype of Prader-Willi syndrome includes all except:
A. mild learning disability
B. frequent spinal deformities
C. hypogonadisim
D. small extremities
E. non-food-related belligerence *

140. Features of the fragile X syndrome, all false except:


A. fragile X I usually associated with a reduction in size of a repeat sequence near the promoter
of a gene called FMR1
B. the responsible gene is on the short arm of the X chromosome at Xp27
C. multiple repeats of a CGG trinucleotide triplet lead to methylation and silencing of the gene
*
D. approximately 80% of affected boys have an IQ less than 50
E. one-third of males with fragile X have epileptic seizures

143. Which of the following medications is contraindicated in patients


taking clozapine?

A. Lithium.
B. Carbamazepine. *
C. Fluoxetine.
D. Nortriptyline.
E. Clonazepam.

145. All of the following non-stimulant medications have empirical support for attention –deficit
hyperactivity disorder (ADHD) EXCEPT:

A. Fluoxetine. *
B. Clonidine.
C. Bupropion
D. Imipramine.
E. Atomoxetine.

146. The most frequent cause of death following a tricyclic antidepressant overdose is:

A. Rhabdomyolysis.
B. Hypotension.
C. Stroke.
D. Arrhythmia. *
E. Hyperpyrexia

147. The difference in the rate and extent to which a brand-name vs. a generic drug becomes
available to the site of action , given the same dose and conditions, is measured as:

A. Bioequivalence. *
B. Bioavailability.
C. Half-life.
D. Time to peak.
E. Biometabolism.

148. Which of the following benzodiazepines is reliably absorbed when given


intramuscularly?
A. Lorazepam. *
B. Chlordiazepoxide.
C. Diazepam.
D. Alprazolam.
E. Clonazepam.

149. A clinically significant increase in the concentration of lamotrigine may occur if


it is coadministered with.

A. Lithium.
B. Carbamazepine.
C. Valproic acid. *
D. Risperidone.
E. Clozapine.

150. Which of the following benzodiazepines has the longest half-life?

A. Alprazolam.
B. Flurazepam. *
C. Lorazepam.
D. Temazepam
E. Triazolam.

151. Lithium exposure during the first trimester of pregnancy increases the risk of
congenital abnormalities in which of the following organs?

A. Heart. *
B. Brain.
C. Lungs.
D. Kidneys.
E. Liver.

152. Which of the following medications decreases benzodiazepine plasma levels?

A. Carbamazepine. *
B. Fluoxetine.
C. Cimetidine.
D. Disulfiram

154. The number of cases of an illness at a particular time is known as the

(A) incidence
(B) point prevalence*
(C ) frequency
(D) risk factor
(E) cohort

155. . The central feature of the experimental method is:


(A) random assignment of subjects*
(B) attention to interrater reliability
(C) the use of statistical methods
(D) total elimination of observer bias
(E) the use of the observational method of study

159. Which of the following is not a clue suggestive of organic mental disorders?

(A) history of drug or alcohol abuse


(B) family history of inherited metabolic disease
(C) fluctuating mental status
(D) focal neurologic signs
(E) psychiatric symptoms before the age of 30*

160. In a young confused adult patient, the finding of diffuse delta waves in an awake
electroencephalographic recording would be most consistent with:

(A) seizure disorder


(B) episodic explosive disorder
(C) chronic schizophrenia
(D) major depression with melancholia
(E) delirium*

161. The half-life of a drug refers to:

(A) how long the drug will last unused


(B) how long it takes to produce the drug
(C) how long the drug will remain at least one-half active
(D) how long it will take to metabolize one-half the drug*
(E) the temperature a drug must be kept at to keep it from losing half its potency

162. In Erikson's theory of psychological tasks, which of the following is NOT a correct task?

(A) Integrity vs. Despair


(B) Industry vs. Inferiority
(C) Intimacy vs. Isolation
(D) Generativity vs. Stagnation
(E) Identity vs. Shame & Doubt*

163. By all indications, the most popular drug of abuse among young people today is:

(A) Stimulants
(B) Marijuana
(C) Alcohol
(D) Tobacco*
(E) Sedative hypnotics

164. The first sign of beginning cerebral disease is often impairment in

(A) remote memory


(B) long-term memory
(C) immediate memory
(D) recent memory*
(E) none of the above

165. From studies of twins reared together and reared apart, which of the following characteristics
has been confirmed as having the highest heritability?

(A) aggressiveness
(B) extroversion
(C) empathy
(D) stress reaction
(E) neuroticism*

166. The complex of severe psychologic disorders known as schizophrenia has a demonstrable
basis that is characterized as

(A) environmentally determined


(B) polygenic*
(C) a chromosomal aberration
(D) a simple recessive trait
(E) an inborn error of metabolism

167. A disorder resulting from a single gene defect that may produce severe mental problems is

(A) manic-depressive psychosis


(B) dyslexia
(C) phenylketonuria*
(D) Porter’s syndrome
(E) Down’s syndrome

168. The prominence of genetic factors in alcoholism is illustrated by all the following findings
EXCEPT that:

(A) close relatives of alcoholics have a fourfold increased risk


(B) an alcoholic’s children who are given up for adoption at birth are at fourfold increased risk
(C) the risk for the identical twin of an alcoholic is much higher than for a fraternal twin
(D) close relatives of alcoholics are significantly more vulnerable for other psychiatric illnesses*
(E) children of an alcoholic become less intoxicated at a given alcohol level than do controls

169. Of the following illnesses or syndromes, which is LEAST related to genetic factors?

(A) Schizophrenia
(B) Panic disorder*
(C) Bipolar disorder
(D) Alcoholism
(E) Antisocial personality

170. Ethology has made major contributions to understanding human behaviour through all the
following concepts EXCEPT

(A) fixed action pattern


(B) critical period
(C) imprinting
(D) ethnic bonds*
(E) sign stimulus
171. The ion that is most important for the release of a neurotransmitter is

(A) sodium
(B) potassium
(C) calcium*
(D) iron
(E) manganese

172. The following associations exist between neuropsychiatric disorders and disturbances in
specific neurotransmitter systems EXCEPT

(A) Parkinson's disease and dopaminergic dysfunction


(B) sleep disturbances and glutamatergic dysfunction*
(C) Alzheimer's disease and cholinergic dysfunction
(D) depression and noradrenergic dysfunction
(E) schizophrenia and dopaminergic dysfunction

173. All of the following statements about glutamate neurotransmission are true EXCEPT

(A) glutamate is involved in the neurotoxic effects of ischemia


(B) glutamate is a major central nervous system (CNS) excitatory neurotransmitter
(C) glutamate is involved in learning and memory
(D) glutamate is involved in seizure activity
(E) glutamate causes long-term potentiation by binding to the kainate receptor*

174. Basic principles to follow when prescribing medications include the following EXCEPT

(A) individualization of treatment


(B) informing the patient of potential side effects
(C) noting potential interactions with other drugs the patient is taking
(D) providing multiple refills for convenience*
(E) considering the likelihood of nonspecific placebo effects

175. In a 3O-year-old man with chronic schizophrenia, treatment is changed from administration
of haloperidol to administration of the atypical antipsychotic drug clozapine. Which of the
following side effects will probably NOT occur with clozapine?

(A) extrapyramidal symptoms*


(B) constipation
(C) weight gain
(D) sedation
(E) agranulocytosis

176. The following psychotropic medications require regular blood monitoring EXCEPT

(A) clozapine
(B) lorazepam*
(C) lithium
(D) valproate
(E) carbamazepine

177. Displacement behavior is an example of


(A) conflict behavior*
(B) schedule-induced behavior
(C) conditioned neuroses
(D) classical conditioning
(E) primitive learning

178. Which of the following is often found in the families of patients with SDAT?
A. Trisomy 19
B. Trisomy 23
C. Trisomy 21
D. Huntington’s disease*

179. Which of the following is the least likely finding in the history and physical examination
of a patient with anorexia nervosa?
a.)    Lanugo hair
b.)    Amenorrhea
c.)    Hypothermia
d.)    Bradycardia
e.)    Tachycardia*

180. Which of the following medications should be avoided in depressed or anxious patients
with bulimia?
a.)    Fluoxetine
b.)    Bupropion*
c.)    Buspirone
d.)    Sertraline

181. Cross cultural studies have shown greatest agreement on prevalence of:
* A psychoses
B. psychophysiologic disorders
C . phobic neuroses
D . personality disorder s
E. sexual deviations

182. The following are normal fatures of t he aging process except :

A . increasing preoccupation with one's past


B. increasing concerns about one's health
* C . increasing hours spent in sleep*
D . increasing thought s about death and dying

183. Which of t he following is true of psychopharmacology of The elderly?

* A . Elderly frequently respond to much lower doses of medication than younger pts.
B. The elderly are not at great err risk for ex trapperamidal side effects.
D . The therapeutic effcts of antipsychotic medications typically are Evident within four weeks
of commencing taking the medic at ion.
D . The elderly are no more susceptible to orthostatic side effect

184. Which of the following is NOT true regarding serotonin?


A.      Monoamine oxidase is the principle metabolic enzyme for serotonin
B.       The major metabolite of serotonin is 5 hydroxyindolacetic acid (5HIAA)
C.       Low 5HIAA levels are found in the CSF of aggressive patients.
D.      The 'mouse that roared' is a strain of mice that lack 5HT1b receptors and show heightened
aggression toward intruders.
E.       The rate limiting step for serotonin metabolism is tyrosine hydroxylase*

185. Which of the following is not part of the negative cognitive triad?
A.      negative perception of self
B.       negative perception of the future
C.       negative perception of the world
D.      negative perception of your wife*

186. Which of the following is considered the marker enzyme for the nucleus basalis of Meynert
(substantia innominata)?
A.      Catechol- O - methyltransferase
B.       Dopamine beta hydroxylase
C.       Choline Acetyl transferase*
D.      Glutamic acid decarboxylase
E.       Tyrosine hydroxylase

187. Which of the following is not true regarding dopamine and its pathways?
A.      Nigrostriatal pathway runs from the substantia nigra to the caudate and putamen
B.       The mesolimbic system and the mesocortical system project from the ventral tegmental
area
C.       The mesolimbic system projects to the amygdala, hippocampus, nucleus accumbens, and
the septal area
D.      The mesocortical system projects to the frontal cortex
E.       Stimulation of the tuberoinfundibular system stimulates the release of prolactin*

188.  Which of the following is not true regarding norepinephrine (NE)?


A.      The marker enzyme for NE producing neurons is dopamine beta hydroxylase
B.       Norepinephrine is primarily metabolized to MHPG in the periphery*
C.       Norepinephrine is primarily metabolized to MHPG in the central nervous system
D.      The locus ceruleus and the lateral tegmental neurons are the primary norepinephrine
containing neurons in the CNS.
E.       Locus ceruleus neurons are sometimes considered the REM Off cell in sleep cycle

189. Which of the following is least correct regarding the 5 stages of death and dying as
described by Kubler-Ross?
A.      1st stage is usually Shock and Denial- pts describe numbness and sense of unreality
B.      The second stage is usually Anger at self, God, the doctor.
C.      The third stage is usually Bargaining
D.      4th stage is usually Clinical Depression- pts in this stage usually require antidepressant *
E.       Acceptance- Patient becomes comfortable talking about death realistically. Religious
outlook often helps. A chaplain might help for those who have no regular minister, pastor. 

190. Which of the following is most correct regarding normal development in old age?
A.      The usual struggle is between generativity and stagnation
B.       The usual struggle is between intimacy and isolation
C.       The usual struggle is between integrity and despair*
D.      The usual finding is loneliness
E.       80% of the elderly view their health as a problem
191. Which of the following is the best test of frontal lobe functioning?
A.      Wisconsin Card Sort*
B.       Minnesota multiphasic personality Inventory
C.       Rorschach test
D.      Thematic Apperception test
E.       Wechsler Adult Intelligence Test-Revised

192. Which of the following is considered a projective psychological test?


A.      Minnesota Multiphasic Personality Inventory
B.       Millon clinical multiaxial inventory
C.       Halsted Reitan battery
D.      Thematic Apperception Test*
E.       Bender-Gestault test

193. In a patient with bulimia who is actively purging which of the following is the least likely
laboratory value?
A.      hyperkalemia
B.       hypochloremia
C.       alkalosis
D.      elevated serum bicarbonate
E.       hypokalemia

197. Weaknesses of cohort study include all the following EXCEPT


A. unsuitable for rare diseases
B. expensive
C. long delay before availability of results
D. losses to follow-up can affect validity
E. can estimate incidence rate*

198. The normal distribution, or normal curvet has the following properties EXCEPT:
F. it is bimodal*
G. it is continuous
H. it is symmetrical about its mean
I. the mean, median, and mode are all equal
J. the area under the curve is one

199. The following are true regarding non-parametric methods EXCEPT:


F. do require precise values*
G. less powerful
H. less dependent on sample size
I. sample size should be less than 50
J. increase the chance of type II error
II-1. The most frequent indication for psychological testing in clinical psychiatry is to
(A) determine the correct dosage of medication
(B) determine the most effective psychotherapeutic style
(C) assist when there is uncertainty about the diagnosis
(D) assist in determining the length of treatment
(E) assist in generating clinical impressions

II-2.Evaluation of thyroid function may be particularly helpful in the diagnosis and treatment of
which of the following conditions?
(A) Phobic disorder
(B) Schizotypal personality disorder
(C) Major depression
(D) Schizophrenia
(E) None of the above

II-3. Brian-imaging techniques, such as computed tomography (CT) would be most useful in
evaluating
(A) bipolar disorder
(B) schizophrenia
(C) panic disorder
(D) Alzheimer dementia
(E) Sleep apnea

II-4.The intelligence quotient (IQ) is best described as a measure of


(A) innate cognitive endowment
(B) future cognitive potential
(C) environmentally determined cognitive skill
(D) present functional cognitive ability
(E) learned verbal skills

II-5.Psychiatric rating scales that have been developed to evaluate symptoms and
psychopathology include all the following EXCEPT
(A) Hamilton rating scale for depression (Hamilton )
(B) Mental status examination record (MSER)
(C) Present state examination ( PSE )
(D) Brief psychiatric rating scale ( BPRS)
(E) Social adjustment scale (SAS)

II-6.The developmental theories of Margaret Mahler are associated with


(A) genetic field theory
(B) psychosexual maturation
(C) cognitive development
(D) ethologic development
(E) separation-individuation

II-7.Modern psychoanalytic theory holds that narcissism is


(A) a pathologic state
(B) a normal part of human personality development
(C) a normal part of human personality development
(D) the most frequent cause of hypersexuality
(D) first manifested during the oedipal period
(E) of little consequence except in children

II-8.The theories of Carl Jung include all the following concepts EXCEPT
(A) animus and anima
(B) libido as sexual energy
(C) collective unconscious
(D) archetypes
(E) the “shadow’

II-9.Which of the following theorists primarily focused on the maturation of the sense of self
from the infantile fragility and fragmentation into the cohesive and stable structure of adulthood?
(A) Piaget
(B) Erikson
(C) Freud
(D) Klein
(E) Kohut

II-10.According to the developmental theories of Piaget, the concrete operational stage, during
which the child becomes less literal and therefore able to generalize, occurs between ages
(A) 1 and 2
(B) 2 and 3
(C) 4 and 7
(D) 7 and 14
(E) 15and 17

II-11.All the following statements about the psychoanalytic concept of the oedipal phase of
development are true EXCEPT that
(A) it occurs between the phallic and the latency stages
(B) it is usually followed by identification with the parent of the same sex
(C) it occurs only in the development of children destined to become neurotic
(D) it is associated with the phenomenon of castration anxiety
(E) it occurs in both males and females

II-12.All the following statements about rapid eye movement (REM) sleep are true EXCEPT
(A) REM sleep is associated with hypotonia
(B) The amount of REM sleep declines between adolescence and old age
(C) REM sleep is the only state in which dreams occur
(D) A person is more apt to awaken after REM than non-REM(NREM) sleep
(E) Penile erections commonly occur during REM sleep
II-13.Sleepwalking is correctly characterized by all the following statements EXCEPT
(A) it occurs most frequently late in the sleep cycle
(B) it often disappears as the person reaches adolescence or adulthood
(C) it occurs during the same period of the sleep cycle as sleep terrors
(D) it is associated with difficulty in awakening the sleepwalker
(E) It is associated with full amnesia for the event
II-14.Identuty diffusion, as described by Erik Erikson, occurs primarily during
(A) infancy
(B) childhood
(C) adolescence
(D) adulthood
(E) old age
II-15.In psychoanalytic theory the superego
(A) is totally unconscious
(B) is a defense mechanism
(C) functions to reduce guilt and shame
(D) contains the sexual and aggressive drives
(E) contains the ego-ideal
II-16.The early studies of Réné Spitz suggested that
(A) disturbed mothers are instrumental in causing behavioral problems in early infancy
(B) infants reared with little maternal contact are more susceptible to infections and behavioral
problems
(C) infants reared in an institutional setting are likely to become autistic
(D) the number of toys available to infants is a crucial factor in their development
(E) environmental variables have little impact on the health of infants
II-17.Psychoanalytic theory describes the ego as a coherent system of functions, including all the
following EXCEPT
(A) primary process thinking
(B) regulation of instinctual drives
(C) formation of relationships
(D) adaptation to reality
(E) speech
II-18.All the following statements concerning children’s IQ scores are true EXCEPT that
(A) the scores can vary widely if an individual child is tested more than once
(B) the scores can increase over time in children who are highly motivated
(C) the scores correlate fairly well with achievement in school
(D) the scores are determined predominantly by heredity
(E) the mean of the scores remains fairly constant within a given group
II-19. Piaget is best known for his theories and investigations of
(A) cognitive development
(B) affective component of development
(C) mood-related development
(D) motor development
(E) kinesthetic development
II-20.True statements about the latency stage of development include all the following EXCEPT
(A) it begins following the resolution of the Oedipus complex
(B) it is the phase in which identity crisis usually occurs
(C) it consolidates identification with the parent of the same sex
(D) it is often associated with some sexual awareness or activity
(E) it involves wider peer contact
II-21.The highest density of cholinergic innervation of any brain structure is found in the
(A) cerebral cortex
(B) caudate nucleus and putamen
(C) cerebellum
(D) spinal cord
(E) locus ceruleus
II-22.In the brain, transmission along the neuron and across synapses is accomplished by which
of the following mechanisms ?
(A) Mechanical
(B) Neurohormonal and mechanical
(C) Chemical
(D) Chemical and electrical
(E) Electromagnetic
II-23.The state of cataplexy
(A) may be precipitated by an orgasm
(B) is associated with unconsciousness
(C) involves a sudden increase in general muscle tone
(D) often lasts for 1 to 2 h
(E) is usually treated with neuroleptics
II-24.Hypothalamic function is closely related to all the following EXCEPT
(A) sleep
(B) appetite
(C) memory
(D) sexual behavior
(E) fear
II-25.Biologic rhythms – cyclic, internally regulated bodily responses – are operative in all the
following EXCEPT
(A) birth rate
(B) death
(C) body temperature
(D) personality
(E) depression
II-26.True statements about beta-endorphins include all the following ECXEPT
(A) they are involved in the perception of pain
(B) they are releases from the pituitary in response to stress
(C) they are classified as neurotransmitters
(D) they are highly localized to the cerebral cortex
(E) chemically they are peptides
II-27.All the following evidence supports the dopamine hypothesis of schizophrenia EXCEPT
(A) the largest concentrations of dopamine are found in the cerebral cortex
(B) the basal ganglia may be metabolically hyperactive in unmedicated schizophrenia
(C) the phenothiazine drugs block dopamine receptors
(D) many of the antipsychotic drugs increase the level of dopamine metabolites
(E) parkinsonism is a side effect of many antipsychotic medications
II-28.Most studies suggest that the major inhibitory neurotransmitter in the brain is
(A) serotonin
(B) dopamine
(C) beta-endorphin
(D) y-aminobutyric acid
(E) somatostatin
II-29.The Klüver-Bucy syndrome is characterized by
(A) compulsive anal activity
(B) rage attacks
(C) hypersexuality
(D) hypophagia
(E) catalepsy
II-30.The right (nondominant) cerebral hemisphere is thought to mediate or control all the
following functions EXCEPT
(A) visuospatial organization
(B) logical reasoning
(C) perception of body image
(D) perception of rhythm
(E) perception of part-whole relationships
II-31.Typical behavior in patients displaying the catastrophic reaction, as defined by Goldstein
(1939) and Gainotti (1972), includes all the following EXCEPT
(A)restlessness and hypermotility
(B)ingratiating behavior toward the examiner
(C)sudden bursts of tears
(D)cursing
(E)refusal to continue the examination
II-32. The dietary amino-acid precursor of catecholamines is
(A)Tryptophan
(B)glutamic
(C)aspartic acid
(D)tyrosine
(E)glycine
II-33.The orbitofrontal syndrome, associated with injury or tumor of the frontal lobes, is
characterized by all the following ECXEPT
(A) apathy
(B) irritability
(C) jocular affect and euphoria
(D) impulsive behavior
(E) emotional lability
II-34.While psychosis plus delirium can be found in a wide variety of severe endocrinopathies,
psychosis with a clear sensorium is most likely to be found in
(A) hypothyroidism
(B) hyperthyroidism
(C) hypoparathyroidism
(D) hyperparathyroidism
(E) hypercortisolism
II-35.The cell bodies of serotonin-releasing neurons are located in an area of the brain known as
the
(A) raphe nuclei
(B) locus ceruleus
(C) cingulate cortex
(D) basal forebrain
(E) frontal cortex
II-36.For a substance to be classified as a neurotransmitter, all the following must be true
EXCEPT
(A) the substance must be concentrated in the presynaptic nerve terminal
(B) the substance must be released by a depolarizing stimulus applied to the neuron
(C) the effects on the postsynaptic receptor are the same whether the substance is released from
the presynaptic neuron or applied exogenously
(D) the neuron must be able to synthesize the substance
(E) there must be no mechanism for inactivation of the substance after its release from the
presynaptic nerve terminal
II-37.Which of the following types of studies offers the most promise in elucidating the
interaction between genetic and environmental factors in psychiatric illness ?
(A) Family risk studies
(B) Twin studies
(C) Adoption studies
(D) Genetic marker studies
(E) Prospective longitudinal studies
II-38.Information is transmitted along a neuron in a series of electrochemical events know as
(A) translators
(B) resting potentials
(C) action potentials
(D) polarity maintainers
(E) none of the above
II-39.The group of neurotransmitters known as biogenic amines include all the following
EXCEPT
(A) y-aminobutyric acid (GABA)
(B) serotonin
(C) dopamine
(D) acetylcholine
(E) epinephrine
II-40.All the following endocrine glands are subject to control by the brain EXCEPT the
(A) pancreatic islets
(B) pituitary
(C) Fparathyroid
(D) thyroid
(E) adrenal
II-41.In the treatment of psychophysiologic disorders, all the following statements about minor
tranquilizers are true EXCEPT
(A) they fail to result in long-term improvement
(B) they reduce interpersonal dilemmas
(C) they produce drowsiness
(D) they result in need for increased dosages
(E) they reduce high levels of stress
II-42.All the following hormones are recognized as being important in fluencing sexual behavior
EXCEPT
(A) luteinizing hormone releasing factor
(B) oxytocin
(C) estradiol
(D) testosterone
(E) androstenedione
II-43.All the following are common forms of psychophysiologic disorders EXCEPT
(A) bronchial asthma
(B) dysmenorrhea
(C) headache
(D) diabetes
(E) neurodermatitis
II-44.When an axon is cut, all the following events take place EXCEPT
(A) there is a rapid local degeneration of the axon and myelin sheath
(B) macrophages from the general circulation are unable to enter the area to phagocytose axonal
debris
(C) proliferation of fibrous astrocytes forms a glial scar around the zone of trauma
(D) scarring can block the course taken by the regenerating axons
(E) degeneration spreads in both directions along the axon from the zone of trauma
II-45.Clinical features of major depressive disorders suggest a defect in the
(A) frontal lobes
(B) pituitary
(C) hippocampus
(D) hypothalamus
(E) corpus callosum
II-46.Immunocompetence can be suppressed by all the following EXCEPT
(A) the endocrine system
(B) chronic stress
(C) the central nervous system
(D) the autonomic nervous system
(E) psychosocial factors
II-47.All the following statements on the influence of environment on brain structure and
behavior are true EXCEPT
(A) the structure of the brain is to an important degree specified by genetic and developmental
processes
(B) the pattern of interconnections between neurons is influenced considerably by experience
(C) there is very little evidence for the notion of the critical period in the development of normal
social and perceptual competence
(D) at certain stages in its development, the integrative action of the brain and its cellular
structure are dependent upon the brain’s interaction with its environment
(E) the action of the environment on the brain varies with age
II-48. The pituitary secretion of endorphins is closed linked to the secretion of
adrenocorticotropic hormone (ATCH) so that endorphins facilitate the ability to respond to
(A) retarted growth
(B) severe hypertension
(C) stress
(D) chronic pain
(E) tachycardia
II-49.Antibody titers to latent viruses, e.g., Epstein-Barr or herpes simplex, can be increased at
least initially by all the following EXCEPT
(A) relaxation training
(B) immunosuppressive chemotherapy
(C) the stress of academic examination
(D) loneliness
(E) sustained noise
II-50.Psychosocial stress increases the secretion of catecholamines and thus induces an increase
in a number of cardiovascular pathogenic phenomena. All the following are increased EXCEPT
(A) damage to intima of coronary arteries
(B) blood pressure and heart rate
(C) blood lipids
(D) first-degree AV block
(E) ventricular arrthythmias
II-51.Hyperventilation from anxiety, threat, or fear results in all the following EXCEPT
(A) reduced CO2 in the blood
(B) reduced acid level in the blood
(C) increased sympathetic activity
(D) increased risk of cardiac arrhythmia
(E) increased oxygen supplied to brain tissue
II-52.Studies of the effects of acute and chronic stress in humans indicate that the activity of all
the following components of the immune system are decreased EXCEPT
(A) polymorphonuclear granulocytes
(B) lymphocyte T-cell cytotoxicity
(C) production of interferon
(D) latent virus antibody titers
(E) activity of natural killer cells
II-53.Dream deprivation in humans tends to produce
(A) impaired performance on simple verbal tasks with no emotional content
(B) less total need for dream sleep
(C) major impairment of psychological functions
(D) retardation of formation of memory
(E) integration of emotional material with memories of other experiences
II-54.Experimental preavoidance stress-conditioning procedures can induce significant increases
in all the following EXCEPT
(A) plasma potassium
(B) plasma sodium
(C) plasma aldosterone
(D) blood pressure
(E) heart rate
II-55.The diathesis-stress model of psychophysiologic disorders postulates the presence of which
of the following major factor ?
(A) Inadequate coping style
(B) Individual response stereotype
(C) Lack of health belief resolution
(D) Adequate homeostatic restraints
(E) Unconditioned stimulus
II-56.harry Harlow’s work with inanimate surrogate mothers for monkeys suggests that the early
experience critical to the ultimate development of “love “ is
(A) positive reinforcement
(B) protection from danger
(C) contact comfort
(D) need-reduction by nursing
(E) sexual stimulation
II-57.The hormone with the greatest role in aggression is
(A) thyroxine
(B) testosterone
(C) estrogen
(D) progesterone
(E) aldosterone
II-58.A direct pathologic consequence of stress-induced elevation of plasma cortisone is injury to
the
(A) neuroendocrine system
(B) immune system
(C) cardiovascular musculature
(D) hypothalamus
(E) central nervous system
II-59.Which of the following substances is the most responsive to psychosocial stress ?
(A) Catecholamines
(B) Growth hormone
(C) Endorphins
(D) Adrenocorticotropic hormone (ACTH)
(E) Insulin
II-60.The gate control theory of pain assumes all the following EXCEPT
(A) the substantia gelatinosa is the primary vehicle for gating
(B) the spinal gate mechanism is influenced by nerve impulses that descend from the brain
(C) the activity in the large nerve fibers will tend to facilitate the transmission by opening the
gate
(D) motivation, emotion, and cognition modulate the pain experience
(E) the spinal gate mechanism in the dorsal horn modulates the transmission from afferent fibers
to spinal cord transmission cells
II-61.Research on rapid eye movement (REM) sleep patterns shows all the following findings
EXCEPT that
(A) a newborn baby spends more than 50 percent of sleep time in REM sleep
(B) under normal circumstances, 80 percent of the adult sleep cycle consists of REM sleep
(C) deep sleep begins to be replaced with longer periods of REM sleep after the age of 30
(D) the amount of REM sleep determines the amount of actual rest
(E) when REM sleep is interrupted consistently, tiredness and neurotic tendencies develop
II-62.The most common organic explanation for a sleep disturbance in a healthy person is
(A) disruption of normal circadian rhythms
(B) accumulation of hepatic enzymes
(C) the inevitable consequences of high activity
(D) suppressed REM sleep
(E) misuse of hypnotics
II-63.A lesion of the axons of motor neurons that innervate skeletal muscle (lower motor
neurons) will result in all the following consequences EXCEPT
(A) paralysis of individual muscles on the side of the lesion
(B) a paradoxical increase in reflex activity
(C) reduction in muscle mass (atrophy)
(D) decrease in muscle tone
(E) sealing off of the axoplasm
II-64. All the following statements about sleeping pills are correct EXCEPT
(A) they lose their effectiveness in about 2 weeks
(B) barbiturates gradually lead to an increase in hepatic enzymes
(C) patients develop tolerance for benzodiazepines much less rapidly than for pentobarbital
(D) barbiturates enhance REM sleep
(E) there is often a broad cross-tolerance to other hypnotics
II-65.All the following response patterns will usually accompany dreaming EXCEPT
(A) electroencephalographic desynchrony
(B) more visual imagery during non -REM sleep
(C) rapid eye movements
(D) loss of sternocleidomastoid tonus
(E) cardiorespiratory irregularities
II-67.The relationship between social and biologic processes in the generation of behavior has
historically been classified by which of the following terms ?
(A) Classically conditioned
(B) Organic and functional
(C) Genetic and familial
(D) Neuropathologic and sociopathologic
(E) Psychoanalytic and dynamic
II-68.When one has no sense of control over a stressful situation, all the following physiologic
reaction will occur EXCEPT
(A) weaker corticosteroid elevations will develop
(B) an endogenous opioid peptide will be released
(C) a brief endogenous opiate analgesia will be produced
(D) greater suppression of the cyto-toxicity of natural killer cells will occur than in controllable
stress
(E) a weaker immune response to mitogen stimulation will develop than in controllable stress
II-69.Dream deprivation is most apt to produce
(A) a rebound phenomenon of increased dreaming
(B) an increase in anxiety and irritability
(C) acceleration of memory formation of emotionally toned words
(D) a decrement in intellectual function
(E) a temporary increase in night-mares
II-70.Siblings of schizophrenics are more likely to become schizophrenic than more persons
chosen randomly from the population by a factor of about
(A) 2
(B) 4
(C) 6
(D) 8
(E) 10
II-71.The prominence of genetic factors in alcoholism is illustrated by all the following findings
EXCEPT that
(A) close relatives of alcoholics have a fourfold increased risk
(B) an alcoholic’s children who are given up for adoption at birth are at fourfold increased risk
(C) the risk for the identical twin of an alcoholic is much higher than for a fraternal twin
(D) close relatives of alcoholics are significantly more vulnerable for other psychiatric illnesses
(E) children of an alcoholic become less intoxicated at a given alcohol level than do controls
II-72.The most important ( frequent ) genetic cause of mental retardation is
(A) Bartholin-Patau syndrome
(B) Edwards’ syndrome
(C) Down’s syndrome
(D) Turner’s syndrome
(E) Klinefelter’s syndrome
II-73.Genetic factors aremost likely to predispose a person to alcoholism by increasing any of the
following EXCEPT
(A) positive reinforcement of alcohol consumption
(B) functional tolerance of alcohol
(C) level of alcohol dependence
(D) rate of alcohol metabolism
(E) level of acetylaldehyde
II-74.A person with Klinefelter’s syndrome has the genotype
(A) XY
(B) YY
(C) XXY
(D) XYY
(E) XXX
II-75.Factors that influence a person’s gender identity include all the following EXCEPT
(A) chromosomal configuration (XX or XY )
(B) gender assigned at birth
(C) internal reproductive structures
(D) role model of parent
(E) response of parent to child’s assigned gender
II-76.Heredity accounts for approximately what percentage of total variation in IQ scores within
a family ?
(A) 5 percent
(B) 25 percent
(C) 50 percent
(D) 75 percent
(E) 100 percent
II-77.Ethology is characterized by all the following concepts EXCEPT
(A) behavior is best studied under controlled conditions
(B) behavior is best studied comparatively
(C) behavior is the result of interaction between genetic endowment and environment
(D) innate behavior is triggered by a sign stimulus or releaser
(E) previous learning is unnecessary for the successful expression of a fixed action pattern
II-78.The complex of severe psychologic disorders known as schizophrenia has a demonstrable
basis that is characterized as
(A) environmentally determined
(B) polygenic
(C) a chromosomal aberration
(D) a simple recessive trait
(E) an inborn error of metabolism
II-79.Ethology has made major contributions to understanding human behavior through all the
following concepts EXCEPT
(A) fixed action pattern
(B) critical period
(C) imprinting
(D) ethnic bonds
(E) sign stimulus
II-80.A disorder resulting from a single gene defect that may produce severe mental problems is
(A) manic-depressive psychosis
(B) dyslexia
(C) phenylketonuria
(D) Porter’s syndrome
(E) Down’s syndrome
II-81.Emotional expressions involving a stereotyped sequence of fixed action patterns include all
the following EXCEPT
(A) smiling
(B)brow flash response
(C)startle response
(D)Apprehension
(E)joy
II-82.Which of the following is LEAST likely to be considered an ontogenetic stage of synaptic
development and modification?
(A)Synapse formation under genetic and developmental control
(B)Maintenance of newly developed synapses occurring during critical periods
(C)Regulation of transient and long-term effectiveness of synapses
(D)Integration of cellular structure for human mentation
(E)Alteration of preexisting pathways and development of new patterns
II-83.Human behavioral patterns that have the potential of containing major innate (i.e., genetic,
or not learned ) components include all the following EXCEPT
(A) intelligence
(B) brow flash response
(C)Facial expressions of anger, fear, disgust, and joy
(D) smiling
(E) handshake greeting
II-84.True statements about personality traits and aging include all the following EXCEPT
(A) a person’s personality traits after age 30 tend to be less variable
(B) the personalities of adolescents show lower levels of stability than those of adults
(C) most people redefine and make alterations in their personalities during the period of midlife
crisis
(D) the neuroticism trait in personality has a high correlation with lower levels of stability
(E) personality traits are relatively stable over one’s life span
II-85.Although there is no uniform asthmatic personality type, the most frequent psychological
characteristic of boys with bronchial asthma is
(A) hostility
(B) general anxiety
(C) frustrated oral needs
(D) dependency
(E) latent homosexuality
II-86.Freud maintained that neuroses were primarily a result of
(A) overly severe toilet training
(B) inappropriate identification
(C) primary processes
(D) inadequate superego development
(E) sexual disturbances
II-87.In traditional psychoanalysis, transference is the process where in
(A) psychic energy, or libido, is transferred from the id to the ego and superego
(B) a patient invests the analyst with attitudes and feelings derived from vital earlier associations
(C) certain psychological symptoms seemingly defer to new symptoms that frequently are more
accessible to analysis
(D) early object choices are gradually decathected
(E) latent dream content is transformed into manifest content
II-88.In Carl Roger’s personality theory, the essential components of a psychotherapeutic
process involve all the following EXCEPT
(A) a therapist’s empathy
(B) a therapist’s unconditional positive regard
(C) interpretation of unconscious wishes and feelings
(D) reflection of a client’s feelings
(E) clarification of conflictive feelings and attitudes
II-89.All the following are examples of projective tests EXCEPT
(A) thematic apperception test (TAT)
(B) draw-a-person test
(C) sentence completion test
(D) Rorschach test
(E) Minnesota multiphasic personality inventory (MMPI)
II-90.All the following statements about schizophrenia are true EXCEPT that
(A) twin, family, and adoption studies have demonstrated that schizophrenia has a significant
genetic basis
(B) higher socioeconomic status correlates positively with the incidence of schizophrenia
(C) auditory hallucinations are frequently present
(D) chlorpromazine is an effective form of treatment
(E) communication conflicts in families of schizophrenics play an etiologic role
II-91.Anterograde amnesia is associated with which of the following disorders?
(A) Mild retardation
(B) Hypochondriasis
(C) Sociopathy
(D) Korsakoff’s Psychosis
(E) Manic-depressive psychosis
II-92.All the following statements about paranoid disorders are true EXCEPT that
(A) denial and projection are the most common defense mechanisms
(B) affected patients characteristically experience high levels of anxiety
(C) hallucinations are frequent, although thinking is rarely delusional
(D) repression of homosexual wishes may be a frequent psychodynamic determinant of the
disorder
(E) the prognosis for patients who have classic paranoia is poor
II-93.ccording to Freud, the superego contains the
(A) conscience
(B) pleasure principal
(C) ego instincts
(D) reality principal
(E) internalized ego
II-94.All the following are tenets of psychoanalytic theory EXCEPT that
(A) all mental events and attitudes have unconscious antecedent causes
(B) a significant portion of the contents of the mind is unconscious
(C) normal and pathologic mental functioning are qualitatively different
(D) people strives to maximize pleasure and minimize tension
(E) early experience is important in the formation of individual personality
II-95.Which of the following type A, or coronary-prone, behavioral factors appears to be the
most important?
(A) Hostility
(B) Competitiveness
(C) Time urgency
(D) Explosive speech
(E) Hyperactivity
II-96.Prominent symptoms associated with the psychiatric diagnosis of “adjustment disorder “ in
adults include all the following EXCEPT
(A) substance abuse
(B) anxiety
(C) irritability
(D) depression
(E) physical complaints
II-97.Which of the following methods of assessment would be most useful in the detection of
brain damage?
(A) Thematic apperception test
(B) Bender-Gestalt test
(C) California personality inventory
(D) Minnesota multiphasic personality inventory (MMPI)
(E) Milion behavioral health inventory ( MBHI)
II-98.All the following are basic principles of the neural basis of learning and memory EXCEPT
(A) memory has stages and is continually changing
(B) Long-term memory results in physical changes in the brain
(C) Memory traces are widely distributed throughout the nervous system
(D) The hippocampus and temporal lobes are most actively involved in the human memory
process
(E) Localized trauma can completely destroy stored memory
II-99.A behavior pattern that increases in frequency when followed by a reward is an example of
(A) classical conditioning
(B) shaping
(C) respondent conditioning
(D) operant conditioning
(E) generalization
II-100.In order for operant conditioning to be effectively applied to a psychiatric disorder (e.g.,
shouting obscenities), one can increase positive, constructive behavior by all the following
EXCEPT
(A) precisely defining desired behavior
(B) identifying an effective reinforcer
(C) providing reinforcement when desired behavior is emitted
(D) punishing undesired behavior
(E) exchanging a preferred food for desired behavior
II-101.In operant conditioning, the rate of extinction is most effectively slowed when the
response or learning has been maintained on a reinforcement schedule of
(A) fixed ratio
(B) variable ratio
(C) fixed interval reinforcement
(D) continuous reinforcement
(E) piecework reinforcement
II-102.Learned helplessness is associated with depression in that both are characterized by each
of the following EXCEPT
(A) continued failure from lack of ability
(B) passive behavior
(C) negative expectations
(D) hopelessness
(E) feelings of no control over events
II-103.The idea that experimental extinction, which is produced by non-reiforcement after
classical conditioning, is an active inhibitory process finds support in the phenomenon of
(A) generalization
(B) spontaneous recovery
(C) experimental neurosis
(D) passive avoidance
(E) active avoidance
II-104.All the following factors are critical in both classical and operant conditioning EXCEPT
(A) timing
(B) stimulus discrimination
(C) specific reflex responses
(D) extinction
(E) learning of predictive relationships
II-105.In classical conditioning, the “ partial reinforcement effect” is the
(A) inability to learn with fractional reward
(B) increased response retention following partial reinforcement
(C) increased conditioned response amplitudes after nonreinforced trials
(D) reduced response latencies on nonreinforced trials
(E) contrast effects for partial reinforcement schedules
II-106.A social learning approach to attitude change includes all the following methods EXCEPT
(A) modeling
(B) desensitization
(C) extinction
(D) direct reinforcement
(E) integration of nature (vs. Nurture)
II-107.A major distinction between the methods for producing classical and instrumental
conditioning is that in instrumental conditioning the reinforcing stimulus is
(A) always appetitive in nature
(B) contingent on the behavior to be learned
(C) invariant
(D) solely under the control of the experimenter
(E) unnecessary to produce learning
II-108.Correct statements regarding systematic desensitization include all the following
EXCEPT
(A) it was developed by Joseph Wolpe
(B) it is used to decrease neurotic anxiety or phobia
(C) anxiety-provoking situations are brought to mind to be resolved through reality therapy
(D) anxiety-provoking situations are paired with anxiety-reducing relaxation
(E) the patient is taught a technique of muscular relaxation
II-109.The term learning disabilities most often refers to problems in learning
(A) to do arithmetic
(B) to read
(C) to write compositions
(D) eye-hand coordination
(E) achievement orientation
II-110.The most important component of biofeedback
(A) feelings
(B) information
(C) motivation
(D) attitude
(E) interpersonal relations
II-111.In B.F. Skinner’s view, the study of behavior
(A) should be concerned with the fundamental motives that are found in all organisms
(B) should employ factor-analytic procedures to identify the basic dimensions of personality
(C) must be based entirely on an understanding of classical conditioning
(D) cannot make extrapolations about principles of learning from subhuman species to humans
(E) need not make inferences about unobservable organismic states and inner motives
II-112.Erik Erikson views the stage of young adulthood as being best described by a crisis of
(A) trust versus mistrust
(B) intimacy versus isolation
(C) identity versus role confusion
(D) initiative versus guilt
(E) autonomy versus dependence
II-113.All the following statements about aging are true EXCEPT
(A) the maximal life span has increased in populations over the past 20 years
(B) the risk of death increases exponentially with time
(C) life expectancy would increase only 20 years if heart disease and cancer were eliminated
(D) the biomedical model is becoming increasingly inapplicable in accounting for death in the
elderly
(E) the Vo2max can be used as an integrated measure of the functional limits of the whole body
II-114.The environmental factor with the most underused potential for developing cognitive and
interpersonal competence in children is the
(A) school
(B) family setting
(C) community mental health center
(D) health care system
(E) child welfare agency
II-115.Sequential studies of human development suggest that the growth of intelligence may
continue through
(A) late adolescence
(B) early adulthood
(C) middle adulthood
(D) late adulthood
(E) old age
II-116.In the theory developed by Jean Piaget, the first unit to appear in cognitive development is
(A) language
(B) schema
(C) rules
(D) images
II-117.All the following factors are included in gender identity EXCEPT
(A) gender role behavior
(B) parental and cultural attitudes
(C) external genitalia
(D) sexual behavior
(E) genetic influence
II-118.Which of the following statements about the development of moral standards during
childhood is true?
(A) Only at puberty do a child’s moral standards become independent of external rewards and
punishments
(B) The development of guild as a reaction to transgressions is fostered by parental warmth
(C) Children of 11 to 12 years of age are more likely to make inflexible, absolute moral
judgments than those 7 or 8 years of age
(D) Older children are more likely than younger children to judge behavior as right or wrong in
terms of its reinforcement outcomes
(E) The development of moral reasoning is independent of general intellectual maturation
II-119.Research findings on early gender-role identification and behaviors show that all the
following statements are true EXCEPT
(A) gender-related differences in play behavior are evident as early as 13 months
(B) males are generally more aggressive in their play and problem-solving activities than are
females
(C) children who experience gender reassignment after the age of 2 are considered to be high-
risk candidates for psychological disorders
(D) children between 3 and 4 years of age are able to make gender-appropriate choices according
to Western stereotypes
(E) females 3 to 5 years old appear to be more concerned about gender-appropriate play
activities than are males of a similar age
II-120.In Piaget’s theory of child development, the processes that allow a child to move from one
stage of cognitive development to the next include all the following EXCEPT
(A) reinforcement
(B) representational ability
(C) equilibration
(D) assimilation
(E) accommodation
II-121.Language development in childhood is characterized by which of the following ?
(A) Infants 15 to 20 weeks of age use different patterns of vocalization to identify discomforts
such as hunger, fright, and pain
(B) Most children master all consonant sounds by the age of 3 to 3½ years
(C) The first words spoken are usually verbs
(D) Vowels emerge in hierarchical order between age 3½ and 7 years
(E) None of the above
II-122.Intelligence and cognitive development during adolescence are usually characterized by
each of the following EXCEPT
(A) the emergence of hypothetico-deductive reasoning
(B) attitudes and reasoning that tend to be egocentric
(C) a growing capacity for probabilistic thinking
(D) a type of reasoning that subordinates reality to possibility
(E) less attention to and ability to infer what others are thinking and feeling
II-123.Perinatal gonadal hormones have a development impact on all the following EXCEPT
(A) sex-linked neural differentiation
(B) sexuality in adulthood
(C) neural morphology
(D) the permanent organization of parts of the nervous system
(E) hermaphroditism in genetic males
II-124.The view that leardership is a reciprocal process of social influence with leaders and
followers being influenced by each other is known as
(A) situational leadership
(B) contingency model
(C) social facilitation
(D) collective behavior model
(E) transactional leadership
II-125.Social support provides all the following EXCEPT
(A) a means for interpretation of situations
(B) a good substitute for engaging in health-promoting behaviors
(C) the ability for social comparison and validation
(D) models for behavior
(E) access to significant others
II-126.Persons who have social supports experience all the following results EXCEPT
(A) longer life
(B) less practice of denial
(C) fewer somatic illnesses
(D) more dependency
(E) fewer crises
II-127.Studies on attitude change show that people are LEAST strongly influenced by
(A) facts and information
(B) comparison of their own attitudes and beliefs with those of others
(C) their own personal and social needs
(D) the need to reduce unpleasant motivational states
(E) the attributes of the communicator who is recommending change
II-128.Most mental health care authorities agree that the recent trend toward community-based
treatment of patients who have emotional disorders, as opposed to their incarceration in public
mental institutions, can be ascribed principally to
(A) improved therapeutic results when treatment is available within a patient’s community
(B) the development of preventive measures for various mental illnesses
(C) the use of new drugs that allow many emotionally disturbed patients to function outside an
institution
(D) an increasing willingness by general hospitals to accept mentally disturbed patients
(E) the 1961 report of the Joint Commission on Mental Illness and-Health, which strongly
recommended community mental health programs
II-129.All the following statements about the development of independence during adolescence
are true EXCEPT
(A) sex differences in the age at which independence is achieved tend to be greater in societies
whose economy depends on the male’s physical strength and motor skills (e.g., hunting and
herding versus food-gathering cultures)
(B) compared with the majority of other cultures, independence training in contemporary
America begins relatively early but also is relatively protracted
(C) in general, the shift of parental attitudes toward permissiveness that occurs during
adolescence is greater in father than in mothers
(D) in general, authoritarian parental attitudes are viewed by adolescents as more appropriate in
fathers than in mothers
(E) confidence and self-esteem are enchanced by parental attitudes that encourage autonomy yet
express interest in an adolescent’s behavior and opinions
II-130.Ethnocentrism is the
(A) belief that ethnic groups should remain geograhically segregated
(B) tendency to think that one’s own group or way of life is preferable or superior
(C) belief that ethnic groups should not intermarry
(D) belief that values held by marriage partners should be in accord
(E) belief that multiple cultural practices should be incorporated into marriage and the family
II-131.Risk factors for problem behaviors in adolescence can be identified. All the following
personal or personality factors have a major influence EXCEPT
(A) unconventionality
(B) rebelliousness
(C) high risk-taking
(D) low value on achievement
(E) low value on autonomy
II-132.In studying human development, the remeasurement of a cross-sectional sample of people
after a given fixed interval of time has passed is called
(A) time-lag design
(B) sequential design
(C) multivariate design
(D) cross-sectional design
(E) longitudinal design
II-133.If one is conducting research that requires comparisons of content and material gathered
for certain patients by different interviewers, the best format to use would be one that is
(A) projective
(B) structured
(C) unstructured
(D) analytical
(E) none of the above
II-134.The revised third edition of Diagnostic and Statistical Manual of Mental Disorders (DSM
III-R) offers an improvement over earlier psychiatric classifications. Its major advantage is that it
(A) identifies mental illness as a disease
(B) deals with predisposing factors
(C) confirms and supports the nosologic system
(D) is an all-inclusive classification system
(E) is a multiaxial classification system
II-135.A sample of a population selected so that each observation or unit has equal chance of
being included is called a
(A) random population parameter
(B) systematic sample
(C) systematic random sample
(D) selected sample
(E) simple random sample
II-136.Two drugs were administered in several different doses, and the effects were measured
and recorded. The data were subjected to regression and correlation analyses .The computed
regression lines and correlation coefficients are presented below. Which of the following
statement is true ?
(A) In general, the strength of the causal relationship between two variables is indicated by the
square of the correlation coefficient
(B) The proportion of variation in response to Drug I accounted for by dose is (0.80)²
(C) The data indicate Drug 1 is more effective than Drug 2
(D) The data indicate Drug 1 is more potent than Drug 2
(E) There is more variability in responses to Drug 1 than to Drug 2
II-137. Which of the following statements about patients with affective disorders is true?
(A) the lifetime prevalence of unipolar disorder is approximately equivalent in men and women
(B) In bipolar disorder, episodes of elevated mood are more common than episodes of depression
(C) The lifetime prevalence of bipolar disorder is approximately equivalent in men and women
(D) Patients with bipolar disorder rarely have relatives with unipolar illness
(E) The concordance rate for unipolar disorder is higher than for bipolar disorder
II-138.Which of the following personality disorders has been linked to alcoholism in relatives?
(A) Histrionic
(B) Borderline
(C) Obsessive-compulsive
(D) Avoidant
(E) Antisocial
II-139.Which of the following statements about the incidence and origins of personality
disorders is true?
(A) high incidence of schizoid personality disorder occurs in families of patients with mood
disorders
(B) High incidence of schizotypal personality disorder occurs in families of patients with mood
disorders
(C) The concordance rate personality disorders is equivalent in monozygotic twins and in
dizygotic twins
(D) Evidence for genetic factors is seen in antisocial personality disorder
II-140.All of the following statements about the genetics of neuropsychiatric disorders are true
EXCEPT
(A) in cri du chat syndrome, part of chromosome 5 is missing
(B) a high concordance rate for Alsheimer’s disease is seen in twins
(C) an anomaly on chromosome 21 has been linked to Alzheimer’s disease
(D) lesch-Nyhan syndrome involves autosomal dominant transmission
(E) there is probably a genetic component in infantile autism
II-141.The second most common genetic cause of mental retardation is
(A) Huntington’s disease
(B) Cri du chat syndrome
(C) Klinefelter’s syndrome
(D) Fragile X syndrome
(E) Lesch-Nyhan syndrome
II-142.A 25 year old male patient sustains a serious head injury in an automobile accident. He
had been aggressive and assaultive, but after the accident he is placid and cooperative. He makes
conditional suggestive gestures and comments and masturbates a great deal. The patient’s injury
is most likely to affect what portion of the brain?
(A) Frontal lobes
(B) Temporal lobes
(C) Parietal lobes
(D) Occipital lobes
(E) Basal ganglia
II-143.Sleep-arousal mechanisms are effected by damage to the
(A) cerebellum
(B) basal ganglia
(C) thalamus
(D) reticular system
(E) amygdala
II-144.Increased pain perception is associated with damage to what area of the brain?
(A) Thalamus
(B) Corpus callosum
(C) Basal ganglia
(D) Reticular system
(E) Hypothalamus
II-145.Parkinson’s disease is associated with damage to what area of the brain?
(A) Cerebellum
(B) Hypothalamus
(C) Basal ganglia
(D) Frontal lobes
(E) Temporal lobes
II-146.What is the major area of the brain implicated in Alzheimer’s disease and amnestic
disorders?
(A) Amyglada
(B) Parietal lobes
(C) Hippocampus
(D) Thalamus
(E) Basal ganglia
II-147.All of the following statements about neurotransmission are true EXCEPT
(A) receptors are proteins in the membranes of neurons
(B) presynaptic receptors bind neurotransmitters
(C) postsynaptic receptors change the ionic condition of membranes
(D) neurotransmitters exert their effects only on postsynaptic neurons
(E) postsynaptic receptors bind neurotransmitters
II-148.Which of the following statements about neurotransmission is true ?
(A) Amino acid neurotransmitters account for 15% of the synapses in the human brain
(B) The biogenic amines are involved in 50% of the synapses in the brain
(C) Dopamine is a second messenger
(D) Diacylglycerol is a second messenger
(E) Reuptake of neurotransmitters by the presynaptic neuron not occur
II-149.Which of the following statements about dopamine is true?
(A) At autopsy, the brains of schizophrenics show decreased numbers of dopamine receptors
(B) Blockade of dopamine receptors results in decreased prolactin levels
(C) Antipsychotic drugs that block postsynaptic dopamine receptors may cause Parkinson-like
symptoms
(D) Dopamine enchances the release of prolactin
(E) Patients with mania show dopamine hypoactivity
II-150.Which of the following statements about GABA is true ?
(A) It is primarily an excitatory neurotransmitter
(B) It is an amino acid neurotransmitter
(C) It mediates postsynaptic inhibition in the CNS
(D) Increased GABA activity has been reported in Huntington’s disease GABA activity is seen
in epilepsy
II-151.All of the following statements about the endogenous opioids are true EXCEPT
(A) They are associated with effects of stress and pain
(B) Alterations in opioid levels may be involved in seizures
(C) They include the enkephalins
(D) They include glutamic acid
(E) Alterations in opioid levels may be involved in alcoholism
II-152.Which of the following neuropeptides has been implicated in the pathology of
schizophrenia?
(A) Somatostatin
(B) Neurotensin
(C) Substance P
(D) Vasopressin
(E) Vasoactive intestinal peptide
II-153.The left hemisphere of the brain is thought to be associated with which one of the
following functions?
(A) Perception of social cues
(B) Musical ability
(C) Spatial relations
(D) Language
(E) Facial recognition
II-154.All of the following structures connect the two cerebral hemispheres EXCEPT
(A) the corpus callosum
(B) the anterior commissur
(C) the hippocampal commissure
(D) the posterior commissure
(E) the amygdala
II-155.Which of the following statements about neurotransmitters and their metabolites is true?
(A) MHPG levels in urine are increased in patients with severe depressive disorders
(B) MHPG is increased in the cerebrospinal fluid of patients who have attempted suicide
(C) 5-HIIA is decreased in aggressive, violent individuals
(D) MHPG is a metabolite of serotonin
(E) VMA is a metabolite of serotonin
II-156.Blockade of H2 receptors is associated primarily with
(A) depression
(B) weight gain
(C) increased anxiety
(D) epilepsy
II-157.Degeneration of cholinergic neurons is involved primarily
(A) mania
(B) depression
(C) movement disorders
(D) anxiety
(E) sleep disorders
II-158.Which of the following statements about monoamines and their metabolites is true?
(A) Mania is associated with dopamine hypoactivity
(B) Free dopamine is metabolozed by 5-HIIA
(C) There is more plasma monoamine oxidase (MAO) in unmedicated schizophrenics than in
controls
(D) There is no association between levels of MHPG and suicidal behavior
(E) Decreased HVA is seen with clinical improvement in patients treated with neuroleptics
II-159.The antipsychotic drug producing the greatest sedative effect is
(A) haloperidol
(B) chlorpromazine
(C) perphenazine
(D) thioridazine
(E) thrifluoperazine
II-160.A 45-years-old male employed as an air traffic controller presents for treatment of
depression. Which of the following heterocyclic agents is least likely to cause sedation in this
patient ?
(A) Desipramine
(B) Imipramine
(C) Amitriptyline
(D) Doxepin
(E) Amoxapine
II-161.Which of the following statements about the characteristics of tardive dyskinesia is true?
(A) It is more common in men than in women
(B) It includes abnormal movements of the tongue
(C) It usually occurs within 3 months of beginning drug treatment
(D) It occurs more often in treatment using clozapine than treatment using other antipsychotic
agents
(E) It is a consequence of antidepressant drug treatment
II-162.Parkinsonian symptoms are associated mainly with which one of the following
antidepressants?
(A) Desipramine
(B) Imipramine
(C) Amitriptyline
(D)Amoxapine
(E) Nortriptyline
II-163.Which of the following statements about electroconvulsive therapy (ECT) is true?
(A) The most common indication is in the treatment of schizophrenia
(B) It involves induction of a generalized seizure
(C) It is associated with considerable risk
(D) It is an effective treatment for anxiety
(E) Maximum response is usually seen after two to four treatments
II-164.Which of the following statements about the characteristics of antidepressant agents is
true?
(A) They structurally resemble the anxiolytic agents
(B) They frequently induce euphoria in nondepressed patients
(C) They rarely have antcholinergic effects
(D) The heterocyclics block reuptake of serotonin at the synapse
(E) Acommon side effect of heterocyclic agents is weight loss
II-165.Which one of the following statements about MAO inhibitors is true?
(A) The inhibition of MAO is reversible
(B) MAO inhibitors are used to treat depression
(C) Adverse effects of MAO inhibitors are distinct from those of heterocyclic agents
(D) MAO-Bs are more likely than MAO-As to cause a hypertensive crisis
II-166.A patient with bipolar disorder is being treated with lithium. Which adverse effect is this
patient least likely to experience?
(A) Cardiac conduction abnormalities
(B) Gastric distress
(C) Tremor
(D) Mild cognitive impairment
(E) Food allergy
II-167.Antipsychotic agents are usually associated with all of the following adverse effects
EXCEPT
(A) orthostatic hypotension
(B) dry mouth
(C) amenorrhea
(D) weight loss
(E) breast enlargement
II-168.Which of the following antipsychotic agents is least likely to cause parkinsonian effect?
(A) Chlorpromazine
(B) Haloperidol
(C) Perphenazine
(D) Trifluoperazine
(E) Pimozide
II-169.The major method used to investigate the unconscious mental processes of a patient in
psychoanalysis is
(A) symbolism
(B) repression
(C) free association*
(D) Primary process
(E) Examination of the preconscious
II-170.All of the following statements about defense mechanisms are true EXCEPT
(A) they are used to keep conflicts out of consciousness
(B) they are unconscious mental techniques
(C) they decrease anxiety
(D) they are always manifestations of immature functioning *
(E) they help maintain an individual’s sense of self-esteem
II-171.Which of the following defense mechanisms is seen in a patient who exhibits multiple
personalities?
(A) Repression
(B) Displacement
(C) Dissociation
(D) Isolation
(E) Intellectualization
II-172.A physician who has been given a diagnosis of terminal pancreatic cancer constantly
discusses the technical aspects of his case with many of the other physicians in the hospital. This
is an example of which one of the following defense mechanisms?
(A) Repression
(B) Displacement
(C) Dissociation
(D) Regression
(E) Intellectualization
II-173.The defense mechanism used when unacceptable feelings are prevented from reaching
awareness is known as
(A) repression
(B) regression
(C) displacement
(D) rationalization
(E) isolation of affect
II-174.Which of the following defense mechanisms is classified as the most mature?
(A) Sublimation
(B) Repression
(C) Rationalization
(D) Projection
(E) Regression
II-175.A previously toilet-trained child is hospitalized and begins wetting the bed again. This is
an example of
(A) isolation of affect
(B) displacement
(C) projection
(D) regression
(E) denial
II-176.All of the following statements concerning psychoanalysis are true EXCEPT
(A) the therapist plays an active role
(B) treatment is usually conducted 4 times/week
(C) treatment is usually conducted for 3-4 years
(D) inhibition of free association is called resistance
(E) resistance blocks unconscious ideas that are unacceptable to the conscious mind
II-177.A patient’s reexperiencing of feelings about the parent with the therapist during
psychoanalysis is known as
(A) interference
(B) resistance
(C) association
(D) transference
(E) cognitive dissonance
II-178.Which of the following statements correctly describes aspects of Freud’s structural theory
of the mind?
(A) The id is conscious
(B) The mind is divided into the preconscious and conscious
(C) The superego is unconscious
(D) The id is closely associated with reality
(E) The ego is under the domination of primary process thinking
II-179.Which of the following is a function of the superego?
(A) Maintaining relationships to the outside world
(B) Reality testing
(C) Maintaining object relationships
(D) Controlling secondary process thinking
(E) Controlling id impulses
II-180.All of the following elements of classical conditioning must be learned EXCEPT
(A) the conditioned response
(B) the unconditioned response
(C) the conditioned stimulus
(D) the acquisition phase
(E) stimulus generalization
II-181.Which of the following schedules of reinforcement is most resistant to extinction?
(A) Continuous
(B) Fixed ratio
(C) Fixed interval
(D) Variable ratio
(E) Discontinued
II-182.A child who likes and looks up to her physician states that she wants to become a doctor
when she grows up. This behavior is an example of
(A) stimulus generalization
(B) modeling
(C) shaping
(D) positive reinforcement
(E) variable reinforcement
II-183.All of the following statements about learning are true EXCEPT
(A) learning involves the acquisition of behavior patterns
(B) classical conditioning is a method of learning
(C) operant conditioning is a method of learning
(D) history of the individual is important in all forms of learning
(E) stimuli may be internal or external to the individual
II-184.A 2-year-old child is afraid of nurses in white uniforms. When his grandmother comes to
visit him wearing a white jacket, he begins to cry. The best explanation for this phenomenon is
(A) stimulus generalization
(B) habit
(C) instrumental conditioning
(D) learning by trial and error
(E) an emitted operant
II-185.Which of the following statements about reinforcement in operant conditioning is true ?
(A) It establishes a connection between a stimulus and a response
(B) It can only be positive
(C) It decreases the rate at which a behavior occurs
(D) It precedes a behavior
(E) It was first described by the work of Pavlov
II-186.All of the following statements about biofeedback are true EXCEPT
(A) it is based on classical conditioning
(B) it has been used in the treatment of generalized anxiety disorder
(C) control over physiologic activity is learned
(D) it can be used to control autonomic activity
(E) relaxation of striated muscle can be achieved
II-187.All of the following factors are important in the successful use of biofeedback EXCEPT
(A) the patient must receive continuous information about the physical parameter
(B) the physical parameter must be detectable and measurable
(C) the patient’s motivation is important
(D) it is used to gain control over the central nervous system
(E) a large amount of practice is required
II-188.Although a mother slaps a child on the hand every time she touches a stove, the child
begins to touch the stove more frequently. This is an example of
(A) punishment
(B) negative reinforcement
(C) positive reinforcement
(D) aversive conditioning
(E) classical conditioning
II-189.Which of the following statements concerning narcolepsy is true?
(A) The REM sleep of a narcoleptic person is abnormal
(B) Narcolepsy occurs in approximately 50 out of every 10,000 persons
(C) Narcolepsy occurs most frequently in the elderly
(D) Sedatives are used to treat narcolepsy
(E) Cataplexy rarely occurs
II-190.Which neurotransmitter is particularly involved in creasing of REM sleep?
(A) Serotonin
(B) Norepinephrine
(C) Acetylcholine (ACh)
(D) Dopamine
(E) Histamine
II-191.Which of the following is characteristic of a patient in REM sleep?
(A) Hyperactivity of skeletal muscles
(B) Decreased blood pressure
(C) Penile and clitoral erection
(D) Decreased brain oxygen use
(E) Decreased pulse
II-192.Which of the following signs is characteristic of a patient in non-REM sleep?
(A) Paralysis of skeletal muscles
(B) Descreased blood pressure
(C) Increased pulse
(D) Increased respiration
(E) Agitation
II-193.A patient who is suffering from major depression has sleep problems. Which of the
following is likely to be found in this patient?
(A) Increased slow wave sleep
(B) Lengthened REM latency
(C) Reduced percentage of REM sleep
(D) Shift in REM from last to first part of the night
(E) Short first REM period
II-194.All of the following statements about hypersomnia are true EXCEPT
(A) it is associated with narcolepsy
(B) it is associated with sleep apnea
(C) it is associated with withdrawal of CNS stimulants
(D) it is seen frequently in patients with mania
(E) it includes complaints about somnolence
II-195.Slow wave sleep is characterized by
(A) penile erection
(B) dreaming
(C) night terrors
(D) total recall of the arousal
(E) increased brain oxygen use
II-196.Which of the following statements about sleep is true?
(A) Patients with Alzheimer’s disease have increased slow wave sleep
(B) Drugs that decrease brain dopamine produce wakefulness
(C) Patients with Alzheimer’s disease have reduced REM sleep
(D) Dopamine blockers decrease sleep time
(E) Increased noradrenergic activity is associated with increased REM sleep
II-197.Which of the following statements about patients with insomnia is most likely to be true?
(A) It occurs in less than 5% of the population
(B) It has little effect on daytime functioning
(C) It is rarely associated with anxiety
(D) It may be an early sign of severe depression
(E) It must occur daily for at least 6 months to be to be diagnosed as insomnia
II-198.Which of the following is characteristic of sleep in a person age 79?
(A) Increased total sleep time
(B) Decreased REM sleep
(C) Decreased sleep latency
(D) Increased stage 3 sleep
(E) Increased stage 4 sleep
II-199.Which of the following is used to test intelligence in children?
(A) Thematic Apperception Test (TAT)
(B) Minnesota Multiphasic Personality Inventory (MMPI)
(C) Wechsler Intelligence Scale for Children-Revised (WISC-R)
(D) Halstead –Reitan Battery (HRB)
II-200.An individual with a mental age of 8 and a chronological age of 10 has an IQ of
(A) 40
(B) 60
(C) 80
(D) 100
(E) 180
1. In classical conditioning (A)

a. subject is passive
b. responses are typically autonomic or emotional
c. continued pairings of a CS with UCS strengthen conditioned responses
d. responses are usually voluntary

2. Some applications of classical conditioning include (E)

a) Learned immune responses (Ader & Cohen, 1993)


b) Phobias
c) Systematic desensitization (Joseph Wolpe)
d) Predator Control

3. In Operant Conditioning: (B. F. Skinner, 1938) (E)

1. the organism is free to respond at any time


2. subject is active
3. likely to be using consciously controlled behaviours
4. stimulus generalization, and spontaneous recovery also
occur in operant conditioning

4. Social learning theory (B)

1.  is based on work by Albert Bandura


2. tends to focus on biological factors rather than nurture
3. cnceptualizes people as active
4. excludes desensitization

5. Sensory memory (A)

1. allows comparison of stimulus with LTM to assign significance


2. echoic (auditory) or iconic (visual)
3. fade/ loss time about 0.5 secs
4. capacity of about 7 2 items

1. The following names and terms are associated ( e)

1. Abnormal Illness Behaviour :Pilowsky


2. Advocated treatment without mechanical Restraints: Conolly (1794-
1866)
3. Aggressive and Creative Psychopaths :Henderson
4. Agnosia Term coined by Freud

2. Some applications of classical conditioning include (e)

1. learned immune responses (Ader & Cohen, 1993)


2. phobias
3. systematic desensitization (Joseph Wolpe)
4. predator Control
3. In Operant Conditioning: (B. F. Skinner, 1938) (b)

1.  the organism learns a response by operating on the environment


2. subject is passive
3. likely to be using consciously controlled behaviors
4. stimulus generalization, discrimination, extinction and spontaneous
recovery
do not occur in operant conditioning

4. Functions in observational learning include: (e)

1. attention to relevant aspects of model’s behavior


2 visual image of model
3. remembering/ rehearsal of behavior
4. anticipation of consequences

5. Social learning theory .. ©

1. isbased on work by Freud


2. isoriginally applied to attempts to integrate psychoanalysis and learning
theory –
3. tends to focus on nature rather than social factors
4. Conceptualizes people as active

6. Principles that describe how the perceptual system organizes stimuli include: (e)

1. proximity
2. similarity
3. continuity
4. closure

7. long term memory includes: (a)

1. episodic (memories of specific events)


2. procedural (the memory of how to do things)
3. semantic (generalized knowledge about the world)
4. sensory memory

8. Procedural memory (implicit) includes: (b)

1. motor skills
2. emotional events
3. intuitive cognitive skills
4. factual knowledge

9. The following are true regarding Serotinergic system and memory (e)

1. ACh release is under inhibitory 5-HT tone


2. 5-HT inhibition or destruction of 5-HT cells increases ACh
release in cortex, hippocampus and striatum
3. m-chlorophenylpiperazine (mCPP), a 5-HT1C agonist impairs
cognition
4. ONDANSETRON, a selective 5-HT3 receptor antagonist improves
cognition in animals

10. Causes of slower speech development include ©

1. being female
2. being a twin
3. shortened second-stage labour
4. larger family size

11. According to Freud the structural model of personality includes (d)

1. unconscious
2. preconscious
3. conscious
4. id

12. The trait theories make the following basic assumptions: (a)

1. personality traits are relatively stable over time


2. personality traits are relatively stable across diverse situations
3. people differ with regard to how much of a particular trait they possess
4. personality traits are totally not stable

13. The following are true regarding Long-term potentiation (LTP) (a)

1. the long-lasting increase in the efficiency of a single set of synapses


2. postulated that it could be the substrate for associative learning
3. depends on the activation of NMDA receptors in the hippocampus
4. depends on the activation of 5-HT receptors in the cortex

14. The following are true regarding Evoked brain potentials (a)

1. the evoked brain potential is a small, temporary change in voltage on an EEG that
occurs in response to specific events
2. there is a negative peak (N100) followed by a large positive peak (P300)
3. the exact timing of P300 is sensitive to factors that alter the speed of perceptual
processes
4. the evoked brain potential is a large, temporary change in voltage on an EEG that
occurs in response to non-specific events

15. High scorers on Neuroticism are: (d)

1. reliable
2. efficient
3. dependable
4. moody

16. The “big-five “ model of personality includes the following dimensions: (e)

1. The Openness to experience


2. Conscientiousness
3. Extraversion
4. Neuroticism

17. According to Maslow’s hierarchy, the Safety needs include (b)

1. being cared for as a child


2. belongingness and love
3. secure income as an adult
4. being part of various social groups

18. Hungers for different types of food seem to be related to the action of
different neurotransmitters on PVN neurons (e)

1. neuropeptide Y stimulates carbohydrate eating


2. serotonin reduces carbohydrate consumption
3. galanin motivates eating of high fat food
4. enterostatin reduces fat consumption

19. Physiological Stress Responses include: (e)

1. increased titres to latent viruses (e.g. herpes)


2. reduced activity of natural killer cells
3. reduced lymphocyte toxicity
4. reduced response of T-lymphocytes to mutagens

20. Problem-focused coping includes : a

1. confronting
2. seeking social support
3. planful problem solving
4. distancing

21. Emotion-focused coping includes: e

1. self-controlling”
2. distancing
3. positive reappraisal
4. accepting responsibility

22. Stress-related health problems are more common among people who: a

1. persist at mentally evading stressors


2. perceive them as long-term, catastrophic threats they brought on themselves
3. are pessimistic about their ability to overcome negative situations
4. are optimistic about their ability

23. Sensorimotor stage (birth - 2 years ) involves: e

1. mental representations
2. object permanence
3. primary circular reactions
4. secondary circular reactions

24. In Preoperational stage children: a


1.  can think in images and symbols
2. acquire language
3. play games of pretend
4. areable to make general, logical statements

25. Stage of Concrete Operations ( 7-11 years) include: e

1. understanding of logical principles that apply to external objects


2. conservation of length & volume
3. reversibility
4. sort objects into categories - classification

26. Normal attachment behavior comprises: c

1. crying when mother enters the room


2. calling for her or crawling after her
3. avoiding hard when anxious
4. talking and playing more in her company

27. Effects of maternal deprivation include: a

1. poor growth
2. developmental language delay
3. indiscriminate affection seeking
4. affectionate relationship

28. Children with difficult temperament show: d

1. negative, withdrawing responses to novelty


2. slow to adapt to change
3. predominantly negative mood
4. regular rhythmicity

29. Slow to warm up children exhibit : b

1. negative responses to change or novelty


2. positive approaches to new situations
3. slow adaptability
4. intense emotional reactions

30. Authoritarian parents : e

1. tend to be strict, punitive, and unsympathetic


2. do not encourage independence
3. are detached and seldom praise their children
4. theirchildren tend to be unfriendly, distrustful, and withdrawn

31. Fixed actions patterns include: a

1. smiling
2. brow flash response
3. expressions of joy, anger, fear, disgust
4.  previous learning for expression

32. The structure of attitudes consist of the following components: a

1. cognitive (beliefs)
2. affective (feelings)
3. behavioral (actions)
4. physiological

33. Attitude-behavior consistency depends on: e

1. accessibility of evaluations in memory


2. subjective norms
3. perceived control over the behavior
4. prior direct experience with the attitude object

37. Obstruction b

1. consists of irregular hindrance and sudden arrest of fluent movement, like a spoke stuck
in a wheel
2. consists of slow arrest of fluenent movement
3. is seen in catatonia
4. is usually seen in OCD

38. Mannerisms are a

1. abnormal,
2. repetitive,
3. goal-directed movements
4. commonly seen in chronic delusional disorder

39. Perseveration includes c

1. echolalia
2. palilalia
3. echopraxia
4. logoclonia

40. Organic brain lesions that cause stupor include: e

1. diencephalon and upper brain stem


2. frontal lobe
3. basal ganglia
4. the ‘locked in’ syndrome is due to lesions in the ventral pons

41. Constructional apraxia a

1. isinability to copy designs and structures


2. istwice as frequent in right- as in left-sided lesions
3. isclosely associated with visuospatial agnosia
4. is always associated with bilateral occipital lesions
42. Sensory distortions include e

1. hyperaesthesias
2. hypoaesthesias
3. changes in quality
4. dysmegalopsia

43. Sensory deceptions include a

1. illusions
2. hallucinations
3. pseudohallucinations
4. delusions

44. Gustatory hallucinations occur in: e

1. schizophrenia
2. temporal lobe epilepsy
3. lithium carbonate
4. disulfiram

45. Autoscopic (heautoscopic) hallucinations ccur in: b

1. depressive illness
2. OCD
3. temporal lobe epilepsy (in 30%)
4. delusional disorder

46. Characteristics of pseudohallucinations include: c

1.  they have the substantiality of normal perception


2. located in subjective, rather than objective space
3. willed, and subject to conscious control or manipulation
4. retention of insight

47. Approximate answers are seen in: c

1. depression
2. hysterical pseudodementia
3. panic disorder
4. Organic brain disease

48. Pseudologia fantastica are: d

1. seen in paranoid personality disorder


2. seen in schizoid personality disorder
3. seen in dependent personality disorder
4. seen in histrionic/ asocial personality disorder

50. Normal thinking (Fish, 1967) includes: a


1. Dereistic Thinking (daydreams)
2. imaginative thinking
3. rational thinking
4. irrational thinking

51. Circumstantial thinking occurs in: a

1. epilepsy
2. mental retardation
3. obsessional personality
4. schizoid personality

52. The borderline syndrome is characterized by: A

1. splitting
2. primitive idealization
3. feeling of emptiness
4. euphoric affect

53. Somatic symptoms of severe anxiety include:A

1. impotence
2. diarrhea
3. constipation
4. hypoventilation

54. Symptoms of combat neurosis include: D

1. grandiose ideas
2. good interpersonal relationships
3. lack of guilt
4. flashbacks

55. Autochthonous delusions are: B

1. synonymous with primary delusions


2. rarely preceded by a delusional atmosphere
3. a source of secondary delusion
4. pathognomonic of schizophrenia

56. The feotal alcohol syndrome:B

1. causes hydronephrosis
2. causes severe mental retardation
3. causes cleft lip and palate
4. is associated with liver abnormalities

57. Normal experiences include: B

1. jamais vu
2. delesional perception
3. derealization
4. visiual hallucinations

58. In Briquet's syndrome: D

1. There is usually an organic basis


2. recurrence is unusual
3. the prognosis is excellent
4. somatic complaints are usually multiple

59. Dissociative states:A

1. occur in hysteria
2. may be seen under hypnosis
3. include fugue states
4. exclude multiple personalties

60. Near-death experiences are:E

1. associated with the name Moody


2. seen in life-threatening experiences
3. feature of altered states of consciousness
4. occurrences in life or death

61. Delusional perception:A

a. has two stages


b. is an autochthonous delusion
c. is often preceded by 'delusional mood'
d. occurs secondary to a hallucination

62. Pseudohallucinations:D

1. are dependent on environmental stimuli


2. may occur in real world
3. may possess the vivid quality of normal perceptions
4. arise in inner space

63. The dysmnesic syndrome occurs in: A

1. Korsakoff's psychosis
2. mamillary body lesions
3. thalamic lesions
4. uncomplicated psychosis

64. Psuedodementia may be characterized by:B

1. onset with depressive features


2. abnormal EEG
3. past or family of manic-depressive psychosis
4. chronic course
65. Mesencephalon includes : a

1. Tectum
2. Basis Pedunculi
3. Tegmentum
4. hypothalamus

66. Dopaminergic cell bodies are situated in:  a

1. ventral tegmental area T


2. Substantia nigra
3. Arcuate nucleus of hypothalamus T
4. Median raphe nucleus F

67. Telencephalic structures include: d  

1. Oculomotor nerves F
2. Crura cerebri F
3. Optic nerves F
4. Cerebral hemispheres T

68. Functions of astrocytes might include: c  

A. Myelin sheath production F


B. Filling the role of fibrous tissue T
C. Lining the cerebral ventricles F
D. Most numerous of the glial cells T
F

69. Components of the basal ganglia may include: b  

1. Amygdaloid nucleus T
2. Dentate nucleus F
3. Caudate nucleus T
4. Subthalamic nucleus
F
70. Weaknesses for cohort studies include: e

1. · unsuitable for rare diseases


2. · expensive
3. · long delay before availability of results
4. · losses to follow-up can affect validity

71. Strengths for case control studies include e

1. · suitable for rare diseases


2. · better for studying causes in the distant past
3. · can examine many risk factors for a single disease
4. · quick and cheap

72 . HIV infection b
1. is greatly feared by the culture in which we live.
2. is commonly transmitted by such body fluids as saliva, tears, or urine
3. has infected between 20 and 60 % of all onmonogamous sexually active gay
men in the United States
4. can cause depression but does not cause dementia
73. Non-parametric methods a

1. do not require precise values, only their relative ranks


2. are less powerful
3. are less dependent on sample size
4. require large sample size

74. The following are differences between illness and disease: disease d

1. emphasizes the effect of poor health on social functioning


2. focuses on chronic rather than acute medical problems
3. is not well explained by the biopsychosocial model
4. represents objective pathology rather than a subjective experience of sickness

75. . The following factors can play a role in shaping the course of an illness e

1. the presence of an environmental pathogen


2. cultural and religious beliefs
3. stressful experiences early in life
4. the presence of a supportive environment

76 . The mutual participation model of the physician-patient relationship c

1. is based on the principle of beneficient paternalism


2. has become more important as patients are increasingly seen as consumers
3. was proposed by Talcott Parsons
4. is most useful in the treatment of chronic illnesses

77. The principle of learned helplessness c

1. was proposed by Sigmund Freud


2. describes the response to an inescapable or uncontrollable aversive event
3. describes people with an internal locus of control
4. may be operational in some abused children or spouses

78. Deficits in executive functioning c

1. are common in males who exhibit type A behavior.


2. can result from injuries to the frontal lobe
3. are always associated with deficits in traditional intelligence tests
4. can be associated with poor judgment and personality changes

79. Punishment b

1. is an aversive stimulus
2. is a long term effective deterrent to aggressive behavior
3. reduces the probability of a behavior’s recurring in the short run
4. is a negative reinforcement
80. Placebos b

1. are effective in about 35 % of patients


2. are effective in about 15 % of patients
3. can lower a patient’s blood glucose level
4. are more effective in patients who have a placebo personality

81. Lack of treatment adherence e

1. has been found in 38 to 95 % of patients with hypertension


2. is frequently underestimated by physicians
3. is more of a problem when disorders don’t cause obvious symptoms
4. can be reduced by simplifying medication regimens

82.The important determinants of maternal behavior in humans include: C

1. oestrogen
2. early experience of the mother
3. progesterone
4. characteristics of the neonate

83. Corticotrophin releasing hormone ( CRH): a

1. is found in cerebral cortex


2. increase blood pressure
3. increases plasma glucose levels
4. increases sexual behavior

84. The parasympathetic nervous system mediates: a

1. peristalsis
2. salivary secretion
3. micturition
4. sweating

85. Catecholamines c

1. include acetylcholine
2. are synthesized from phenylalanine
3. include serotonin
4. are monoamines

86. Characteristic features of Gerstmann's syndrome include: d

1. perseveration
2. retrograde amnesia
3. hyperreflexia
4. dyscalculia

87. Recognized side – effects of tricyclic antidepressants include: d

1. hypothyroidism
2. cataract
3. coarse tremor
4. ileus

88. Beta-endorphin is: b

1. released from pituitary at times of stress


2. a glycolipid
3. involved in the perception of pain
4. not a neurotransmitter

89. Cerebral lesions are associated with : C

1. festinant gait
2. ipsilateral dysdiadochokinesia
3. bradykinesia
4. ipsilateral intention tremor

90. The following statements are true: a

1. . In an acid pH, basic drugs will be poorly absorbed


2. . Gastric emptying is delayed by MAOIs
3. . Food increases the absorption of diazepam
4. . Rectal administration results in extensive 1st pass metabolism

91. The following statements about the distribution of neurotransmitters are


correct: b
 
1. Acetylcholine is found in the basal ganglia
2. Dopamine cell bodies are found in the limbic system
3. 5-HT predominates in the raphe nuclei in the brainstem
4. GABA is found in the peri-aqueductal grey matter

92. Regarding Benzodiazepines: d  

1. Tolerance develops in 4-6 weeks of therapy


2. BZDs are effective in phobic states
3. Zopiclone and Zolpidem act upon the BZ 2 receptor in a similar way to BZDs
4. Withdrawal symptoms peak at 7-8 days

93. Regarding SSRIs: c  


1. They reach peak levels within 1 hour of ingestion
2. Long-term use results in reduced 5-HT2 function
3. Fluoxetine has a half-life of around 7 days
4. OCD is an indication for their use

94. Common side effects of SSRIs include: e  


1. Diarrhoea
2. Constipation
3. Loss of appetite
4. Tremor

95. Functions of the limbic system may include: e  


1. Emotional behaviour T
2. Motivation T
3. Sexual activity T
4. Conditioned reflexes T
T
96. Functions of the reticular formation include: b  
1. Arousal T
2. Principal input to basal ganglia F
3. Sleep T
4. Vigilance

97. Possible components of the cerebellum include: c  


1. Caudate nucleus F
2. Dentate nucleus T
3. Lentiform nucleus F
4. Vermis T
98. The following are associated with wakefulness: a 

1. Hypothalamus
2. TRH
3. Noradrenaline
4. 5-HT

99. The following are true: c


 
1. Sleep spindles occur during stage 1
2. K complexes occur during stage 2
3. Nocturnal enuresis is most likely during stage 2
4. Delta waves can be seen during stages 3 and 4

100. Narcolepsy: e
 
1. Commonly involves sleep paralysis
2. Hypnagogic hallucinations are common
3. The condition often runs in families
4. REM latency at night is frequently seen

You might also like